SSC CGL Tier-I 06 March 2020 Shift-II Previous Year Paper

SSC CGL 6th March 2020 Shift-II

Q. 1 Select the option that is related to the third word in the same way as the second word is related to the first word. 

Crop : Farmer:: Food : ? 

(A) Vegetables 

(B) Chef 

(C) Utensil 

(D) Kitchen 

Answer: (B) 

Explanation: 

Farmers prepare the crop similarly, chef prepare the food. 

∴ The correct answer is option (B) 

 

Q. 2 Select the letter-cluster that can replace the question mark (?) in the following series. 

qDx, nHs, kLn, hPi, ? 

(A) EtD 

(B) fTe 

(C) eTd 

(D) eSd 

Answer: (C) 

Explanation: 

The series follows pattern as, 

(q – 3)((D) + 4)(x – 5) = nHs 

(n – 3)(H + 4)(s – 5) = kLn 

(k – 3)(L + 4)(n – 5) = hPi 

(h – 3)(p + 4)(i – 5) = eTd 

∴ The correct answer is option (C) 

 

Q. 3 If the following words are arranged as per their order in the English dictionary, which word will come fifth in the sequence? 

1. Weekly 

2. Weather 

3. Weird 

4. Weaver 

5. Weight 

6. Wielded 

(A) Weight 

(B) Weird 

(C) Wielded 

(D) Weekly 

Answer: (B) 

Explanation: 

Arrangement of the word according to the English dictionary, 

Weather, Weaver, Weekly, Weight, Weird, Wielded 

So, word which comes fifth number = Weird 

∴ The correct answer is option (B) 

 

Q. 4 Select the cube that can be formed by folding the given sheet (unfolded box) along the lines. 

Answer: (C) 

 

Q. 5 Four number-pairs have been given, out of which three are alike in some mannerandoneis different. Select the number-pair that is different. 

(A) 16, 8 

(B) 36, 2 

(C) 14,8 

(D) 28, 5 

Answer: (D) 

Explanation: 

16 + 8 = 24 

36 + 2 = 38 

14 + 8 = 22 

28 + 5 = 33 

Except option D, in the remaining options have the sum of the number is even. 

∴ The correct answer is option (D) 

 

Q. 6 The sequence of folding a piece of paper and the manner in which the folded paper has been cut is shown in the following figures. How would this paper look when unfolded? 

Answer: (A) 

 

Q. 7 Which of the following sets is best represented by the given Venn diagram. 

(A) House, Window, Tenant 

(B) Paragraph, Sentence, Pen 

(C) Weeks,Years,Months 

(D) Truck,Ships,Transport 

Answer: (C) 

 

Q. 8 How many triangles are present in the given figure? 

(A) 18 

(B) 23 

(C) 22 

(D) 20 

Answer: (C) 

 

Q. 9 The two given expressions on both the side of the ‘=’ sign will have the same value if two numbers from either side or both side are interchanged. Select the correct numbers to be interchanged from the given options. 

4 + 6 × 7 − 27 ÷ 3 = 7 × 8 − 4 + 39 ÷ 3 

(A) 8, 7 

(B) 6, 8 

(C) 3,4 

(D) 6,4 

Answer: (B) 

Explanation: 

4 + 6 × 7 − 27 ÷ 3 = 7 × 8 − 4 + 39 ÷ 3 

After interchanged, 

4 + 8 × 7 − 27 ÷ 3 = 7 × 6 − 4 + 39 ÷ 3 

4 + 8 × 7 − 9 = 7 × 6 − 4 + 13 

4 + 56 − 9 = 42 − 4 + 13 

51 = 51 

∴ The correct answer is option (B) 

 

Q. 10 Select the option that is related to the third number in the same way as the second number is related to the first number and the sixth number is related to the fifth number. 

78 : 117 :: 90 : ? :: 108 : 162 

(A) 144 

(B) 126 

(C) 117 

(D) 135 

Answer: (D) 

Explanation: 

78 + 78/2 = 78 + 39 = 117 

108 + 108/2 = 108 + 54 = 162 

Similarly, 

90 + 90/2 = 90 + 45 = 135 

∴ The correct answer is option (D) 

 

Q. 11 In a certain code language, BREATHER is coded as 29512859. How will AVIATION be coded as in that language? 

(A) 14912965 

(B) 14912956 

(C) 12912954 

(D) 12910965 

Answer: (A) 

Explanation: 

B = 2 

R(18) = 1 + 8 = 9 

E = 5 

A = 1 

T(20) = 2 + 0 = 2 

H = 8 

E = 5 

R(18) = 1 + 8 = 9 

Similarly, 

A = 1 

V(22) = 2 + 2 = 4 

I = 9 

A = 1 

T(20) = 2 + 0 = 2 

I = 9 

O(15) = 1 + 5 = 6 

N(14) = 1 + 4 = 5 

‘AVIATION’ is coded as ‘14912965’. 

∴ The correct answer is option (A) 

 

Q. 12 Which of the option figures is the exact mirror image of the given figure when the mirror is held at the right side?

Answer: (A) 

 

Q. 13 Select the set of letters that when sequentially placed in the blanks of the given letter series will complete the series. 

_m_ln_mlmnk_k_nl 

(A) l,n,k,l,n 

(B) k,n,k,l,m 

(C) k,l,k,l,m 

(D) m,n,k,l,m 

Answer: (B) 

Explanation: 

The series as, kmnl/nkml/mnkl/kmnl 

∴ The correct answer is option (B) 

Q. 14 A + B means ‘A is the father of B’: 

A − B means ‘A is the sister of B’; 

A × B means ‘A is the brother of B’; 

A ÷ B means ‘A is the daughter of B’. 

If, R + S × T − V ÷ U, then how is S related to U? 

(A) Son 

(B) Daughter 

(C) Brother 

(D) Husband 

Answer: (A) 

Explanation: 

In the diagram, Circle shows the female, square shows the male, vertical lines show the generation, single horizontal line shows the brother/sister and double horizontal lines show the couple. 

From the diagram, 

‘S’ is the son of ‘U’. 

∴ The correct answer is option (A) 

 

Q. 15 Select the number that can replace the question mark (?) in the following series. 

11, 12, 36, 41, ?, 296 

(A) 205 

(B) 287 

(C) 48 

(D) 123 

Answer: (B) 

Explanation: 

The series follows pattern as, 

11 + 1 = 12 

12×3 = 36 

36 + 5 = 41 

41×7 = 287 

∴ 287 + 9 = 296

∴ The correct answer is option (B) 

 

Q. 16 Four words have been given, out of which three are alike in some manner, while one is different. Select the odd word 

(A) Copper 

(B) Iron 

(C) Brass 

(D) Silver 

Answer: (C) 

Explanation: 

All the others are pure metal except brass as we know brass is an alloy of copper and zinc, in proportions which can be varied to achieve varying mechanical and electrical properties. 

 

Q. 17 Study the given pattern carefully and select the number that can replace the question mark (?) in it.

(A) 14 

(B) 16 

(C) 10 

(D) 18 

Answer: (B) 

Explanation: 

(15 – 14)×10 = 1 × 10 = 10 

(8 – 6)×7 = 2 × 7 = 14 

(25 – 12)×5 = 13 × 5 = 65 

Similarly, 

(7 – 3)×4 = 4 × 4 = 16 

∴ The correct answer is option (B) 

Instructions 

Read the given statements and conclusions carefully. Assuming that the information given in the statements is true,even if it appears to be at variance with commonly known facts, decide which of the given conclusions logically follow from the statements. 

 

Q. 18 Statements: 

1. All books are magazines. 

2. Some books are novels. 

Conclusions: 

I. Some magazines are books. 

II. No magazine is a book. 

III. No novel is a magazine. 

IV. Some novels are books. 

(A) Only conclusionsI, III and IV follow. 

(B) Only conclusions I and IV follow. 

(C) Only conclusions II and III follow. 

(D) Only conclusions I and II follow. 

Answer: (B) 

Explanation: 

From the Venn diagram, 

Conclusion I and IV are follow. 

∴ The correct answer is option (B) 

Instructions 

For the following questions answer them individually 

 

Q. 19 Select the option figure in which the given figure is embedded(rotation is NOT allowed).

Answer: (B) 

 

Q. 20 In a certain code language, ‘VERBAL’ is written as ‘QNMYXK’ and ‘REACH?’is written as ‘MNXZF’. How will ‘BRAVE’ be written as in that language? 

(A) KQNMX 

(B) YNXQM 

(C) YMXQN 

(D) KMQXN 

Answer: (C) 

Explanation: 

V is written as Q, 

E is written as N, 

R is written as M, 

(B) is written as Y, 

(A) is written as X, 

L is written as K, 

And 

R is written as M, 

E is written as N, 

(A) is written as X, 

(C) is written as Z, 

H is written as F, 

Similarly, 

‘BRAVE’ is written as ‘YMXQN’ 

∴ The correct answer is option (C) 

 

Q. 21 A group of three friends, K, L and M, are sitting in a cafe. Their average age is 24 years. Another friend ‘N’ joins the group and the new average age of the group becomes 23 years. If another friend ‘R’, whose age is 2 years more than that of ‘N’, replaces ‘K’, then the average age of L. M, N and R becomes 22.5 years. What is the age of K? 

(A) 15 years 

(B) 10 years 

(C) 12 years 

(D) 14 years 

Answer: (D) 

Explanation: 

Average of K, L and M = 24 

Sum of the age of K, L and M = 24×3 = 72 

Average of K, L, M and N = 23 

Sum of the age of K, L, M and N = 23×4 = 92 

Age of N = 92 – 72 = 10 

Age of R = 2 + Age of N = 2 +10 = 12 

Average age of L, M, N and R = 22.5 years 

Sum of age of L, M, N and R = 22.5×4 = 90 

Sum of age of L, M and N = 90 – 12 = 78 

Age of K = sum of the age of K, L, M and N – sum of age of L, M and N = 92 – 78 = 14 

 

Q. 22 Four Letter-clusters have been given, out of which three are alike in some manner, while one is different. Select the odd letter-cluster. 

(A) SWIK 

(B) VINE 

(C) NOHA 

(D) LAIM 

Answer: (A) 

Explanation: 

Except option (A) remaining all have 2 vowels. 

∴ The correct answer is option (A) 

 

Q. 23 Select the figure that can replace the question mark (?) in the following series. 

Answer: (A) 

 

Q. 24 Select the option in which the numbers are related in the same way as are the numbers in the given set.

(343, 98, 21) 

(A) (217, 72, 18) 

(B) (1728, 288, 24) 

(C) (1331, 242, 33) 

(D) (512, 126, 27) 

Answer: (C) 

Explanation: 

In (343, 98, 21), 

(7)3 = 343 

(7)2 × 2 = 98 

7×3 = 21 

Similarly, 

In (1331, 242, 33), 

(11)3 = 1331 

(11)2 × 2 = 242 

11× 3 = 33 

∴ The correct answer is option (C) 

 

Q. 25 Select the combination of letters that when sequentially placed in the blanks of the given series will complete the series. _uvwuv_wvtuw_uvwuv_wv_uw 

(A) t t t t t 

(B) t u t u t 

(C) t v w t t 

(D) t t u t v 

Answer: (A) 

Explanation: 

Sequence of the letters,  tuvwuvt/wvtuw/tuvwuvt/wvtuw 

∴ The correct answer is option (A) 

General Awareness 

Instructions 

For the following questions answer them individually 

 

Q. 26 Which of the following states does NOT share its boundary with Bhutan? 

(A) West Bengal 

(B) Meghalaya 

(C) Sikkim 

(D) Arunachal Pradesh 

Answer: (B) 

 

Q. 27 The first electron shell which is the nearest to the nucleus never holds more than ‘n’ electrons, where ‘n’ is equal to: 

(A)

(B)

(C)

(D)

Answer: (D) 

 

Q. 28 With which sport is Archana Kamath associated? 

(A) Lawn Tennis 

(B) Squash 

(C) Table Tennis 

(D) Badminton 

Answer: (C) 

 

Q. 29 In the context of rivers and their tributaries, which of the following pairs is correct? 

(A) Krishna – Manjra 

(B) Godavari – Kabini 

(C) Indus – Subansiri 

(D) Alaknanda – Pindar 

Answer: (D) 

 

Q. 30 Which of the following is a heritage site of Madhya Pradesh? 

(A) Lepakshi 

(B) Bhimbetka 

(C) Sasaram 

(D) Hampi 

Answer: (B) 

 

Q. 31 The Arid Forest Research Institute is located in ……. 

(A) Jodhpur 

(B) Jaipur 

(C) Ahmedabad 

(D) Dehradun 

Answer: (A) 

 

Q. 32 Which female Indian cricketer won the BCCI C.K. Nayudu Lifetime Achievement Award for 2019? 

(A) Shantha Rangaswamy 

(B) Diana Edulji 

(C) Anjum Chopra 

(D) Jhulan Goswami 

Answer: (C) 

 

Q. 33 Which ISO certification pertains to Environmental Management Systems? 

(A) 9001 

(B) 27001 

(C) 14001 

(D) 22000 

Answer: (C) 

 

Q. 34 Who is the author of the book ‘Hazaar Chaurasi Ki Maa’? 

(A) Krishna Sobti 

(B) Sugathakumari 

(C) Rita Kothari 

(D) Mahasweta Devi 

Answer: (D) 

 

Q. 35 In January 2020, with which country did India sign a MoU for content exchange programme between All India Radio and Betar? 

(A) Sri Lanka 

(B) Bangladesh 

(C) Bhutan 

(D) Nepal 

Answer: (B) 

 

Q. 36 The SASTRA – Ramanujan Prize is awarded in the field of: 

(A) Dance 

(B) Chemistry 

(C) Literature 

(D) Mathematics 

Answer: (D) 

 

Q. 37 Under which Article can the Parliament amend the Constitution? 

(A) Article 269 

(B) Article 74 

(C) Article 374 

(D) Article 368 

Answer: (D) 

 

Q. 38 The Act that transferred the power from the British East India Company to the British Crown in India was: 

(A) Government of India Act, 1835 

(B) Government of India Act, 1947 

(C) Government of India Act, 1833 

(D) Government of India Act, 1858 

Answer: (D) 

 

Q. 39 In July 2019, which among the following operations was launched by the BSF to fortify the Pakistan border in Punjab and Jammu? 

(A) Chakravyuh 

(B) Garuda 

(C) Vayuputra 

(D) Sudarshan 

Answer: (D) 

 

Q. 40 Which of the following pairs is associated with the taxation system of the Marathas? 

(A) Polaj and Parauti 

(B) Iqta and Jagir 

(C) Chauth and Sardeshmukh 

(D) Zat and Sawar 

Answer: (C) 

 

Q. 41 How many degrees does the Earth rotate about its own axis in one hour? 

(A) 10 

(B) 20 

(C) 15 

(D) 24 

Answer: (C) 

 

Q. 42 Under which of the following Amendments to the Constitution of India is defection to another party after election made illegal? 

(A) 92nd 

(B) 61st 

(C) 52nd 

(D) 86th 

Answer: (C) 

 

Q. 43 In which year was NABARD established? 

(A) 1979 

(B) 1978 

(C) 1982 

(D) 1981 

Answer: (C) 

 

Q. 44 Which among the following is the food tube? 

(A) Thymus 

(B) Aorta 

(C) Larynx 

(D) Oesophagus 

Answer: (D) 

 

Q. 45 Which of the following monuments is NOT a part of the Qutub Complex? 

(A) Quwwat-ul-Islam Mosque 

(B) Qutub Minar 

(C) Buland Darwaza 

(D) Alai Darwaza 

Answer: (C) 

 

Q. 46 Which of the following statements is correct with reference to the Karachi Session of the Congress (1931)? 

(A) Mahatma Gandhi presided over the session 

(B) The Gandhi-Irwin Pact was ratified 

(C) The Khilafat movement was launched at this event 

(D) The Quit-India Resolution was passed 

Answer: (B) 

 

Q. 47 Ozone at the higher level of the atmosphere is a product of ______ acting on oxygen molecules. 

(A) IR radiation 

(B) UV radiation 

(C) gamma rays 

(D) x-rays 

Answer: (B) 

 

Q. 48 The Battle of Chausa was fought between Humayun and ……. 

(A) Nadir Shah 

(B) Krishnadeva Raya 

(C) Hemu 

(D) Sher Shah Suri 

Answer: (D) 

 

Q. 49 Which of the following is an east flowing river? 

(A) Sabarmati 

(B) Sharavati 

(C) Mahi 

(D) Brahmani 

Answer: (D) 

 

Q. 50 Which country hosted the 13th South Asian Games? 

(A) Maldives 

(B) Bhutan 

(C) Nepal 

(D) India 

Answer: (C) 

Quantitative Aptitude 

Instructions 

For the following questions answer them individually 

Q. 51 During a medical check-up, the heights of 40 students in a class were recorded as shown in the following table.

How many students have a height of 165 cm or more? 

(A) 15 

(B) 10 

(C) 25 

(D) 16 

Answer: (A) 

Explanation: 

Number of students which have a height of 165 cm or more = total student – less than 165 = 40 – 25 = 15 

 

Q. 52 In the given figure, if ∠APO = 35, then which of the following options is correct?

(A) ∠AP(B) = 60 

(B) ∠AP(B) = 80 

(C) ∠AP(B) = 35 

(D) ∠AP(B) = 55 

Answer: (C) 

 

Q. 53 In the given figure, if AB = 10 cm, CD = 7 cm, SD = 4 cm and AS = 5 cm, then BC = ? 

(A) 9 cm 

(B) 7.5 cm 

(C) 6 cm 

(D) 8 cm 

Answer: (D) 

 

Q. 54 If a + 1/a = 5 then a3 + 1/a3 is: 

(A) 110 

B 10 

(C) 80 

(D) 140 

Answer: (A) 

 

Q. 55 The following table shows the daily earnings of 45 skilled workers. 

How many workers earn less than ₹1,100 in a day? 

(A) 39 

(B) 10 

(C) 29 

(D) 43 

Answer: (A) 

Explanation: 

Number of workers earn less than ₹1,100 in a day = 4 + 15 + 10 + 10 = 39 

 

Q. 56 Amit and Sunil together can complete a work in 9 days, Sunil and Dinesh together can complete the same work in 12 days, and Amit and Dinesh together can complete the same work in 18 days. In how many days will they complete the work if Amit, Sunil and Dinesh work together? 

(A) 12 days 

(B) 8 days 

(C) 14 days 

(D) 16 days 

Answer: (B) 

Explanation: 

Let the total work be 36 units. 

∵ ( LCM of 9, 12 and 18 is 36.) 

Efficiency of Amit and Sunil = work/time = 36/9 = 4 units/day Efficiency of Sunil and Dinesh = 36/12 = 3 units/day Efficiency of Amit and Dinesh = 36/18 = 2 units/day 

4+3+2 

Efficiency of Amit, Sunil and Dinesh = = 9/2 2 

36 

9/2 

Time taken by Amit, Sunil and Dinesh to complete the work = = 8 days 

 

Q. 57 Solve the following 

113 × 87 =? 

(A) 9831 

(B) 10026 

(C) 10169 

(D) 10000 

Answer: (A) 

Explanation: 

113 × 87 =? 

(100 + 13) × (100 − 13) =? 

(a + b)(a − b) = a2 − b2 

(100)2 − (13)2 =? 

? = 10000 – 169 = 9831  

 

Q. 58 Amit borrowed a sum of ₹25,000 on simple interest. Bhola borrowed the same amount on compound interest(interest compounded yearly). At the end of 2 years, Bhola had to pay ₹160 more interest than Amit. The rate of interest charged per annum is: 

(A) 16/25 %

(B) 8/25 %

(C) 8%

(D) 3⅛%

Answer: (C) 

 

Q. 59 The number of students enrolled in a college is shown in the following table. 

The ratio of the total number of boys to that of girls in the college is: 

(A) 13 : 42 

(B) 1:1 

(C) 23:27 

(D) 27:23 

Answer: (D) 

Explanation: 

Total number of boys = 18 + 47 + 40 + 30 = 135 

Total number of girls = 35 + 25 + 45 + 10 = 115 

The ratio of the total number of boys to that of girls in the college = 135 : 115 = 27 : 23 

 

Q. 60 A manufacturer sells cooking gas stoves to shopkeepers at 10% profit, and in turn they sell the cooking gas stoves to customer to earn 15% profit. If a customer gets a cooking gas stove for ₹7,590, then what is its manufacturing cost? 

(A) ₹ 6,500 

(B) ₹5,000 

(C) ₹5,090 

(D) ₹6,000 

Answer: (D) 

 

Q. 61 A shopkeeper marked a computer table for ₹7,200. He allows a discount of 10% on it and yet makes profit of 8%. What will be his gain percentage if he does NOT allow any discount? 

(A) 20 % 

(B) 9 % 

(C) 18 % 

(D) 2 % 

Answer: (A) 

 

Q. 62 The value of sin 30 sin 60 / cos 60cos 30− tan 45 is: 

(A)

(B)

(C) 

2− 2 2 

(D)

Answer: (B) 

 

Q. 63 Solve the following. 

4/3 ÷ ⅙ × 2 – 1 = ?

(A)

(B) -2 

(C)

(D) 15 

Answer: (D) 

 

Q. 64 In a school, 4 % of the students did not appear for the annual exams. 10% of the students who appeared for the exams could not pass the exam. Out of the remaining students, 50% got distinction marks and 432 students passed the exam but could not get distinction marks. The total number of students in the school is: 

(A) 1000 

(B) 1200 

(C) 960 

(D) 878 

Answer: (A) 

 

Q. 65 24 students collected money for donation. The average contribution was ₹50. Later on, their teacher also contributed some money. Now the average contribution is ₹56. The teacher’s contribution is: 

(A) ₹56 

(B) ₹200 

(C) ₹106 

(D) ₹194 

Answer: (B) 

 

Q. 66 In the given figure, if A(B) = 8cm, A(C) = 10cm, ∠ABD = 90and AD = 17cm , then the measure of CD is: 

(A) 8 cm 

(B) 9 cm 

(C) 11 cm 

(D) 10 cm 

Answer: (B) 

 

Q. 67 If (cos2 θ − 1)(1 + tan2 θ) + 2 tan2 θ = 1, 0< θ < 90then θ is: 

(A) 30 

(B) 90 

(C) 60 

(D) 45 

Answer: (D) 

 

Q. 68 The largest number which should replace * in the number 2365*4 to make the number divisible by 4 is: 

(A)

(B)

(C)

(D)

Answer: (D) 

Explanation: 

For the divisibility by 4, last 2 digit of the number should be divisible by 4. So, 

For the largest number, 

value of * = 8. 

 

Q. 69 In a triangle, if the measures of two sides are 5 cm and 8 cm,then the third side can be: 

(A) 3 cm 

(B) 14 cm 

(C) 2 cm 

(D) 4 cm 

Answer: (D) 

Explanation: 

For third side of triangle, 

8 – 5 < third side of triangle < 5 + 8 

= 3 < third side of triangle < 13 

Third side should be greater than 3 and less then 13 so, 

By option, correct answer is 4 cm. 

 

Q. 70 Two cyclists X and Y start at the same time from place A and go towards place B at a speed of 6 km/h and 8 km/h,respectively. Despite stopping for 15 minutes during the journey, Y reaches 10 minutes earlier than X. The distance between the places A and B is: 

(A) 6 km 

(B) 16.5 km 

(C) 8 km 

(D) 10km 

Answer: (D) 

 

Q. 71 Solve the following. 

sin 0sin 30sin 45sin 60sin 90=? 

(A)

(B)

(C) 6/8 

(D)

Answer: (A) 

 

Q. 72 The coefficient of x in (x − 3y)3 is : 

(A) 3y2 

(B) 27y2 

(C) −27y2 

(D) −3y2 

Answer: (B) 

 

Q. 73 The lengths of the diagonals of a rhombus are 16 cm and 12 cm. Its area is: 

(A) 48 cm2

(B) 28 cm2

(C) 69 cm2

(D) 96 cm2

Answer: (D) 

Explanation: 

Area of a rhombus = (1/2) × multiplication of diagonals 

= (1/2)×16 × 12 = 96 cm2 

 

Q. 74 In wallet, there are notes of the denominations ₹10 and ₹50. The total number of notes is 12. The number of ₹10 and 50 notes are in the ratio of 1 : 2. Total money in the wallet is: 

(A) ₹ 360 

(B) ₹ 280 

(C) ₹ 110 

(D) ₹ 440 

Answer: (D) 

Explanation: 

The ratio of the number of ₹10 and ₹50 notes = 1 : 2 

The total number of notes = 12 

The number of ₹10 notes = $$\frac{1}{1 + 2} \times 12 = 4 

Money from the ₹10 notes = 4×10 = Rs.40 

The number of ₹50 notes = 12 – 4 = 8 

Money from the ₹50 notes = 8×50 = Rs.400 

Total money in the wallet = 40 + 400 = Rs.440 

 

Q. 75 The following table shows the income(in ₹) for a particular month, together with their source, in respect of 5 employees (A, B, C, D and E) 

How many employees got more than a total of ₹10,000 as income other than salary? 

(A)

(B)

(C)

(D)

Answer: (D) 

Explanation: 

Salary of A = 52000 

Other income = total – salary 

Other income of A = 66000 – 52000 = 14000 

Salary of B = 48500 

Other income of B = 59000 – 48500 = 10500 

Salary of C = 42000 

Other income of C = 52000 – 42000 = 10000 

Salary of D = 31000 

Other income of D = 40000 – 31000 = 9000 

Salary of E = 25000 

Other income of E = 31700 – 25000 = 6700 

∴ 2 employees(A and B) got more than a total of ₹10,000 as income other than salary. 

English Comprehension 

Instructions 

For the following questions answer them individually 

Q. 76 Select the correct passive form of the given sentence. 

(A) A child could understand his theory. 

(B) His theory is being understood by a child. 

(C) His theory could be understood by a child. 

(D) His theory was understood by a child. 

Answer: (C) 

 

Q. 77 Select the most appropriate meaning of the given idiom. 

Pull a fast one 

(A) Take quick action 

(B) Believe someone easily 

(C) Progress fast 

(D) Trick someone 

Answer: (D) 

 

Q. 78 Select the most appropriate option to fill in the blank. 

The poor man was ______ entry to the restaurant. 

(A) derived 

(B) denied 

(C) devoid 

(D) deluded 

Answer: (B) 

Explanation: 

The word that fits the best is ‘denied’. 

 

Q. 79 Select the most appropriate antonym of the given word. 

FICKLE 

(A) Flighty 

(B) Subtle 

(C) Stable 

(D) Crafty 

Answer: (C) 

 

Q. 80 In the sentence identify the segment which contains the grammatical error. 

We had a long discussion at a cup of coffee. 

(A) We had 

(B) a long discussion 

(C) at a cup 

(D) of coffee 

Answer: (C) 

 

Q. 81 Given below are four jumbled sentences. Out of the given options pick the one that gives their correct order. 

(A) These lotteries organised by State Governments sell dreams. 

(B) Lured by this, millions of poor people waste their hard-earned money. 

(C) Crores of rupees and gold are offered as prizes. 

(D) People buy lotteries hoping to become rich overnight. 

(A) CABD 

(B) CBAD 

(C) DACB 

(D) DBCA 

Answer: (C) 

Instructions 

In the following passage some words have been deleted. Fill in the blanks with the help of the alternatives given. Select the most appropriate option for each blank. 

Comprehension: 

Agra’s marble-marvel has such an overwhelming presence (1)______ other monuments in the town have been relegated (2) ______ oblivion. Amongst such 60 odd monuments in (3) ______ town, the Red Taj is the most (4) ______ and relatively well-preserved. The tomb in red sandstone was (5) ______ by Ann Hessing, widow of a Dutch officer, after his death in 1803. 

 

Q. 82 Select the correct option to fill in blank 1. 

(A) but 

(B) that 

(C) if 

(D) than 

Answer: (B) 

Explanation: 

The word that fits the blank appropriately is “that”. 

 

Q. 83 Select the correct option to fill in blank 2. 

(A) with 

(B) to 

(C) on 

(D) by 

Answer: (B) 

 

Q. 84 Select the correct option to fill in blank 3. 

(A) their 

(B)

(C) the 

(D) an 

Answer: (C) 

 

Q. 85 Select the correct option to fill in blank 4. 

(A) active 

(B) elegant 

(C) efficient 

(D) adequate 

Answer: (B) 

 

Q. 86 Select the correct option to fill in blank 5. 

(A) build 

(B) building 

(C) to build 

(D) bulit 

Answer: (D) 

Instructions 

For the following questions answer them individually 

 

Q. 87 Select the INCORRECTLY spelt word 

(A) Delegait 

(B) Rebel 

(C) Edible 

(D) Monarch 

Answer: (A) 

 

Q. 88 Given below are four jumbled sentences. Out of the given options pick the one that gives their correct order. 

(A) These are generated by a violent undersea disturbance or ocean activity. 

(B) Once generated, they travel outward on the ocean surface in all directions. 

(C) A tsunami is made up of a series of very long waves. 

(D) Spreading thus, they look like ripples caused by throwing a rock in a pond. 

(A) BADC 

(B) BDCA 

(C) CBDA 

(D) CABD 

Answer: (D) 

 

Q. 89 Select the correct indirect form of the given sentence. 

Mahesh said to Rita, “Don’t play in the sun.” 

(A) Mahesh advised Rita not to play in the sun 

(B) Mahesh told to Rita to not play in the sun. 

(C) Mahesh ordered Rita that not play in the sun 

(D) Mahesh requested Rita that don’t play in the sun 

Answer: (A) 

 

Q. 90 Select the most appropriate synonym of the given word. 

DISHEARTENE(D) 

(A) depressed 

(B) disgraced 

(C) disgusted 

(D) disliked 

Answer: (A) 

 

Q. 91 Select the most appropriate option to substitute the underlined segment in the given sentence. If there is no need to substitute it, select No improvement. 

They stopped to laugh as soon as the teacher entered the classroom. 

(A) No improvement 

(B) stop laughing 

(C) stopped laughing 

(D) stopped laugh 

Answer: (C) 

 

Q. 92 Select the most appropriate one word substitution for the given group of words. 

Pertaining to an individual from birth 

(A) Congenital 

(B) Anomaly 

(C) Chronic 

(D) Habitual 

Answer: (A) 

 

Q. 93 Select the most appropriate option to fill in the blank. 

Have you ______ for a job in this company? 

(A) appointed 

(B) supplied 

(C) approved 

(D) applied 

Answer: (D) 

 

Q. 94 In the sentence identify the segment which contains the grammatical error. 

The both children go to the same school. 

(A) to the 

(B) The both 

(C) children go 

(D) same school 

Answer: (B) 

 

Q. 95 Select the most appropriate meaning of the given idiom. 

See eye to eye 

(A) Stare at someone 

(B) See clearly 

(C) Be suspicious 

(D) Agree with someone 

Answer: (D) 

 

Q. 96 Select the most appropriate synonym of the given word. 

CORDIAL 

(A) cold 

(B) warm 

(C) hard 

(D) rude 

Answer: (B) 

 

Q. 97 Select the most appropriate option to substitute the underlined segment in the given sentence. If there is no need to substitute it, select No improvement. 

If they will get married, they will probably settle in Mumbai. 

(A) If they get married 

(B) No improvement 

(C) If they had got married 

(D) If they will be getting married 

Answer: (A) 

 

Q. 98 Select the most appropriate one word substitution for the given group of words. 

To increase the speed 

(A) Assimilate 

(B) Exhilarate 

(C) Activate 

(D) Accelerate 

Answer: (D) 

 

Q. 99 Select the INCORRECTLY spelt word. 

(A) Obscure 

(B) Omision 

(C) Opportunity 

(D) Obsolete 

Answer: (B) 

 

Q. 100 Select the most appropriate antonym of the given word. 

HASTE 

(A) Rush 

(B) Lose 

(C) Dash 

(D) Delay 

Answer:

SSC CGL Tier-I 06 March 2020 Shift-I Previous Year Paper

SSC CGL 6th March 2020 Shift-1 

Q. 1 Four number-pairs have been given, out of which three are alike in some manner and one is different. Select the number-pair that is different from the rest. 

(A) 11:121 

(B) 13: 169 

(C) 17:289 

(D) 15:250 

Answer: (D) 

Explanation: 

(11)2 = 121 

(13)2 = 169 

(17)2 = 289 

(15)2 = 225 

∴ The correct answer is option (D) 

 

Q. 2 Study the given pattern carefully and select the number that can replace the question mark (?) in it 

(A) 92 

(B) 98 

(C) 88 

(D) 82 

Answer: (C) 

 

Q. 3 Read the given statements and conclusions carefully. Assuming that the information given in the statements is true, even if it appears to be at variance with commonly known facts, decide which of the given conclusions logically follow(s) from the statements. Statements: 

1. Some machines are kites. 

2. No machine is a pigeon. 

Conclusions: 

I. Some machines are pigeons. 

IL. All kites are pigeons. 

Ill. Some kites are not pigeons. 

(A) Only conclusions I and III follow. 

(B) Only conclusion II follows. 

(C) Only conclusions I and II follow. 

(D) Only conclusion ITI follows. 

Answer: (D) 

Explanation: 

Only conclusion III follows. 

∴ The correct answer is option (D) 

 

Q. 4 Select the number that can replace the question mark (?) in the followingseries. 

5, 10, 26, ?, 122, 170 

(A) 82 

(B) 50 

(C) 65 

(D) 77 

Answer: (B) 

Explanation: 

(2)2 + 1 

= 4 + 1 = 5 

(3)2 + 1 

= 9 + 1 = 10 

(5)2 + 1 

= 25 + 1 = 26 

(7)2 + 1 

= 49 + 1 = 50 

(11)2 + 1 

= 121 + 1 = 122 

(13)2 + 1 

= 169 + 1 = 170 

∴ The correct answer is option (B) 

 

Q. 5 Select the letter that can replace the question mark (?) in the following series. 

A, E, I, ?, Q, U 

(A)

(B)

(C)

(D)

Answer: (A) 

Explanation: 

The series follows pattern as, 

(A) + 4 = E 

E + 4 = I 

I + 4 = M 

M + 4 = Q 

Q + 4 = U 

∴ The correct answer is option (A) 

 

Q. 6 Select the letter-cluster that can replace the question mark (?) in the following series. ADG, CGK, EJO, ? , IPW 

(A) GMZ 

(B) GNS 

(C) GNZ 

(D) GMS 

Answer: (D) 

Explanation: 

((A) + 2)((D) + 3)(G + 4) = CGK 

((C) + 2)(G + 3)(K + 4) = EJO 

(E + 2)(J + 3)(0 + 4) = GMS 

(G + 2)(M + 3)(S + 4) = IPW 

∴ The correct answer is option (D) 

 

Q. 7 Select the correct mirror image of the given figure when a mirror is placed on the right of the figure. 

Answer: (A) 

 

Q. 8 Select the option in which the given figure is embedded (rotation is not allowed) 

Answer: (A) 

 

Q. 9 How many triangles are there in the given figure? 

(A) 25 

(B) 22 

(C) 16 

(D) 28 

Answer: (A) 

 

Q. 10 Select the option in which the words share the same relationship as that shared by the given pair of words.

North : South 

(A) Frown : Smile 

(B) Whole : Total 

(C) Ductile : Bendable 

(D) Integrity : Honesty 

Answer: (A) 

Explanation: 

Opposite of North is South. 

Similarly, 

Opposite of the frown is smile. 

∴ The correct answer is option (A) 

 

Q. 11 In a certain a group of men and horses, the total number of legs is 14 more than twice the number of heads. How many horses are there in the group? 

(A) 11 

(B)

(C)

(D)

Answer: (B) 

Explanation: 

The total number of legs is 14 more than twice the number of heads. 

A horse have 2 more legs than man so, 

Number of horse = 14/2 = 7 

∴ The correct answer is option (B) 

 

Q. 12 Select the option that depicts how the given transparent sheet of paper  would appear if it is folded at the dotted line. 

Answer: (B) 

 

Q. 13 Three different positions of the same dice are shown. Select the symbol that will be on the face opposite to the one showing ‘ +’. 

(A)

(B)

(C)

(D)

Answer: (C) 

 

Q. 14 In a certain code language, ‘PAGE’ is coded as ‘161495’. How will ‘PART’ be coded as in that language? 

(A) 16125020 

(B) 16132420 

(C) 16128620 

(D) 16136220 

Answer: (B) 

Explanation: 

P = 16 

(A) = 1 

 G =(7)2 = 49 

E = 5 

Similarly, 

P = 16 

(A) = 1 

R =(18)2 = 324 

T = 20 

So, ‘PART’ is coded as ‘16132420’. 

∴ The correct answer is option (B) 

 

Q. 15 ‘Astronomy’ is related to ‘Stars’ in the same way as ‘Agronomy’ is related to ‘…….’ 

(A) Mines 

(B) Crops 

(C) Planets 

(D) Emotions 

Answer: (B) 

Explanation: 

‘Astronomy’ is related to ‘Stars’ in the same way as ‘Agronomy’ is related to ‘crops’ 

∴ The correct answer is option (B) 

 

Q. 16 Four letter-clusters have been given, out of which three are alike in some manner and one is different. Select the odd letter-cluster. 

(A) BJT 

(B) GCV 

(C) DDP 

(D) FBL 

Answer: (B) 

Explanation: 

 B = 2, J = 10, 2× 10 = 20(T) 

G = 7, C = 3, 7×  3 = 21(U) 

D = 4, D= 4, 4 ×4 = 16(P) 

 F = 6, (B) = 2, 6 ×2 = 12(L) 

∴ The correct answer is option (B) 

 

Q. 17 Four words have been given, out of which three are alike in some manner and one is different. Select the odd word. 

(A) Clarinet 

(B) Trumpet 

(C) Xylophone 

(D) Saxophone 

Answer: (C) 

Explanation: 

Except option C, remaining all are play by the mouth. 

∴ The correct answer is option (C) 

 

Q. 18 A+B means ‘A is the husband of B’; 

A-B means ‘B is the sister of A’; 

(A)× (B) means ‘(A) is the mother of B’; 

(A) ÷ (B) means ‘(B) is the son of A’. 

P + R × T − Q ÷ S + U, 

If, then how is P related to S? 

(A) Maternal grandfather 

(B) Father-in-law 

(C) Paternal grandfather 

(D) Uncle 

Answer: (A) 

Explanation: 

In the diagram, square shows the males, circle shows the females, vertical lines shows the generation, single horizontal lines shows the brother/sister and double horizontal lines shows the couples. 

From the diagram, 

P is the maternal grandfather of S. 

 

Q. 19 In the given Venn diagram, the ‘rectangle’ represents ‘travellers who like to travel by plane’, the ‘circle’ represents ‘travellers who like to travel by bus’, and the ‘triangle’ ‘represents travellers who like to travel by train’. The numbers given in the diagram represent the number of travellers in that particular category. 

How many travellers like to travel either by train or plane but NOT by bus? 

(A) 36 

(B) 27 

(C) 30 

(D) 29 

Answer: (C) 

 

Q. 20 Select the figure that can replace the question mark (?) in the following series. 

Answer: (B) 

 

Q. 21 Which two signs should be interchanged to make the given equation correct? 

12 + 81 − 27 × 9 ÷ 3 =36

(A) −and÷

(B) ÷and×

(C) ×and−

(D) +and

Answer: (A) 

Explanation: 

12 + 81 − 27 × 9 ÷ 3 = 36 

From the option A, 

On changing the sign, 

12 + 81 ÷ 27 × 9 − 3 = 36 

12 + 27 – 3 = 36 

36 =36 

∴ The correct answer is option (A) 

 

Q. 22 In a certain code language, ‘DKQX’ is written as ‘WPJC’. How will ‘AFKP’ be written as in that language? 

(A) ZTOK 

(B) ZUQI 

(C) ZUPK 

(D) ZVQL 

Answer: (C) 

Explanation: 

D = (27 – 4 = 23) = W 

K = (27 – 11 = 16) = P 

Q = (27 – 17 = 10) = J 

X = (27 – 24 = 3) = C)

Similarly, 

A = (27 – 1 = 26) = Z 

F = (27 – 6 = 21) = U 

K = (27 – 11 = 16) = P 

P = (27 – 16 = 11) = K 

‘AFKP’ is written as ‘ZUPK’. 

∴ The correct answer is option (C) 

 

Q. 23 Study the given pattern carefully and select the number that can replace the question mark (?) in it. 

(A) 111 

(B) 113 

(C) 95 

(D) 77 

Answer: (D) 

 

Q. 24 Select the number that can replace the question mark (?) in the following series. 62, 66, 74, 90, ? 

(A) 106 

(B) 120 

(C) 116 

(D) 122 

Answer: (D) 

 

Q. 25 Arrange the following words in a logical and meaningful order. 

1. Quadrillion 

2. Quintillion 

3. Million 

4. Octillion 

5. Trillion 

(A) 3-5-1-4-2 

(B) 4-3-5-1-2 

(C) 2-3-5-1-4 

(D) 3-5-1-2-4 

Answer: (D) 

Explanation: 

Arrangement of the words, 

Million, Trillion, Quadrillion, Quintillion, Octillion 

∴ The correct answer is option (D) 

General Awareness 

Instructions 

For the following questions answer them individually 

Q. 26 Name the Indian wrestler who has been named the Junior Freestyle Wrestler of the year (2019) by United World Wrestling (UWW) 

(A) Kamareddy 

(B) Deepak Punia 

(C) B.P. Raju 

(D) Amita Bagchi 

Answer: (B) 

 

Q. 27 Which International organisation released the ‘World Energy Outlook 2019’ Report? 

(A) UNESCO 

(B) United Nations 

(C) World Trade Organisation 

(D) International Energy Agency 

Answer: (D) 

 

Q. 28 The Supreme Court has declared access to the internet a fundamental right under Article ______ of the Indian Constitution 

(A) 21 

(B) 14 

(C) 17 

(D) 19 

Answer: (D) 

 

Q. 29 Mehrunnisa who was known as Nur Jahan was the wife of ……. 

(A) Aurangzeb 

(B) Shah Jahan 

(C) Jahangir 

(D) Akbar 

Answer: (C) 

Explanation: 

Nur Jahan was the twentieth (and last) wife of the Mughal emperor Jahangir. 

 

Q. 30 Alyssa Healy who created a world record by becoming the highest scorer in women’s T20I, belongs to ______. 

(A) India 

(B) Canada 

(C) Japan 

(D) Australia 

Answer: (D) 

 

Q. 31 If the inflation in an economy is rising steadily, the Central Bank might ______. 

(A) decrease the repo rate 

(B) decrease the reverse repo rate 

(C) increase the repo rate 

(D) keep the repo rate unchanged 

Answer: (C) 

 

Q. 32 Merchant discount rate refers to ______ 

(A) the total discount a bank offers to the merchant for promoting online transactions 

(B) the total discount a merchant offers on online transactions 

(C) the rate charged to a merchant for payment processing services on debit and credit card transactions 

(D) only taxes that a digital payment entails 

Answer: (C) 

 

Q. 33 Which Indian journalist was honoured with India’s Most Powerful Woman in Media Award during the prestigious Confluence Excellence Award ceremony held in the British parliament on 27 September 2019? 

(A) Sunetra Chaudhary 

(B) Anubha Bhonsle 

(C) Kalli Puri 

(D) Sona Choudhary 

Answer: (C) 

 

Q. 34 In which year did D.C Sircar publish Indian Epigraphy and Indian Epigraphical Glossary? 

(A) 1965-66 

(B) 1967-68 

(C) 1966-67 

(D) 1964-65 

Answer: (A) 

 

Q. 35 To which of the following Indian states does Mardani Khel, one of the famous martial art forms in India belong? 

(A) Uttar Pradesh 

(B) Madhya Pradesh 

(C) Maharashtra 

(D) Rajasthan 

Answer: (C) 

 

Q. 36 Which of the following diseases is NOT transmitted from one person to another? 

(A) Syphilis 

(B) Hepatitis B 

(C) AIDS 

(D) Cirrhosis 

Answer: (D) 

 

Q. 37 When did the Vikrama Era begin? 

(A) 47 BC 

(B) 55 BC

(C) 57 BC 

(D) 50 BC 

Answer: (C) 

 

Q. 38 Who is the first General of the Indian Army, whose retirement day is celebrated as the ‘Armed Forces Veterans Day’ every year? 

(A) General K.S. Thimayya 

(B) General S.M. Srinagesh 

(C) General K. M. Cariappa 

(D) General Maharaj Rajendra Sinhji 

Answer: (C) 

 

Q. 39 The SDG India index is developed by …… 

(A) Ministry of Environmental, Forests and Climate Change 

(B) Ministry of Home Affairs 

(C) NITI Aayog 

(D) World Bank 

Answer: (C) 

 

Q. 40 Which of the following is the first South Asian country to call match-fixing a crime? 

(A) India 

(B) Sri Lanka 

(C) Nepal 

(D) Pakistan 

Answer: (B) 

 

Q. 41 Who performs the task of capturing oxygen in the blood? 

(A) White blood cells 

(B) Red blood cells 

(C) Chlorophyll 

(D) Haemoglobin 

Answer: (D) 

 

Q. 42 On which river and in which state is the Jayakwadi dam situated? 

(A) Godavari river – Maharashtra 

(B) Banas river – Rajasthan 

(C) Periyar river – Kerala 

(D) Ravi river – Punjab 

Answer: (A) 

 

Q. 43 Which leader gave us the C R Formula? 

(A) C R. Das 

(B) C Rajagopalachari 

(C) Tilak 

(D) Mahatma Gandhi 

Answer: (B) 

 

Q. 44 Name the only metal that is antibacterial. 

(A) Iron 

(B) Sodium 

(C) Aluminium 

(D) Copper 

Answer: (D) 

 

Q. 45 Madhavpur Mela takes place in ……. 

(A) Maharashtra 

(B) Madhya Pradesh 

(C) Uttar Pradesh 

(D) Gujarat 

Answer: (D) 

 

Q. 46 Which of the following is a natural flame retardant? 

(A) Flowers 

(B) Skin 

(C) DNA 

(D) RNA

Answer: (C) 

 

Q. 47 In the sequence of planets in the Solar system, which planet comes in between Mercury and Earth? 

(A) Mars 

(B) Venus 

(C) Jupiter 

(D) Uranus 

Answer: (B) 

 

Q. 48 Which of the following gases is consistently seen to be most abundant in a volcanic eruption? 

(A) Sulphur Dioxide 

(B) Carbon Dioxide 

(C) Water vapour 

(D) Hydrogen Sulphide 

Answer: (C) 

 

Q. 49 Which of the following is the primary sex organ in females? 

(A) Uterus 

(B) Fallopian tube 

(C) Ovary 

(D) Vagina 

Answer: (C) 

 

Q. 50 UN Women propose to partner with ______ gender park with an aim to secure gender parity in South Asia 

(A) Punjab’s 

(B) Haryana’s 

(C) Rajasthan’s 

(D) Kerala’s 

Answer: (D) 

Quantitative Aptitude 

Instructions 

For the following questions answer them individually 

Q. 51 Two tangents PA and PBare drawn to a circle with centre O from an external point P. If ∠OAB = 30, then ∠APB is: 

(A) 120 

(B) 30 

(C) 15 

(D) 60 

Answer: (D) 

 

Q. 52 ABC is an equilateral triangle. P,Q and R are the midpoints of sides AB,BC and CA, respectively. If the length of the side of the triangle △ PQR 

ABC is 8 cm, then the area of is: 

(A) 3/3 cm2 

(B) √3/4 cm2 

(C) 4√3 cm2 

(D) 8√3 cm2 

Answer: (C) 

Q. 53 Solve the following. 

(A)

(B) -2 

(C) -1 

(D)

Answer: (B) 

 

Q. 54 The average of the marks of 30 boys is 88, and when the top two scores were excluded, the average marks reduced to 87.5. If the top two scores differ by 2, then the highest mark is: 

(A) 92 

(B) 96 

(C) 94 

(D) 90 

Answer: (B) 

Explanation: 

Total marks of 30 boys = 88 $$\times 30 = 2640 

( ∵ Average = sum of all term/number of terms) 

when the top two scores were excluded, the average marks reduced to 87.5 

Total marks of 28 boys = 87.5 $$\times 28 = 2450 

Sum of the top two score = 2640 – 2450 = 190 —(1) 

difference of two score = 2 

Let the top score be x so 2nd top score = x – 2 

From eq(1), 

x + x – 2 = 190 

x = 192/2 = 96 

 

Q. 55 If x cos A − y sin A = 1 and x sin A + y cos A = 4, then the value of 17x2 + 17y2 is: 

(A)

(B)

(C) 49 

(D) 289 

Answer: (D) 

 

Q. 56 A circular disc of area 0.64πm2 rolls down a length of 1.408 km. The number of revolutions it makes is: (Taken π = 22/7). 

(A) 280 

(B) 360 

(C) 140 

(D) 180 

Answer: (A) 

 

Q. 57 An article was sold at a gain of 18%. If it had been sold for ₹ 49 more, then the gain would have been 25%. The cost price of the article is: 

(A) ₹ 700 

(B) ₹ 570 

(C) ₹ 650 

(D) ₹ 890 

Answer: (A) 

 

Q. 58 If 3a = 27b = 81c and abc = 144, then the value of 12(1/a + 1/2b + 1/5c) is: 

(A) 18/120

(B) 18/10

(C) 33/10

(D) 17/120

Answer: (C) 

 

Q. 59 If a + b + c = 9 and ab + bc + ca = −22, then the value of a3 + b3 + c3 − 3abc is: 

(A) 783 

(B) 1323 

(C) 1571 

(D) 487 

Answer: (B) 

Explanation: 

a + b + c = 9 

ab + bc + ca = -22 

We need to find the value of a³ + b³ + c³- 3abc 

We know, 

a³ + b³ + c³- 3abc = ( a + b + c ) ( a² + b² + c² – ab – bc – ca ) 

Now we need to find the value of a² + b² + c² 

We also know, 

( a + b + c )² = a² + b² + c² + 2 ( ab + bc + ca ) 

Putting the value of a + b + c = 9 and ab + bc + ca = -22 

( 9 )² = a² + b² + c² + 2 ( -22 ) 

81 = a² + b² + c² -44 

a² + b² + c² = 81 + 44 

a² + b² + c² = 125 

Now, 

Substituting value in the formula for a³ + b³ + c³- 3abc 

a³ + b³ + c³- 3abc = ( a + b + c ) ( a² + b² + c² – ab – bc – ca ) 

a³ + b³ + c³- 3abc = ( a + b + c ) ( a² + b² + c² – ( ab + bc + ca ) ) 

a³ + b³ + c³- 3abc = ( 9 ) ( 125 – ( =22) ) 

a³ + b³ + c³- 3abc = ( 9 ) ( 147 ) 

a³ + b³ + c³- 3abc = 1323 

 

Q. 60 A shopkeeper marks the price of an article in such a way that after allowing a discount of 22%, he gets a gain of 11%.If the marked price is Rs.888, then the cost price of the article is: 

(A) ₹ 624 

(B) ₹ 782 

(C) ₹ 550 

(D) ₹ 895 

Answer: (A) 

 

Q. 61 In the following table, the production of various crops (in tonnes) is given from 2015 to 2019. Study the table and answer the questions that follow. 

The percentage growth of maize in the year 2019 over the year 2015 is: 

(correct to two decimal places) 

(A) 65.12% 

(B) 71.43% 

(C) 77.77% 

(D) 60.28% 

Answer: (B) 

Explanation: 

Production of maize in the year 2019 = 840 

Production of maize in the year 2015 = 490 

The percentage growth of maize in the year 2019 over the year 2015 = $$\frac{840 – 490}{490} \times 100 = \frac{350}{490} \times 100 = 71.43% 

 

Q. 62 If the area of an equilateral triangle is 36√3cm2 , then the perimeter of the triangle is: 

(A) 36√3 cm 

(B) 36 cm 

(C) 18√3 cm 

(D) 12 cm 

Answer: (B) 

 

Q. 63 If the value of(a + b − 2)2 + (b + c − 5)2 + (c + a − 5)2 = 0 , then the value of (b + c)a + (c + a)b − 1 is: 

(A)

(B)

(C)

(D)

Answer: (C) 

 

Q. 64 If (2 sin (A) + cosec A) = 22 0< A < 90, then the value of 2(sin4 A + cos4 A)  is: 

(A)

(B)

(C)

(D)

Answer: (B) 

 

Q. 65 In the following table, the production of various crops (in tonnes) is given from 2015 to 2019. Study the table and answer the questions that follow. 

The average production of wheat(in tonnes) in the period given in the table is: 

(A) 3600 

(B) 3482 

(C) 3300 

(D) 4218 

Answer: (A) 

Explanation: 

Total production of wheat(in tonnes) = 2500 + 4218 + 3482 + 4500 + 3300 = 18000 

Average = sum of all terms/number of terms 

Average production of wheat(in tonnes) = 18000/5 = 3600 

 

Q. 66 A, B and C can individually complete a task in 24 days, 20 days and 18 days, respectively. B and C start the task, and they work for 6 days and leave. The number of days required by A alone to finish the remaining task, is: 

(A) 15 ⅔ days 

(B) 12 ½ days 

(C) 8 ⅘ days 

(D) days 

Answer: (C) 

 

Q. 67 What will be the difference in compound interest on a sum of ₹ 7,800 at 8% for 1 year, when the interest is paid yearly and half yearly? 

(A) ₹ 29.18 

(B) ₹ 12.48 

(C) ₹ 24.72 

(D) ₹ 19.46 

Answer: (B) 

 

Q. 68 In the following table, the production of various crops(in tonnes) is given from 2015 to 2019. Study the table and answer the questions that follow. 

The difference (in tonnes) between the average production of barley and average production of rice is: 

(A) 231 

(B) 780 

(C) 549 

(D) 471 

Answer: (A) 

Explanation: 

Total production of barley = 975 + 825 + 700 + 625 + 775 = 3900 

Average = sum of the terms/total number of terms 

Average production of barley = 3900/5 = 780 

Total production of rice = 415 + 520 + 585 + 625 + 600 = 2745 

Average production of rice = 2745/5 = 549 

The difference (in tonnes) between the average production of barley and average production of rice = 780 – 549 = 231 

 

Q. 69 If ‘+’ means ‘-’,‘-’ means ‘+’, ‘x’ means ‘ ’ and ‘ ’ means ‘x’, then the value of 305+8465231842is: 

(A) -2 

(B)

(C) -1 

(D)

Answer: (A) 

 

Q. 70  A student takes 1.25 hours to travel from home to school at a speed of 4 km/h. By what percentage should he increase his speed to reduce the time by 25% to cover the same distance from school to home? 

(A) 33 ⅓ % 

(B) 50 % 

(C) 45 % 

(D) 25 % 

Answer: (A) 

 

Q. 71 If a sum of ₹1,180 is to be divided among A, B and C,such that 2 times A’s share, 5 times B’s share and 7 times C’s share, are equal, then A’s share is: 

(A) ₹ 500 

(B) ₹ 750 

(C) ₹ 650 

(D) ₹ 700 

Answer: (D) 

 

Q. 72 If the number 687×29 is divisible by 9, then the value of 2x is: 

(A)

(B)

(C)

(D)

Answer: (A) 

 

Q. 73 The population of a city increased by 30%in the first year and decreased by 15% in the next year. If the present population is 11.050 then the population 2 years ago was: 

(A) 10,000 

(B) 99,500 

(C) 99,000 

(D) 10,050 

Answer: (A) 

 

Q. 74 In the following table, the production of various crops(in tonnes) is given from 2015 to 2019. Study the table and answer the questions that follow. 

The highest growth of sugarcane production over its previous year is recorded in the year: 

(A) 2018 

(B) 2016 

(C) 2019 

(D) 2017 

Answer: (D) 

Explanation: 

Growth in 2016 = 1120 – 1000 = 120 

Growth in 2017 = 1250 – 1120 = 130 

Growth in 2018 = 1325 – 1250 = 75 

Growth in 2019 = 1450 – 1325 = 125 

The highest growth of sugarcane production over its previous year is recorded in the year 2017. 

 

Q. 75 A metallic sphere of diameter 40 cm is melted into smaller spheres of radius 0.5 cm. How many such small balls can be made? 

(A) 32,000 

(B) 3,200 

(C) 64,000 

(D) 6,400 

Answer: (C) 

English Comprehension 

Instructions 

For the following questions answer them individually 

 

Q. 76 Select the most appropriate ANTONYM of the given word. 

Persist 

(A) Insist 

(B) Cease 

(C) Remain 

(D) Continue 

Answer: (B) 

 

Q. 77 Select the correct indirect form of the given sentence. 

“What a rare flower!” she said. 

(A) She exclaimed what a rare flower it is. 

(B) She asked if it was a rare flower 

(C) She told that what a rare flower it was. 

(D) She exclaimed that it was a very rare flower. 

Answer: (D) 

 

Q. 78 Select the option that can be used as a one-word substitute for the given group of words/phrase. Having two opposing feelings at the same time 

(A) Equivalent 

(B) Coinciding 

(C) Ambivalent 

(D) Contemporary 

Answer: (C) 

 

Q. 79 Select the correct passive form of the given sentence. 

The farmer is ploughing the fields 

(A) The fields were ploughed by the farmer. 

(B) The fields are ploughed by the farmer. 

(C) The fields have been ploughed by the farmer 

(D) The fields are being ploughed by the farmer 

Answer: (D) 

 

Q. 80 Select the most appropriate option to substitute the underlined segment in the given sentence. If there is no need to substitute it, select No improvement. 

We came back because we had ran out of money. 

(A) we have ran 

(B) we had run 

(C) No improvement 

(D) we run 

Answer: (B) 

 

Q. 81 Identify the segment in the sentence which contains the grammatical error. If there is no error, select ‘No error’ 

Those who are late will not be allowed inside the classroom. 

(A) they will not be allowed 

(B) No error 

(C) Those who are late 

(D) inside the classroom 

Answer: (A) 

Instructions 

In the following passage some words have been deleted. Fill in the blanks with the help of the alternatives given. Select the most appropriate option for each blank. 

Atlas was made to hold the sky up on his shoulders as a punishment. Once, Hercules (1)______ to hold it for Atlas while the latter went looking (2)______ golden apples. When Atlas returned, he saw Hercules (3)______ hard to hold the sky. The weight of the sky (4)______ Hercules sweat and groan. Now Atlas did not want to (5)______ a job as arduous as that! 

 

Q. 82 Select the most appropriate option to fill in blank 1. 

(A) offered 

(B) ordered 

(C) afforded 

(D) extended 

Answer: (A) 

 

Q. 83 Select the most appropriate option to fill in blank 2 

(A) at 

(B) for 

(C) on 

(D) after 

Answer: (B) 

 

Q. 84 Select the most appropriate option to fill in blank 3 

(A) tackling 

(B) coping 

(C) struggling 

(D) seeking 

Answer: (C) 

 

Q. 85 Select the most appropriate option to fill in blank 4 

(A) were making 

(B) was making 

(C) has made 

(D) make 

Answer: (B) 

 

Q. 86 Select the most appropriate option to fill in blank 5. 

(A) take out 

(B) take away 

(C) take in 

(D) take back 

Answer: (D) 

Instructions 

For the following questions answer them individually 

Q. 87 Select the most appropriate ANTONYM of the given word. 

Glee 

(A) Joy 

(B) Bliss 

(C) Woe 

(D) Fun 

Answer: (C) 

 

Q. 88 Identify the segment in the sentence which contains the grammatical error. If there is no error, select ‘No error’ Everyone except she have travelled by air 

(A) travelled by air 

(B) No error 

(C) Everyone except 

(D) she have 

Answer: (D) 

 

Q. 89 Select the most appropriate option to fill in the blank. 

The roof of the old building ______ during the storm. 

(A) destroyed 

(B) scratched 

(C) demolished 

(D) collapsed 

Answer: (D) 

 

Q. 90 Select the most appropriate meaning of the given idiom. 

Chicken-hearted 

(A) Generous 

(B) Cowardly 

(C) Miserly 

(D) Selfish 

Answer: (B) 

 

Q. 91 Select the most appropriate synonym of the given word. 

Preferred 

(A) Adored 

(B) Feared 

(C) Compared 

(D) Favoured 

Answer: (D) 

 

Q. 92 Select the most appropriate synonym of the given word. 

Ovation 

(A) Creation 

(B) Preparation 

(C) Applause 

(D) Censure 

Answer: (C) 

 

Q. 93 Select the INCORRECTLY spelt word. 

(A) Alliance 

(B) Acquisition 

(C) Acquaintence 

(D) Abandon 

Answer: (C) 

 

Q. 94 Given below are four jumbled sentences. Out of the given options pick the one that gives their correct order. 

(A) Much of this war had taken place along the Western Front. 

(B) Both sides had dug in deep and each lost many men over little ground. 

(C) This front was a line of trenches across which the two sides faced each other. 

(D) Andre Maginot had fought a war with the French against the Germans. 

(A) DACB 

(B) DBCA 

(C) ACDB 

(D) BCDA 

Answer: (A) 

 

Q. 95 Select the most appropriate option to fill in the blank. 

She ______ how the magician had performed the rope trick 

(A) bewildered 

(B) wondered 

(C) suspected 

(D) puzzled 

Answer: (B) 

 

Q. 96 Select the wrongly spelt word. 

(A) Fierce 

(B) Sieze 

(C) Grief 

(D) Piece 

Answer: (B) 

 

Q. 97 Select the most appropriate meaning of the given idiom. 

By and by 

(A) Gradually 

(B) By any means 

(C) Suddenly 

(D) On the whole 

Answer: (A) 

 

Q. 98 Select the option that can be used as a one-word substitute for the given group of words/phrase 

a group of ships 

(A) squad 

(B) colony 

(C) swarm 

(D) fleet 

Answer: (D) 

 

Q. 99 Select the most appropriate option to substitute the underlined segment in the given sentence. If there is no need to substitute it, select No improvement. 

When it started raining, I ran to the nearer house for shelter 

(A) the near house 

(B) a near house 

(C) the nearest house 

(D) No improvement 

Answer: (C) 

 

Q. 100 Given below are four jumbled sentences. Out of the given options pick the one that gives their correct order. 

(A) The mountain range was covered with snow. 

(B) Napoleon had to cross the Alps in winter. 

(C) Napoleon replied that the word ‘impossible’ did not exist for him. 

(D) So, someone told Napoleon that the task was impossible. 

(A) BADC 

(B) DBCA 

(C) BDCA 

(D) ADBC 

Answer: A

SSC CGL Tier-I 05 March 2020 Shift-III Previous Year Paper

SSC CGL 5th March 2020 Shift-3

 

Q. 1 A + B means ‘A. is the mother of B’; 

A B means ‘A is the brother of B’; 

A × B means ‘A is the father of B’; 

A ÷ B means ‘A is the daughter of B’; 

If P K × Y J ÷ S + R , then which of the following statements is NOT correct? 

A. J is daughter of P. 

B. P is paternal uncle of R. 

C. K is husband of S. 

D. Y is son of S. 

Answer: A. 

Explanation: 

In the diagram, Circle shows the female, square shows the male, vertical lines show the generation, single horizontal line shows the brother/sister and  double horizontal lines show the couple. 

From the diagram, 

Option A is correct. 

∴ The correct answer is option A. 

 

Q. 2 Select the letter-cluster that can replace the Question mark (?) in the following series. 

TULG, WRPC, ZOTY, CLXU,? 

A. FJCQ 

B. FIBQ 

C. FICR 

D. GIAQ 

Answer: B. 

Explanation: 

The series follows pattern as, 

(T + 3)(U – 3)(L + 4)(G – 4) = WRPC. 

(W + 3)(R – 3)(P + 4)(C. – 4) = ZOTY 

(Z + 3)(O – 3)(T + 4)(Y – 4) = CLXU 

(C + 3)(L – 3)(X + 4)(U – 4) = FIBQ 

∴ The correct answer is option B. 

 

Q. 3 Fourletter-clusters have been given, out of which three are alike in some manner and one is different. Select the odD. letter-cluster. 

A. PSVX 

B. ZCFI 

C. JMPS 

D. ORUX 

Answer: A. 

Explanation: 

Except option A, remaining all words have difference of 2 letters between in each latter in the word. 

∴ The correct answer is option A. 

 

Q. 4 Select the option that is relateD. to the thirD. number in the same way as the seconD. numberis relateD. to the first number. 13 : 109 :: 24 : ? 

A. 194 

B. 201 

C. 197 

D. 216 

Answer: C. 

Explanation: 

In 13 : 109, 

13×8 + 5 = 104 + 5 = 109 

similarly, 

24×8 + 5 = 192 + 5 = 197 

∴ The correct answer is option C. 

 

Q. 5 Select the number that can replace the Question mark (?) in the following series. 

45, 47, 53, 65, 85, ? 

A. 115 

B. 130 

C. 145 

D. 105 

Answer: A. 

Explanation: 

The series follows pattern as, 

45 + 2 = 47 

47 + (2 + 4 = 6) = 53 

53 + (6 + 6 = 12) = 65 

65 + (12 + 8 = 20) = 85 

85 + (20 + 10 = 30) = 115 

∴ The correct answer is option A. 

 

Q. 6 Select the letter that can replace the Question mark (?) in the following series. 

A, T, E, U, I, V, O, W, ?, X 

A.

B.

C.

D.

Answer: C. 

Explanation: 

There are 2 series, 

A, E, I, O, ? 

and  T, U, V, W, X 

The first series has the the group of vowels. 

So, next letter is u. 

∴ The correct answer is option C. 

 

Q. 7 Select the option in which the given figure X is embeddeD. (rotation is not allowed). 

Answer: C. 

 

Q. 8 Select the option in which the numbersare relateD. in the same way as are the numbers in the givenset. 

(8, 7, 407) 

A. (12, 8, 555) 

B. (10, 8, 576) 

C. (11, 12, 450) 

D. (9, 6, 303) 

Answer: D. 

Explanation: 

In (8, 7, 407), 

8 × 72 + 8 + 7 = 8×49 + 15 

= 392 + 15 = 407 

Similarly, 

In (9, 6, 297), 

9 × 62 + 9 + 6 = 8×36 + 15 

= 288 + 15 = 303 

∴ The correct answer is option D. 

 

Q. 9 Which two signs and two numbers shoulD. be interchangeD. to make the given equation correct? 

28 − 32 ÷ 2 × 8 + 34 = 132 

A. ×and−; 32 and 34 

B. +and−; 28 and  32 

C. +and−; 32 and  34 

D. ×and−; 8 and  2 

Answer: C. 

Explanation: 

28 − 32 ÷ 2 × 8 + 34 = 132 

From the option C, 

28 + 34 ÷ 2 × 8 − 32 = 132 

28 + 17 × 8 − 32 = 132 

136 – 4 = 132 

132 = 132 

∴ The correct answer is option C. 

 

Q. 10 ‘Criticise’ is relateD. to ‘Condemmn’ in the same way as ‘Solidarity’ is relateD. to ‘………….’ 

A. Unity 

B. Actrocious 

C. Guilty 

D. Prosperity 

Answer: A. 

Explanation: 

‘Criticise’ is relateD. to ‘Condemmn’ in the same ways ‘Solidarity’ is relateD. to ‘unity’. 

∴ The correct answer is option A. 

 

Q. 11 Read the given statements and  conclusions carefully. Assuming that the information given in the statements is true, even if it appears to be at variance with commonly known facts, decide which of the given conclusions logically follow(s) from the statements. Statements: 

1. No bottle is A pen. 

2. All jugs are bottles. 

Conclusions: 

I. Some jugs are pens. 

II. No jug is pen. 

III. Some bottles are jugs. 

A. All conclusions I, II and  III follow. 

B. Only conclusions II and  III follow. 

C. Conclusion III and  either conclusion I or II follow. 

D. Either conclusion I or II follows. 

Answer: B. 

Explanation: 

From the Venn diagram, 

Only conclusion II and  III follow. 

∴ The correct answer is option B. 

 

Q. 12 Arrange the following words in A logical and  meaningful order. 

1. Colonel 

2. Field Marshal 

3. Brigadier 

4. Lieutenant 

5. General 

A. 2, 5, 3, 1, 4 

B. 3, 5, 2, 4, 1 

C. 1, 2, 3, 5, 4 

D. 2, 5, 1, 4, 3 

Answer: A. 

Explanation: 

Arrangement of the post according to the rank, 

Field Marshal, General, Brigadier, Colonel, Lieutenant 

∴ The correct answer is option A. 

 

Q. 13 Three different positions of the same dice are shown, the six faces of which are numbered from 1 to 6. Select the number that will be on the face opposite to the one showing ‘6’. 

A.

B.

C.

D.

Answer: A. 

 

Q. 14 In A certain code language, ‘FASTER’ is written as ‘CdQHO’. How will ‘VIOLIN’ be written as in that language? 

A. SLIRLM 

B. SKIRKO 

C. SLRILK 

D. RLTILK 

Answer: C. 

Explanation: 

F – 3 = C 

A + 3 = D 

S + 3 = V 

T – 3 = Q 

E + 3 = H 

R – 3 = O 

Similarly, 

V – 3 = S 

I + 3= L 

O + 3 = R 

L – 3 = I 

I + 3 = L 

N – 3= K 

‘VIOLIN’ is written as ‘SLRILK’. 

∴ The correct answer is option C. 

 

Q. 15 Four words have been given, out of which three are alike in some manner and  one is different. Select the odd word. 

A. Kidney 

B. Pancreas 

C. Renin 

D. Liver 

Answer: C. 

Explanation: 

Except ‘renin’ remaining all are parts of human body. 

∴ The correct answer is option C. 

Q. 16 Select the option in which the words share the same relationship as that shareD. by the given pair of words. 

Player : Team 

A. Flower : Bouquet 

B. Ant : Bee 

C. Student : Teacher 

D. Purpose : Crowd 

Answer: A. 

Explanation: 

A group of players is called team. 

Similarly, 

A group of flowers is called bouquet. 

∴ The correct answer is option A. 

Q. 17 In A class of 100 students, every student has passed in one or more of the three subjects, i.e. History, Economics and  English. Among all the students, 24 students have passed in English only, 14 students have passed in History only, 11 students have passed in both English and  Economics only, and  12 students have passed in both English and  History only. A total of 50 students have passed in History. If only 5 students have passed in all three subjects, then how many students have passed in Economics only? 

A. 15 

B. 18 

C. 20 

D. 10 

Answer: A. 

Explanation: 

Students have passed in English only = 24 

Students have passed in History only = 14 

Students have passed in both English and  Economics only = 11 

Students have passed in both English and  History only = 12 

Students have passed in History = 50 

Students have passed in all three subjects = 5 

Students have passed in Economics only = 100 – students have passed in History – students have passed in English only – students have passed in both English and  Economics only = 100 -50 – 24 – 11 = 15 

∴ The correct answer is option A. 

Q. 18 Select the figure that can replace the Question mark (?) in the following series. 

Answer: D. 

Q. 19 How many triangles are there in the given figure? 

A. 25 

B. 24 

C. 27 

D. 26 

Answer: A. 

Q. 20 Select the correct mirror image of the given figure when A vertical mirror is placed on the right side of the figure.

Answer: B. 

Q. 21 

Study the given pattern carefully and  select the number that can replace the Q. mark (?) in it.

A. 45 

B. 55 

C. 40 

D. 50 

Answer: A. 

Explanation: 

(252 × 3) ÷ 18 = 756/18 = 42 

(296 × 3) ÷ 24 = 888/24 = 37 

(540 ×3) ÷ 36 = 1620/36 = 45

∴The correct answer is option A. 

Q. 22 Which of the following Venn diagramsbest represents the relationship between the following classes? 

Cricketers, Players, Human beings 

Answer: C. 

Q. 23 Four number-pairs have been given, out of which three are alike in some manner and  one is different. Select the number-pair that is different from the rest. 

A. 4209 : 15 

B. 5121 : 12 

C. 8101 : 10 

D. 7456 : 22 

Answer: B. 

Explanation: 

In 4209 : 15 

4 + 2 + 0 + 9 = 15 

In 5121 : 12 

5 + 1 + 2 + 1 = 9 

In 8101 : 10, 

8 + 1 + 0 + 1 = 10 

In 7456 : 22, 

7 + 4 + 5 + 6 = 22 

∴ The correct answer is option B. 

Q. 24 The sequence of folding A piece of paper and  the manner in which the folded paper has been cut is shown in the following figures. How would this paper look when unfolded? 

Answer: B. 

Q. 25 In A certain code language, ‘AUSTERE’ is coded as ‘13102595’. How will ‘SETTING’ be coded as in that language? 

A. 11544957 

B. 10522948 

C. 10522957 

D. 11522867 

Answer: C. 

Explanation: 

A. = 1 

U(21) = 2 + 1 = 3 

S(19) = 1 + 9 = 10 

T(20) = 2 + 0 = 2 

E = 5 

R(18) = 1 + 8 = 9 

E = 5 

Similarly, 

S(19) = 1 + 9 = 10 

E = 5 

T(20) = 2 + 0 = 2 

T(20) = 2 + 0 = 2 

I = 9 

N(14) = 1 + 4 = 5 

G = 7 

‘SETTING’ is codeD. as ‘10522957’. 

∴ The correct answer is option C. 

General Awareness 

Instructions 

For the following questions answer them individually 

Q. 26 Which of the following companies has roped in cricketer Harbhajan Singh as its brand  ambassador in 2019? 

A. Reebok 

B. TAFE 

C. PUMA 

D. Brune & Bareskin 

Answer: D. 

Q. 27 Berach River, A southern-side tributary of Banas River, originates from the hills in which of the following districts of Rajasthan? 

A. Bharatpur 

B. Jaisalmer 

C. Udaipur 

D. Bikaner 

Answer: C. 

Q. 28 Who was honoured with the 1st recipient of the Muppavarapu National Award for Social Service at Hyderabad on 9th January 2020? 

A. D Mankombu Sambasivan Swaminathan 

B. Chandra Pal Singh Yadav 

C. D Gutta Muniratnam 

D. Uttam Prakash Agarwal 

Answer: A. 

Q. 29 Who among the following conceived the Kakori train robbery in 1925? 

A. Ubaidullah Sindhi 

B. Manmath Nath Gupta and  Matangini Hazra 

C. Baikuntha Shukla 

D. Ram Prasad Bismil and  Ashfaqullah Khan 

Answer: D. 

Q. 30 What is the ranking of India in the World Economic Forums (WEF)’s World Travel, Tourism Competitive ness Index for the year 2019? 

A. 31 

B. 34 

C. 37 

D. 39 

Answer: B. 

Q. 31Who among the following scientists invented the Cotton Gin? 

A. Sonny Perdue 

B. Eli Whitney 

C. Norman Borlaugh 

D. George Washington Carver 

Answer: B. 

Q. 32 Which of the following is also known as Epsom salt? 

A. Calcium Sulphate 

B. Magnesium Sulphate 

C. Sodium Sulphate 

D. Ferrous Sulphate 

Answer: B. 

Q. 33 Which of the following is used to control pest fish? 

A. Ovicides 

B. Predacides 

C. Herbicides 

D. Piscicides 

Answer: D. 

Q. 34 In which of the following cities is the tomb of Rani of Jhansi situated? 

A. Indore 

B. Gwalior 

C. Jabalpur 

D. Ujjain 

Answer: B. 

Q. 35 The International Naval event ‘Milan’ is scheduled to be held in ______ in March 2020. 

A. Kochi 

B. Mumbai 

C. Kolkata 

D. Visakhapatnam 

Answer: D. 

Q. 36 Which of the following authorities launched PPRTMS (Political Parties Registration Tracking Management System)? 

A. National Human Rights Commission 

B. The Supreme Court 

C. Election Commission of India 

D. The Union Public Service Commission 

Answer: C. 

Q. 37 Which of the following Viceroys of India addressed the Bengal famine of 1943 by ordering the army to distribute relief supplies to the starving rural Bengalis? 

A. Lord Linlithgow 

B. Lord Mountbatten 

C. Lord Wavell 

D. Lord Willington 

Answer: A. 

Q. 38 Which of the following acids is NOT present in grapes? 

A. Acetic acid 

B. Malic acid 

C. Tartaric acid

D. Citric acid

Answer: A. 

Q. 39 In memory of whom is the Madhavpur Mela of Gujarat celebrated? 

A. Lord Krishna and  Radha 

B. Lord Krishna and  Rukmini 

C. Lord Krishna and  daupadi 

D. Lord Shiva and  Parvati 

Answer: B. 

Q. 40 In which of the following years was the Congress Socialist Party (CSP) founded? 

A. 1924 

B. 1943 

C. 1914 

D. 1934 

Answer: D. 

Q. 41 According to data available as of 2019, how many national parks are situated in India? 

A. 104 

B. 115 

C. 140 

D. 131 

Answer: A. 

Q. 42 Who among the following is the chairman of the 15th Finance Commission of India? 

A. Ramesh Chand  

B. Subhash Chandra Garg 

C. NK Singh 

D. Arvind Mehta 

Answer: C. 

Q. 43 Who among the following founded the Bombay Stock Exchange? 

A. Vikramjit Sen 

B. Roychand  depchand  

C. Premchand  Roychand  

D. Ashiskumar Chauhan 

Answer: C. 

Q. 44 Who among the following Indian weight lifters has been banned for four years by NAD after being found guilty of A doping violation during the 34th Women Senior National Weight lifting Championship held at Vishakhapatnam? 

A. Sanamacha Thingbaijani Chanu 

B. Seema

C. Satheesha Rai 

D. Sarbjeet Kaur 

Answer: D. 

Q. 45 Who among the following has been appointed as director of National Book Trust on 11th January 2020? 

A. Gobinde Prashad Sharma 

B. Lt Col Yuvraj Malik 

C. Leela Samson 

D. RitA. Chowdhury 

Answer: B. 

Q. 46 Which of the following academies organise the Tansen Sangeet Samaroh in Gwalior? 

A. Renuka Sangeet Academy 

B. Muntirama Sangeet Academy 

C. Ravindra Jain Sangeet Academy 

D. Ustad Alauddin Khan Kala Evam Sangeet Academy 

Answer: D. 

Q. 47 In which of the following states was the 12th naval exercise ‘Naseem-Al-Bahr’ between Indid and  Oman held? 

A. Kerala 

B. Gujarat 

C. Goa 

D. Tamilnadu 

Answer: C. 

Q. 48 Which of the following is used as a substitute of mercury in thermometers? 

A. Selenium 

B. Arsenic 

C. Galinstan 

D. Bromine 

Answer: C. 

Q. 49 

How many national highways were merged to form National Highway 44 (NH 44)? 

A.

B.

C.

D.

Answer: A. 

Q. 50 Under which Article of the Indian Constitution are special provisions provided to language spoken in a section of the population? 

A. Article 347 

B. Article 374 

C. Article 357 

D. Article 337 

Answer: A. 

Quantitative Aptitude 

Instructions 

For the following questions answer them individually 

Q. 51 (a + b + c d)2 − (a b c + d)2 =? 

A. 2a(b + c d)

B. 4a(b + c d)

C. 2a(b + c + d)

D. 4a(b + c + d)

Answer: B. 

Explanation: 

(a + b + c d)2 − (a b c + d)2 

[(a + b + c d)+(a b c + d)][(a + b +c d) – (a b c + d)] 

( ∵ a2 b2 = (a + b)(a b)) 

= (2a)(2a + 2c – 2d) 

= 4a(a + c – d) 

Q. 52 In the given figure, if DE BC, AD = 2.5 cm, D = 3.5 cm and  EC = 4.2 cm, then the measure of AC is:

A. 3 cm 

B. 7.2 cm 

C. 7.4 cm 

D. 3.2 

Answer: B. 

Q. 53 The perimeter of Asquare is 64 cm. Its area will be: 

A. 256 cm2

B. 8 cm2

C. 32 cm2

D. 128 cm2

Answer: A. 

Explanation: 

The perimeter of A square = 64 cm 

4× side = 64 

Side = 64/4 = 16 cm 

Area = (side)2 = (16)2 = 256 cm2

Q. 54 The price of sugar is increaseD. by 20%. By what percentage must one cut down on the consumption of sugar, so that no extra amount has to be incurreD. on sugar? 

A. 16⅔ %

B. 20%

C. 80%

D. 80⅓ %

Answer: A. 

Explanation: 

Percentage decrements in the consumption of sugar = rate /(100+rate) × 100 

20 /(100+20) × 100

20 /(120) × 100 = 16⅔ %

Q. 55 A furniture shopkeeper allows a discount of 16% on the marked price on the goods to his customers and  still gains 20%. What is the marked price of a dining-table, which costs the shopkeeper ₹11,900? 

A. ₹15,000 

B. ₹12,376 

C. ₹17,000 

D. ₹16,000 

Answer: C. 

Explanation: 

Cost price of dining table = Rs.11900 

Profit = 20% 

Selling price = 11900 × 120/100= Rs.14280 

discount = 16% 

Let the marked price 100%. 

Selling price = 100 – 16 = 84% 

84% = 14280 

100% = 14280 / 84 × 100 = 17000 

MarkeD. price of dining table = Rs.17000 

Q. 56 The coefficient of x2 in (2x + y)3 is: 

A. 8

B. 12y2

C. 12y

D. 12

Answer: C. 

Explanation: 

(2x + y)3 

= (2x)3 + y3 + 3.2x.y(2x + y

(∵ (a + b)3 = a3 + b3 + 3ab(a + b) ) 

= 8x3 + y3 + 6xy(2x + y)

= 8x3 + y3 + 12x2y + 6xy22 

The coefficient x2 of = 12y 

Q. 57 If tan θ − cot θ = cosec θ, 0< θ < 90, then what is the value of (2 tan θ−cos θ )/(√3 cot θ+sec θ)

A. (3√3−1)/ 6

B. (4√3−1)/ 6

C. (2√3−1)/ 3

D. 2(2√3−1)/ 3

Answer: B. 

Explanation: 

tan θ − cot θ = cosec θ 

tan θ 1/tanθ = cosec θ 

tan2 θ − 1 = cosec θ tan θ 

sec2 θ − 1 − 1 = sec θ 

sec2 θ − sec θ − 2 = 0 

sec2 θ − 2 sec θ + sec θ − 2 = 0 

sec θ(sec θ − 2) + 1(sec θ − 2) = 0 

(sec θ + 1)(sec θ − 2) = 0 

0< θ < 90 

For , 

sec θ = 2 

cos θ = 1/2 

θ = 60° 

Now, 

(2 tan θ−cos θ )/(√3 cot θ+sec θ)

Put the value of θ,

(2 tan 60°−cos 60° )/(√3 cot 60°+sec 60°)

(2√3− ½ )/(√3×(1/√3)+2)

(4√3−1)/ 6

Q. 58 The value of −1 + 1/4  ÷ 1/2 × 2 + 5  is: 

A. 17/ 4

B. 5

C. 2

D. −7/ 2

Answer: B. 

Explanation: 

−1 + 1/4  ÷ 1/2 × 2 +5

= −1 + 1/4 × 2 × 2 +5

= −1 + 1 +5 = 5

Q. 59 The value of cos 0 cos 30 cos 45 cos 60 cos 90is: 

A.

B.

C.

D. √6 / 8

Answer: C. 

Explanation: 

cos 0 cos 30 cos 45 cos 60 cos 90 = 0

(∵ cos 0= 0 ) 

Q. 60 The heights of some girls in a school were noted and  the data obtained are as shown in the table.

How many girls have a height of 135 cm or more but less than 150 cm? 

A. 88 

B. 35 

C. 80 

D. 64 

Answer: D. 

Explanation: 

Number of girls have A. height of 135 cm or more but less than 150 cm = 20 + 29 + 15 = 64 

Q. 61 Sudhir purchased a laptop for ₹42,000 and  a scanner-cum-printer for ₹8,000. He sold the laptop for a 10% profit and  the scanner cum-printer for a 5% profit. What is his profit percentage? 

A. 9⅖ % 

B. 9⅕ %

C. 15 %

D. 7½ % 

Answer: B. 

Explanation: 

Cost price of laptop = Rs.42000 

Profit = 10% 

Selling price = 42000 ×110 /100= Rs.46200 Cost price of printer = Rs.8000 

Profit = 5%

Selling price = 8000 ×105 /100 = Rs.8400 Total selling price = 46200 + 8400 = Rs.54600 Total cost price = 42000 + 8000 = Rs.50000 Total profit = 54600 – 50000 = Rs.4600 

Total profit% = 4600/50000 × 100 = 9⅕ %

Q. 62 The following table shows the income(in rupees) for a particular month, together with their source, in respect of 5 employees (A, B, C, D and  E). 

How many employees got more arrears than the average arrears received by all the employees? 

A.

B.

C.

D.

Answer: C. 

Explanation: 

The average arrears received by all the employees = sum of terms/number of terms =(5500+4500+4000+3000+1500)/5 

= 18500/5 = 3700 

Required number = 3 

∴ 3 employees(A, B and C) got more arrears than the average arrears received by all the employees. 

Q. 63 In the figure, Two circles with centres P and  Q touch externally at R. Tangents AT and  BT meet the common tangent TR at T. If AP = 6 cm and  PT = 10 cm, then BT = ? 

A. 6 cm 

B. 8 cm 

C. 10 cm 

D. 12 cm 

Answer: B. 

Q. 64 If x 1/x = 11, then x3 1/x3 = ? is: 

A. 1474 

B. 1364 

C. 1188 

D. 1298 

Answer: D. 

Explanation: 

x 1/x = 11

x31/x3 = (x 1/x)3 -3(x 1/x)

∵ (a b)3 = a3 b3 − 3ab(a b

= (11)3-3(11)

= 1331 – 33 = 1298 

Q. 65 The following table shows the monthly salaries (in ₹) of 50 families.

How many families have a monthly salary less than ₹40,000? 

A. 46 

B.

C. 27 

D. 18 

Answer: C. 

Explanation: 

Number of families which have monthly salary less than ₹40,000 = 6 + 12 + 9 = 27 

Q. 66 The greatest number which should be replace ‘*’ in the number 146*48 to make it divisible by 8 is: 

A.

B.

C.

D.

Answer: A. 

Explanation: 

For the divisible by 8, last 3 digits of the number shoulD. be divisible by 8 so, *48 divisible by 8. 

From the option A), 

Put the value of x = 8, 

Last 3 digit = 848 

It is divisible by 8 so, greatest value of * will be 8. 

Q. 67 The following table shows the number of students enrolled in different faculties in a college.

The percentage of students studying in faculties other than Science is: 

A. 60% 

B. 80% 

C. 75% 

D. 20% 

Answer: B. 

Explanation: 

Total number of student = 32 + 18 + 28 + 45 + 42 + 42 + 13 + 30 = 250 

Total number of student in Arts, commerce, vocational = 28 + 45 + 42 + 42 + 13 + 30 = 200 

Required percentage =200/250 × 100 = 80% 

Q. 68 The distance between two stations, A and  B, is 428 km. A train starts from station ‘A’ at 6:00 a.m. and  moves towards station ‘B’ at an average speed of 48 km/h. Another train starts from station ‘B’ at 6:20 a.m. and  moves towards station ‘A’ at an average speed of 55 km/h. At what time will the trains meet? 

A. 10 : 20 a.m. 

B. 9 : 40 a.m. 

C. 10 : 40 a.m. 

D. 10:00 a.m. 

Answer: A. 

Explanation: 

distance from A to B = 428 km 

Speed of train A = 48 km/hr 

Speed of train B = 55 km/hr 

Train A starts at 6:00 am and  B start at 6:20 so, 

distance covered by by train A in 20 min(20/60 hr) = speed × (20/60) = 48 × (1/3) = 16 km Remaining distance = 428 – 16 = 412 km 

Trains run in opposite direction so relative speed = 48 + 55 = 103 km/hr 

Time taken to meet = 412/103 = 4 hr 

Time when trains meet = 6:20 + 4 = 10:20 am 

Q. 69 A certain amount of money at compound interest grows to ₹66,550 in 3 years and  ₹73,205 in 4 years. The rate per cent per annum is: 

A. 9% 

B. 5% 

C. 11% 

D. 10% 

Answer: D. 

Explanation: 

difference in the interest in 4 years and  3 years = 73205 – 66550 = Rs.6655 

Rate percentage =6655 / 66550 × 100 = 10% 

Q. 70 The average height of 5 boys is 175 cm. a sixth boy joined the group and  the average height of all the boys in the group now increased by 1 cm. The height of the sixth boy is: 

A. 180 cm 

B. 181 cm 

C. 175 cm 

D. 179 cm 

Answer: B. 

Explanation: 

The average height of 5 boys = 175 cm 

Average = sum of all terms/number of terms 

Sum of the height of 5 boys = 175×5 = 875 

Average height after joining of 6th boys = 175 + 1 = 176 

Sum of the height of 6 boys = 176×6 = 1056 

Height of 6th boy = 1056 – 875 = 181 cm 

Q. 71 If angles of a triangle are in the ratio of 2 : 3 : 4, then the measure of the smallest angle is: 

A. 50

B. 30

C. 40

D. 20

Answer: C. 

Explanation: 

Ratio of angles = 2 : 3 : 4 

Let the angles be 2x, 3x and  4x.  

We know the sum of the angles of a triangle is 180 

2x + 3x + 4x = 180 

x = 180/9 = 20

Smallest angle = 2x = 2 ×20 = 40

Q. 72 Sixteen men can finish a work in 8 days. Eight men and  nine women working together can finish the same work in 10 days. In how many days will twenty women finish the same work? 

A.

B. 12 

C. 11 

D. 13 

Answer: B. 

Explanation: 

Total work = man/woman × time 

Work is same so, 

16m × 8 = (8m + 9w) \times 10 

⇒ 128m = 80m + 90w 

⇒ 48m = 90w 

⇒ 8m = 15w 

Let the 20 women complete work in days so, 

Total work = 20w d 

⇒ (8m + 9w) × 10 = 20w d 

⇒ (15w + 9w) × 10 = 20w d 

⇒ 24w × 10 = 20w d 

⇒ d = 12 days 

∴ Twenty women finish the same work in 12 days. 

Q. 73 In the given figure, if KLN = 58, then KMN =?

A. 26 

B. 42 

C. 58

D. 32 

Answer: C. 

Q. 74 The value of √(tan 60 + sin 90) − 2 tan 45is: 

A.

B.

C.

D.

Answer: A. 

Explanation: 

√(tan 60 + sin 90) − 2 tan 45 

On putting the values, 

√{√3)2 + 1} − 2 × 1 

= 2 – 2 = 0 

Q. 75 A certain amount is divided among Sunita, Amit and  Vibha in the ratio of 2 : 3 : 4. If Vibha gets ₹14,416, then the total amount is: 

A. ₹16,219 

B. ₹3,604 

C. ₹43,248 

D. ₹32,436 

Answer: D. 

Explanation: 

Ratio of the amount of Sunita, Amit and  Vibha = 2 : 3 : 4 

Let the amount of Sunita, Amit and  Vibha be 2x, 3x and  4x. 

Amount of Vibha = 14416 

4x = 14416 

x = 14416/4 = 3604 

Total amount = 2x + 3x + 4x = 9x 

= 9 × 3604 

= Rs.32436 

English Comprehension 

Instructions 

For the following questions answer them individually 

Q. 76 Select the most appropriate option to substitute the underlined segment in the given sentence. If no substitution is required, select No improvement. 

I wish if I was the Principal of my school. 

A. If I haD. been 

B. I were 

C. No improvement 

D. that I am 

Answer: B. 

Q. 77 Select the passive form of the given sentence. 

People claim to have seen the wild and  mad elephant in several cities. 

A. The wild and  mad elephant has been claimed to be seen by the people in several cities. 

B. The wild and  mad elephant is being claimed to be seen in several cities. 

C. The wild and  mad elephant is claimed to have been seen in several cities. 

D. The wild and  mad elephant was claimed to be seen by people in several cities. 

Answer: C. 

Q. 78 In the sentence identify the segment which contains the grammatical error. 

Lucy has already cut the cake when we reached the party hall. 

A. the party hall 

B. Lucy has already 

C. when we reached 

D. cut the cake 

Answer: D. 

Q. 79 Given below are four jumbled sentences. Out of the given options pick the one that gives their correct order. 

A. There are many gadgets that are operated by a remote control and  contain a silicon chip. 

B. This produces an infra-red beam, made up of electromagnetic waves. 

C. When a button is pressed on the remote control, the chip sets off an electronic vibration. 

D. The beam carries a coded signal for operating the systems. 

A. ADdB 

B. ACD 

C. ACBD 

D. BACD 

Answer: C. 

Instructions 

Select the most appropriate word to fill in the blank. 

Q. 80 Health should be our first priority and  should never be ______. 

A. neglected 

B. suspended 

C. discarded 

D. declined 

Answer: A. 

Q. 81 A colour blind person is not able to ______ between colours. 

A. glance 

B. distinguish 

C. define 

D. observe 

Answer: B. 

Instructions 

In the following passage some words have been deleted. Fill in the blanks with the help of the alternatives given. Select the most appropriate option for each blank. 

Comprehension: 

Corruption is one of the (1)______ problems of our country. It is an act done with (2)______ to gain some undue advantages, inconsistent with official duty. Government has (3)______ many schemes for the welfare of the people but corruption seems impossible to be (4)______ IndiA. can become A. corruption-free nation only when all the citizens (5)______ rules and  regulations with A. strong will power and  cooperate whole heartedly. 

Q. 82 Select the most appropriate option for blank No. 1. 

A. powerful 

B. minor 

C. major 

D. strong 

Answer: C. 

Q. 83 Select the most appropriate option for blank No. 2. 

A. compulsion 

B. intention 

C. intuition 

D. passion 

Answer: B. 

Q. 84 Select the most appropriate option for blank No. 3. 

A. denounced 

B. written 

C. announced

D. concealed 

Answer: C. 

Q. 85 Select the most appropriate option for blank No. 4. 

A. eliminated 

B. permitted 

C. initiated 

D. created 

Answer: A. 

Q. 86 Select the most appropriate option for blank No. 5. 

A. reject 

B. disobey 

C. break 

D. follow 

Answer: D. 

Instructions 

For the following questions answer them individually 

Q. 87 Select the most appropriate antonym of the given word. 

Hostile 

A. Amiable 

B. Violent 

C. Unfavourable 

D. Adverse 

Answer: A. 

Q. 88 Select the correctly spelt word. 

A. dfinite 

B. dfinete 

C. dfiniet 

D. dfinate 

Answer: A. 

Q. 89 Select the most appropriate antonym of the given word. 

Sluggish 

A. Active 

B. dll 

C. Sedentary 

D. Weak 

Answer: A. 

Q. 90 Identify the part of the sentence that contains the grammatical error. 

One of the greatest responsibilities of a scientist is that his discoveries and  inventions are utilized to the overall development of the nation. 

A. is that his discoveries and  inventions 

B. are utilized to the overall 

C. One of the greatest responsibilities of a scientist 

D. development of the nation 

Answer: B. 

Q. 91 Select the indirect narration of the given sentence. 

He said to me,” What time do the banks open and  close?” 

A. He asked me what time do the banks open and  close. 

B. He said me what time did the banks open and  close. 

C. He wanted to know what time the banks open and  close. 

D. He asked me what time the banks opened and  closed. 

Answer: D. 

Q. 92 Select the most appropriate synonym of the given word. 

deficiency 

A. Abundance 

B. Sufficiency 

C. Inadequacy 

D. Efficiency 

Answer: C. 

Q. 93 Given below are four jumbled sentences. Out of the given options pick the one that gives their correct order. 

A. It is called the atmosphere. 

B. All parts of the earth are surrounded by air. 

C. Living beings breathe in and  breathe out the air all the time. 

D. This process is called respiration. 

A. DCB

B. BACD 

C. CBAD 

D. ACD 

Answer: B. 

Q. 94 Select the most appropriate meaning of the given Idiom. 

To add fuel to the fire. 

A. Exploit someone 

B. Simplify the problem 

C. To make A bad situation worse 

D. Calm down the situation 

Answer: C. 

Q. 95 Select the correctly spelt word. 

A. Commemorate 

B. Commemmorate 

C. Comemorate 

D. Comemmorate 

Answer: A. 

Q. 96 Select the most appropriate meaning of the given Idiom. 

Hobson’s Choice 

A. To be in an aggressive mood when facing A challenge 

B. To challenge an unworthy opponent 

C. An apparently free choice where there is no real alternative 

D. To exercise the choice to surrender before the enemy 

Answer: C. 

Instructions 

Select the most appropriate word that means the same as the group of words given. 

Q. 97 A short story with A moral, usually with animals as characters. 

A. Hymn 

B. Fable 

C. Parody 

D. Ode 

Answer: B. 

Q. 98 Enclosed area where aircraft are kept and  repaired 

A. Hangar 

B. Harbinger 

C. Hose 

D. Hanger 

Answer: A. 

Instructions 

For the following questions answer them individually 

Q. 99 Select the most appropriate option to substitute the underlined segment in the given sentence. If no substitution is required, select No improvement. 

Amit has the good command  on English as he was born and  brought up in England  

A. A better command  of 

B. the best command  in 

C. A good command over 

D. No improvement 

Answer: C. 

Q. 100 Select the most appropriate synonym of the given word. 

Catastrophic 

A. dsastrous 

B. Mysterious 

C. Tough 

D. dlicate 

Answer: A. 

SSC CGL Tier-I 05 March 2020 Shift-II Previous Year Paper

SSC CGL Tier-I 05 March 2020 

Q. 1 Vineet, Rajesh and Kriti have different amounts of money with them. Rajesh has just double the amount of money than Vineet. The total amount of money that Rajesh and Kriti have is ₹147. Kriti has ₹6 more than the amount Vineet has. How much money does Kriti have ? 

A. ₹64 

B. ₹53 

C. ₹50 

D. ₹72 

Answer: B. 

Explanation: 

Amount of Rajesh = 2×amount of Vineet 

Amount of Kriti = 6 + amount of Vineet —(1) 

Total amount of Rajesh and Kriti = 147 

Amount of Rajesh + amount of Kriti = 147 

2×amount of Vineet + 6 + amount of Vineet = 147 

Amount of Vineet = 141/3 = 47 

From eq (1), 

Amount of Kriti = 6 + 47 = Rs. 53 

∴ The correct answer is option B. 

 

Q. 2 The sequence of folding a piece of paper and the manner in which the folded paper has been cut is shown in the following figures. How would this paper look when unfolded? 

Answer: C. 

 

Q. 3 Select the letter-cluster that can replace the question mark(?) in the following series. SAT, VEW,YIZ, ? , EUF 

A. BOC. 

B. BUK 

C. FIC. 

D. COD. 

Answer: A. 

Explanation: 

In the series, middle letter of the letter-cluster is vowels so, 

(S + 3)(A)(T + 3) = VEW 

(V + 3)(E)(W + 3) = YIZ 

(Y + 3)(I)(Z + 3) = BOC. 

(B. + 3)(O)(C. + 3) = EUF 

∴ The correct answer is option A. 

 

Q. 4 Select the option in which the numbers are related in the same way as are the numbers in the given set. (7, 98, 196) 

A. (15, 190, 380) 

B. (18, 185, 360) 

C. (11, 154, 308) 

D. (20, 267, 520) 

Answer: C. 

Explanation: 

In (7, 98, 196), 

7×14 = 98 

98×2 = 196 

Similarly, 

In (11, 154, 308), 

11×14 = 154 

154×2 = 308 

∴ The correct answer is option C. 

 

Q. 5 Select the dice that can be formed by folding the given sheet along the lines.

A. Only A. and B. 

B. Only A. 

C. Only B. 

D. Only C. and D. 

Answer: B. 

 

Q. 6 Arrange the following words in the order in which they would appear in an English dictionary. 

1. Category 

2. Caption 

3. Captain 

4. Capsule 

5. Capacity 

A. 5,4,3,1,2 

B. 5,4,3,2,1 

C. 4,5,3,2,1 

D. 5,4,1,3,2 

Answer: B. 

Explanation: 

Order of the words according to the English dictionary, 

Capacity, Capsule, Captain, Caption, Category 

∴ The correct answer is option B.

 

Q. 7 Study the given pattern carefully and select the number that can replace the question mark (?) in it?

A. 14 

B. 24 

C. 30 

D. 28 

Answer: A. 

Explanation: 

(299+100)/21 = 399/21 = 19

(296+100)/18 = 396/18 = 14

Similarly,

(348+100)/32 = 448/32 = 14

∴ The correct answer is option A. 

 

Q. 8 Four Letter-clusters have been given, out of which three are alike in some manner and one is different. Select the odd letter-cluster. 

A. NRW 

B. PTY 

C. ADL 

D. LPU 

Answer: C. 

Explanation: 

N + 4 = R + 5 = W 

P + 4 = T + 5 = Y 

A + 3 = D + 8 = L 

L + 4 = P + 5 = U 

∴ The correct answer is option C. 

 

Q. 9 Four words have been given. out of which three are alike in some manner and is different. 

Select the odd word. 

A. Rabies 

B. Schizophrenia 

C. Polio 

D. Measles 

Answer: B. 

Explanation: 

Except option B, remaining all are the disease which is caused by the virus. 

∴ The correct answer is option B. 

 

Q. 10 Pointing to the photograph of Sanchi, Nitin said, “Her mother’s father’s son’s wife is my mother-in-law’s only daughter”. How is Nitin related to Sanchi’s mother ? 

A. Paternal uncle 

B. Paternal grandfather 

C. Maternal uncle 

D. Brother 

Answer: D. 

Explanation: 

In the diagram, square shows the males, circle shows the females, vertical lines shows the generation, single horizontal lines shows the brother/sister and double horizontal lines shows the couples. 

From the diagram, 

Nitin is the brother of Sanchi’s mother. 

∴ The correct answer is option D. 

 

Q. 11 Which of the following Venn diagram best represents the relationship between the following classes? 

Income tax payers, Employees, Males 

A.

B.

C.

D.

Answer: B. 

 

Q. 12 ‘Vertebrate’ is related to ‘Monkey’in the same wayas ‘Invertebrate’is related to ‘ ……..’ 

A. Starfish 

B. Snake 

C. Deer 

D. «Frog 

Answer: A. 

Explanation: 

‘Vertebrate’ is related to ‘Monkey’in the same wayas ‘Invertebrate’is related to ‘starfish’. 

∴ The correct answer is option A. 

 

Q. 13 Select the option that is related to the third number in the same way as the second number is related to the first number. 

4:69:: 11:2 

A. 1029 

B. 198 

C. 176 

D. 1336 

Answer: D. 

Explanation: 

(4)3 + 3 = 64 + 5 = 69 

(11)3 + 3 = 1331 + 5 =1336 

∴ The correct answer is option D. 

 

Q. 14 In a certain code language. ‘CAUGHT’ is coded as ‘326212087’. How will ‘SOLDER’ be coded asin that language? 

A. 2012152358 

B. 1912122359 

C. 1915124359 

D. 1812122459 

Answer: B. 

Explanation: 

‘CAUGHT’ is coded as ‘326212087’, 

C = 3 

A = 27 – 1 = 26 

U = 21 

G = 27 – 7 =20 

H = 8 

T = 27 – 20 = 7 

Similarly, 

S = 19 

O = 27 – 15 = 12 

L = 12 

D = 27 – 4 = 23 

E = 5 

R = 27 – 18 = 9 

‘SOLDER’ is coded as ‘1912122359’ 

∴ The correct answer is option B. 

 

Q. 15 Which two signs and two numbers should be interchanged to make the given equation correct? 

11 × 7 ÷ 35 − 64 + 56 = 47

A. + and −; 7 and 11

B. × and ÷; 35 and 11

C. × and ÷; 35 and 56

D. + and −; 35 and 11 

Answer: B. 

Explanation: 

11 × 7 ÷ 35 − 64 + 56 = 47 

From the option B, 

on changing the sign, 

35 ÷ 7 × 11 − 64 + 56 = 47 

55 – 64 + 56 = 47 

47 = 47 

∴ The correct answer is option B. 

 

Q. 16 Select the correct mirror image of the given figure when a mirror is placed on its right side. 

Answer: B. 

 

Q. 17 Select the letter-cluster that can replace the question mark (?) in the following series. 

AR, CP, ?, GL, IJ, KH 

A. DM 

B. DN 

C. EN 

D. EM 

Answer: C. 

Explanation: 

(A + 2)(R – 2) = CP 

(C + 2)(P – 2) = EN 

(E + 2)(N – 2) = GL 

(G + 2)(L – 2) = IJ 

(I + 2)(J – 2) = KH 

∴ The correct answer is option C. 

Q. 18 Select the number that can replace the question mark (?) in the following series. 

61, 63, 65, 77, 89, ? , 149, 205 

A. 115 

B. 123 

C. 119 

D. 132 

Answer: C. 

Explanation: 

61 +(1)2 + 1 = 61 + 2 = 63 

63 +(2)2 – 2 = 63 + 2 = 65  

65 +(3)2 + 3 = 65 + 12 = 77 

77 +(4)2 – 4 = 77 + 12 = 89 

89 +(5)2 + 5 = 89 + 30 = 119 

119 +(6)2 – 6 = 119 + 30 = 149 

149 +(7)2 + 7 = 149 + 56 = 205 

∴ The correct answer is option C. 

Q. 19 Four number-pairs have been given, out of which three are alike in some manner and one is different. Select the number-pair that is different from therest 

A. 17:323 

B. 23: 575 

C. 19:475 

D. 11: 143 

Answer: C. 

Explanation:

17 × (17 + 2) = 17 × 19 = 323 

23 × (23 + 2) = 23 × 25 = 575 

19 × (19 + 2) = 19 × 21 = 399 

11 × (11 + 2) = 11 × 13 = 143 

∴ The correct answer is option C. 

Q. 20 Select the figure that can replace the question mark (?) in the following series. 

Answer: C. 

Q. 21 In a certain code language, ‘SANCTION’ is written as ‘XFSHODIT’. How will ‘PROFOUND’ be written as in that language ? 

A. UWTKJPHJ 

B. UWTKKPIJ 

C. UWTKNQIY 

D. VWUKMPIY 

Answer: A. 

Explanation: 

S + 5 = X 

A + 5 = F 

N + 5 = S 

C + 5 = H 

T – 5 = O 

I – 5 = D. 

O – 6 = I 

N + 6 = T 

Similarly, 

P + 5 = U 

R + 5 = W 

O + 5 = T 

F + 5 = K 

O – 5 = J 

U – 5 = P 

N – 6 = H 

D + 6 = J 

‘PROFOUND’ is written as ‘UWTKJPHJ’. 

∴ The correct answer is option A. 

Q. 22 Select the option in which the words share the same relationship as that shared by the given pair of words. Examination: Invigilator 

A. Election : Observer 

B. Bank : Loan 

C. Film : Comedian 

D. Inspection : Approval 

Answer: A. 

Explanation: 

Examinations are conducted by the invigilator. 

Similarly, 

Elections are conducted by the observer. 

∴ The correct answer is option A. 

Q. 23 How many triangles are there in the given figure? 

A. 34 

B. 33 

C. 32 

D. 35 

Answer: C. 

Q. 24 Select the option that is embedded in the given figure X (rotation is not allowed). 

Answer: C. 

Q. 25 Read the given statements and conclusions carefully. 

Assuming that the information given in the statements is true, even if it appears to be at variance with commonly known facts, decide which of the given conclusions logically follow from the statements. Statements: 

1. Some papers are copies. 

2. No copy is an eraser. 

Conclusions: 

I. Some copies are papers. 

II. Some papers are erasers. 

III. No paper is an eraser. 

A. Only conclusion II and either conclusion I or III follow. 

B. Only conclusions I and II follow. 

C. Only conclusion I and either conclusion II or III follow. 

D. Only conclusions I and III follow. 

Answer: C. 

Explanation: 

From the Venn diagram, 

Only conclusion I and either conclusion II or III follow 

∴ The correct answer is option C. 

General Awareness 

Instructions 

For the following questions answer them individually 

Q. 26 The Indian Army is set to commission the first batch of women soldiers by year ……. 

A. 2020 

B. 2022 

C. 2021 

D. 2023 

Answer: C. 

Q. 27 Part VIII of the Constitution of India deals with ______. 

A. States 

B. Municipalities 

C. Union Territories 

D. Panchayats 

Answer: C. 

Q. 28 Which of the following is NOT located in Odisha? 

A. Mayurbhanj Elephant Reserve 

B. Sambalpur Elephant Reserve 

C. Sonitpur Elephant Reserve 

D. Mahanadi Elephant Reserve 

Answer: C. 

Q. 29 Which of the following is NOT an ore of iron? 

A. Cuprite 

B. Haematite 

C. Magnetite 

D. Siderite 

Answer: A. 

Q. 30 The National Commission for Backward Classes (NCBC) was formed by insertion of Article ______ in the Constitution of India. 

A. 328B 

B. 328A 

C. 338A 

D. 338B 

Answer: D. 

Q. 31 

Name the first female amputee to climb Mount Everest. 

A. Premlata Agarwal 

B. Arunima Sinha 

C. Poorna Malavath 

D. Anshu Jamsenpa 

Answer: B. 

Q. 32 Pernicious anaemia is caused by the deficiency of vitamin ……. 

A. B2 

B. B6 

C. B1 

D. B12 

Answer: D. 

Q. 33 India won its ______ successive title at the South Asian Football Federation Women’s Championship in March 2019 

A. fourth 

B. sixth 

C. second 

D. fifth 

Answer: D. 

Q. 34 The excavated remains of ______ found near Patna in Bihar was inscribed as a World Heritage Site by UNESCO in July 2016. 

A. Ashoka Pillar 

B. Ellora Caves 

C. Sanchi Stupa 

D. Nalanda Mahavihara 

Answer: D. 

Q. 35 The headquarters of the Board of Control for Cricket in India (BCCI) are located in 

A. Bengaluru 

B. Mumbai 

C. Kolkata 

D. Delhi 

Answer: B. 

Q. 36 Name the first Lieutenant Governor of the Union Territory of Jammu and Kashmir. 

A. Arif Mohammad Khan 

B. Bhagat Singh Koshyari 

C. Satya Pal Malik 

D. Girish Chandra Murmu 

Answer: D. 

Q. 37 ______ is also known as the ‘Golden City 

A. Dungarpur 

B. Jodhpur 

C. Bundi 

D. Jaisalmer 

Answer: D. 

Q. 38 The aggregate value of goods and services produced in an economy can be calculated by three methods: income method, expenditure method and ______ method. 

A. product / value added 

B. spending 

C. deposit 

D. lending 

Answer: A. 

Q. 39 ……. renounced his knighthood in protest for Jalianwalla Bagh mass killing 

A. Jamsetjee Jejeebhoy 

B. Rabindranath Tagore 

C. Shivajirao Holkar 

D. Surendranath Banerjee 

Answer: B. 

Q. 40 ‘Dharmaraja (Yudhishthir) Ratha’, ‘Bhima Ratha’, ‘Arjuna Ratha’ and ‘Nakula Sahadeva Ratha’ are four of the Panch Rathas at Mahabalipuram. What is the name of the fifth Ratha? 

A. Bhishma Ratha 

B. Krishna Ratha 

C. Karna Ratha 

D. Draupadi Ratha 

Answer: D. 

Q. 41 The ______ in China is the world’s longest man-made waterway 

A. Suzhou Canal 

B. Kiel Canal 

C. Corinth Canal 

D. Grand Canal 

Answer: D. 

Q. 42 The Gadar (or Ghadar) Party was formed in the year ……. 

A. 1921 

B. 1918 

C. 1915 

D. 1913 

Answer: D. 

Q. 43 Alai-Darwaza, the southern gateway of the Quwwat-ul-Islam mosque in Delhi, was constructed by ….. 

A. Mu’izz ad-Din Muhammad Ghori 

B. Ahmad Shāh Durrani 

C. Ala-ud-din Khilji 

D. Muhammad bin Tughlu 

Answer: C. 

Q. 44 The Indus Waters Treaty was signed between India and Pakistan on ……. 

A. 19 September 1960 

B. 12 November 1959 

C. 16 December 1963 

D. 18 October 1969 

Answer: A. 

Q. 45 ______ are homogeneous mixtures of two or more components. 

A. Amalgams 

B. Solutions 

C. Enzymes 

D. Emulsions 

Answer: B. 

Q. 46 The total number of images formed by two mirrors inclined at 120° to each other is …… 

A.

B.

C.

D.

Answer: C. 

Q. 47 Depreciation is an annual allowance for the wear and tear of ………. 

A. work in progress 

B. land 

C. finished goods 

D. capital goods 

Answer: D. 

Q. 48 The Padma Awards are announced around ______ every year. 

A. Republic Day 

B. Sadbhavana Diwas 

C. Independence Day 

D. Hindi Diwas 

Answer: A. 

Q. 49 Which of the following can be used as a catalyst in Hydrogenation reaction? 

A. Barium 

B. Iron 

C. Tungsten 

D. Palladium 

Answer: D. 

Q. 50 The ‘Hyderabad Fund’ had been held by the ______ in the account of the High Commissioner of Pakistan to the UK, Habib Ibrahim Rahimtoola 

A. Bank of England 

B. National Westminster Bank 

C. Butterfield Private Bank 

D. Kingdom Bank 

Answer: B. 

Quantitative Aptitude 

Instructions 

For the following questions answer them individually 

Q. 51 In ABC, ∠B = 90, If the points D and E are on the side BC such that BD = DE = EC then which of the following is true? 

A. 5AE2 = 2AC2 + 3AD2

B. 8AE2 = 5AC2 + 3AD2 

C. 8AE2 = 3AC2 + 5AD2

D. 5AE2 = 3AC2 + 2AD2

Answer: C. 

Explanation: 

Let the BD, BE and BC. be x, 2x and 3x respectively. 

In △ABD, 

(AD)2 = (AB)2 + (BD)2 

(AD)2 = (AB)2 + (x)2  —-(1)

In △ABE,

(AE)2 = (AB)2 + (BE)2 (AE)2 = (AB)2 + (2x)2 

(AE)2 = (AB)2 + 4x2    —(2)

In △ABC,

(AC)2 = (AB)2 + (BC)2 

(AC)2 = (AB)2 + (3x)2 

(AC)2 = (AB)2 + 9x2 —(3)

Eq(1) multiply by 4, 

4AD2 = 4AB2 + 4x2   —(4)

Eq(4) – (2), 

4AD2 AE2 = 3AB2  —(5)

Eq(1) multiply by 9, 

9AD2 = 9AB2 + 9x2   —(6)

Eq(6) – (3),

9AD2 AC2 = 8AB2  —(7) 

Eq(5) multiply by 8 and Eq(7) multiply by 3, 

32AD2 − 8AE2 = 24AB2 —(8)

27AD2 − 3AC2 = 24AB2 —(9)

From eq(8) and (9), 

32AD2 − 8AE2 = 27AD2 − 3AC2 

8AE2 = 5AD2 + 3AC2 

Q. 52 When a positive integer is divided by d, the remainder is 15. When ten times of the same number is divided by d. the remainder is 6. The least possible value of d is: 

A.

B. 16 

C. 18 

D. 12 

Answer: B. 

Explanation: 

When a positive integer is divided by d, the remainder is 15 and When ten times of the same number is divided by d. the remainder is 6. So, 

15 will be also ten times so, 

Remainder = 150 

By the option B) 

When 150 is divided by 16 then remainder will be 6. 

Hence, correct answer is option B) 16. 

Q. 53 Reema sold 48 articles for ₹ 2,160 and suffered a loss of 10%. How many articles should she sell for ₹2,016 to earn a profit of 12%? 

A. 36 

B. 40 

C. 28 

D. 32 

Answer: A. 

Explanation: 

Selling price of 48 articles when loss of 10% = 2160 

Cost price of 48 articles = 2160 / 90 ×100 = 2400 

Cost price of 1 article = 2400/48 = 50 

Selling price of when profit of 12% = 2016 

Cost price = 2016 / 112 ×100 = 1800 

Number of articles = 1800/50 = 36 

Q. 54 A and B together borrowed a sum of ₹51,750 at an interest rate of 7%p.a. compound interest in such a way that to settle the loan, A paid as much amount after three years as paid by B after 4 years from the day of borrowing. The sum (in ₹) borrowed by B was: 

A. 25,650 

B. 26,750 

C. 25,000 

D. 24,860 

Answer: C. 

Explanation: 

let A borrow be Rs.x. 

Borrowed by B = 51,750 – x 

compound interest rate(r) = 7% 

Amount paid by A after 3 year = amount paid by B after 4 year 

Amount = principal(1 + r/100)t

x(1 + 7/100)3 = (51, 750 − x)(1 + 7/100)4 

x = (51, 750 − x) × 107/100 

100x = 5537250 – 107x 

x = 5537250/207 = 26750 

Borrowed by B = 51,750 – x = 51,750 – 26750 = 25000 

Q. 55 is equal to 

A. (x−4)/(x+4)

B. (x+4)/x(4−x)

C. (x+4)/x(x−4)

D. (x+4)/(x−4)

Answer: C. 

Explanation: 

Q. 56 The marked price of an item is 25% above its cost price. A shopkeeper sells it, allowing a discount of x % on the marked price. If he incurs a loss of 8%, then the value of x is 

A. 25.6% 

B. 26.8% 

C. 26.4% 

D. 25.2% 

Answer: C. 

Explanation: 

Let the cost price of an article be 100. 

Marked price = 100 × 125/100 = 125 

Discount = x% 

Loss = 8% 

Selling price = 100 × 92/100 = 92 

Selling price = marked price – discount 

Discount = 125 – 92 = 33 

x% = 33/125×100 = 26.4% 

Q. 57 What x is added to each of 10, 16, 22 and 32, the numbers so obtained in this order are in proportion? What is the mean proportional between the numbers (x + 1) and (3x + 1)? 

A. 12 

B.

C. 15 

D. 10 

Answer: C. 

Explanation: 

If x is added to each of numbers, the numbers so obtained in this order are in proportion 

so, (10 + x)/(16 + x) = (22 + x)/(32 + x) 

(10 + x)(32 + x) = (16 + x)(22 + x) 

320 + 10x + 32x + x2 = 352 + 16x + 22x + x2 

320 + 42x = 352 + 38x 

4x = 32 

x = 8 

(x + 1) = 8 + 1 = 9 

(3x + 1) = 24 + 1 = 25 

Mean proportional = √(9×25) = 15 

Q. 58 What is the value of  

A.

B.

C.

D. -1 

Answer: B. 

Explanation: 

Q. 59 The average of some numbers is 54.6. If 75% of the numbers are increased by 5.6 each, and the rest are decreased by 8.4 each, then what is the average of the numbers so obtained? 

A. 55.6 

B. 55.8 

C. 56.7 

D. 56.3 

Answer: C. 

Explanation: 

New average of the numbers = (average + increment/decrements) 

The average of some numbers = 54.6 

75% of the numbers are increased by 5.6 each, and the rest are decreased by 8.4 each so, 

New average of the numbers = 54.6 + (5.6 × 75/100 – 8.4 × 25/100) = 54.6 + 4.2 – 2.1 = 56.7 

Q. 60 Diameter AB of a circle with centre O is produced to a point P such that PO = 16.8 cm. PQR is a secant which intersects the circle at Q and R such that PQ = 12 cm and PR = 19.2 cm.The length of AB (in cm) is : 

A. 14.2 

B. 15.2 

C. 15.8 

D. 14.4 

Answer: D. 

Explanation: 

Q. 61 A boat can go 3.6 km upstream and 5.4 km downstream in 54 minutes, while it can go 5.4 km upstream and 3.6 km downstream in 58.5 minutes. The time (in minutes) taken by the boat in going 10 km downstream is: 

A. 48 

B. 50 

C. 45 

D. 54 

Answer: B. 

Explanation: 

Let the speed of speed of stream be u and speed of boat in still water be v. 

Speed of boat in upstream = u – v 

Speed of boat in downstream = u + v 

A boat can go 3.6 km upstream and 5.4 km downstream in 54 minutes so, 

Time = distance/speed 

3.6/(u – v) + 5.4/(u + v) = 54/60 

3.6/(u – v) + 5.4/(u + v) = 9/10 

36/(u – v) + 54/(u + v) = 9

4(u + v) + 6(u – v) = u2 – v2 

10u – 2v = u2 – v2 —(1) 

Boat can go 5.4 km upstream and 3.6 km downstream in 58.5 minutes so, 

5.4/(u – v) + 3.6/(u + v) = 58.5/60 

54/(u – v) + 36/(u + v) = 585/60 

6/(u – v) + 4/(u + v) = 13/12

72(u + v) + 48(u – v) = 13(u2 – v2

120u + 24v = 13(u2 – v2

Put the value of eq(1), 

120u + 24v = 13 × (10u – 2v) 

120u + 24v = 130u – 26v 

10u = 50v 

u = 5v —(2) 

put the value of u in eq(1), 

50v – 2v = 25v2 – v2 

48v = 24v2 

v = 2 

put the value of v in eq(2), 

u = 5 × 2 = 10 

Speed of boat in downstream = u + v = 10 + 2 = 12 

The time taken by the boat in going 10 km downstream = distance/speed = 10/12 hours = 60  × 10/12 = 50 min 

Q. 62 If 5cosθ – 12sinθ = 0, then what is the value of (1 + sinθ + cosθ)/(1-sinθ + cosθ

A. 5/4 

B. 3/2 

C. 3/4 

D. 5/2 

Answer: B. 

Explanation: 

5cosθ – 12sinθ = 0

tan θ =5/12 

We know that tanθ = perpendicular/base 

so, 

By the triplet 5-12-13, 

Hypotenuse = 13 

sin  θ = 5/13 

cos  θ= 12/13 

(1 + sin  θ + cos  θ)/(1 – sin  θ + cos θ

= (1 + 5/13 + 12/13)/(1 – 5/13 + 12/13)

= 30/20=3/2

Q. 63 The given table represents the sale (in thousands) of cars by four companies A, B, C and D in six years. Study the table and answer the question that follows. 

The total number of cars sold by companies A in 2017 and C in 2013 is what percentage of the total number of cars sold by all four companies in 2013 and 2016? (correct to one decimal place) 

A. 23.8 

B. 24.2 

C. 25.6 

D. 23.3 

Answer: D. 

Explanation: 

The total number of cars sold by companies A in 2017 and C in 2013 = 52 + 65 = 117 

The total number of cars sold by all four companies in 2013 = 45 + 63 + 65 + 67 = 240 

The total number of cars sold by all four companies in 2016 = 72 + 58 + 70 + 63 = 263 

The total number of cars sold by all four companies in 2013 and 2016 = 240 + 263 = 503 

Required percentage = 117/503 × 100 = 23.26% ~ 23.3% 

Q. 64 If 11sin2θ – cos2θ + 4sin θ – 4 = 0, 00<   θ < 900, then what is the value of (cos 2θ + cot 2θ) / (sec 2θ – tan 2θ)

A. (12+7√3)/6 

B. (12+5√3)/3 

C. (10+5√3)/3 

D. (10+7√3)/6 

Answer: A. 

Explanation: 

11sin2θ – cos2θ + 4sinθ – 4 = 0 

11sin2θ – (1-sin2θ) + 4sinθ – 4 = 0 

12sin2θ + 4sinθ – 5 = 0 

12sin2θ + 10sinθ –6sinθ – 5 = 0 

2sinθ (6sinθ+5)−1(6sinθ+5)= 0

(2sinθ −1) (6sinθ+5) = 0

For  00<   θ < 900

sinθ= ½

θ = 300

(cos 2θ + cot 2θ) / (sec 2θ – tan 2θ

On putting the value of θ

Q. 65 If the radius of a right circular cylinder is decreased by 10%, and the height is increased by 20%, then the percentage increase/decrease in its volume is: 

A. increase by 2.8% 

B. decrease by 1.8% 

C. increase by 1.8% 

D. decrease quadrilateral by in 2.8% 

Answer: D. 

Explanation: 

Volume of right circular cylinder = π r2

Radius of a right circular cylinder is decreased by 10%, and the height is increased by 20% so, 

r1 = r × 90/100 = 0.9r 

h1 = h × 120/100 = 1.2h 

Volume of new right circular cylinder =π r12h1 = π (0.9r)2(1.2h) = 0.972 π r2

Decrements in volume = π r2h – 0.972π r2h = 0.028π r2

Percentage Decrements in volume = (0.028π r2h)/(π r2h) × 100 = 2.8% 

Q. 66 ABCD is a cyclic quadrilateral which sides AD and BC are produced to meet at P, and sides DC and AB meet at Q when produced. If ∠A = 600 and ∠ABC = 720,then ∠PDC – ∠DPC = ? 

A. 240

B. 300 

C. 360 

D. 400 

Answer: A. 

Explanation: 

In ΔABP, 

∠A + ∠ABC + ∠APB = 1800 

∠APB = 1800 – 600 – 720 = 480 

∠ADC = 1800 – ABC = 1800 – 720 = 1080 

∠PDC = 1800 – ∠ADC = 1800 – 1080 = 720 

∠PDC – ∠DPC = 72 – 48 = 240 

Q. 67 The given table represents the sale (in thousands) of cars by four companies A, B, C and D in six years. Study the table and answer the question that follows. 

The Total number of cars sold by company C in 2018 exceeds the average number of cars sold by company A during 2014 to 2018 by: 

A. 15,000 

B. 14,000 

C. 16,000 

D. 12,000 

Answer: C. 

Explanation: 

Total number of cars sold by company C. in 2018 = 76000 

Total number of cars sold by company A during 2014 to 2018 = 52000 + 61000 + 72000 + 52000 + 63000 = 300000 

Average number of cars sold by company A during 2014 to 2018 = 300000/5 = 60000 

Required number = 76000 – 60000 = 16000 

Q. 68 A and B spend 60% and 75% of their incomes, respectively. If the savings of A are 20% more than that of B then by what percentage is the income of A less than the income of B? 

A. 15 

B. 20 

C. 10 

D. 25 

Answer: D. 

Explanation: 

Let the income of A. and B. be X and Y respectively. 

Saving of A = X – 60/100 × X = 0.4X 

Saving of B = Y – 75/100 × Y = 0.25Y 

If the savings of A are 20% more than that of B. then, 

Saving of A = 0.25Y × 120/100 = 3Y/10 

0.4X = 3Y/10 

4X = 3Y 

Ratio of income of A. and B. = 3 : 4 

Required percentage = (4 – 3)/4 × 100 = 25% 

Q. 69 The given table represents the sale (in thousands) of cars by four companies A, B, C and in six years. Study the table and answer the question that follows. 

The total number of cars sold by company B. during 2015, 2017 and 2018 is what percentage less than the total number of cars sold by company C. in 2013, 2015, 2017 and 2018? 

A. 16⅔  

B. 33⅓ 

C. 50 

D. 40 

Answer: B. 

Explanation: 

The total number of cars sold by company B during 2015, 2017 and 2018 = 60 + 53 + 67 = 180 

The total number of cars sold by company C in 2013, 2015, 2017 and 2018 = 65 + 66 + 63 + 76 = 270 

Required percentage = (270 – 180)/270 × 100 = 33⅓% 

Q. 70 To complete a certain task, X is 40 % more efficient than Y, and Z is 40% less efficient than Y. Working together, they can complete the task in 21 days. Y and Z together worked for 35 days. The remaining work will be completed by X alone in: 

A. 8 days 

B. 4 days 

C. 6 days 

D. 5 days 

Answer: D. 

Explanation: 

Efficiency of X = 140% of Y = 1.4Y 

Efficiency of Z = 60% of Y = 0.6Y 

Efficiency of X, Y and Z = 1.4 : 1 : 0.6 = 7 : 5 : 3 

They can complete the task in 21 days so, 

Total work = sum of efficiency ratio × times = (7 + 5 + 3) × 21 = 315 

Y and Z together worked for 35 days so, 

Work done by Y and Z = (5 + 3) × 35 = 280 

Remaining work = 315 – 280 = 35 

Remaining work complete by X = work/efficiency = 35/7 = 5 days 

Q. 71 The given table represents the sale (in thousands) of cars by four companies A, B, C and D in six years. Study the table and answer the question that follows. 

What is the ratio of the total number of cars sold by companies A, B and D in 2017 to the total number of cars sold by all four companies in 2018? 

A. 3:4 

B. 6:13 

C. 9:14 

D. 18:23 

Answer: C. 

Explanation: 

The total number of cars sold by companies A, B and D in 2017 = 52 + 53 + 75 = 180 

The total number of cars sold by all four companies in 2018 = 63 + 67 + 76 + 74 = 280 

The ratio of the total number of cars sold by companies A, B and D in 2017 to the total number of cars sold by all four companies in 2018 = 180 : 280 = 9 : 14 

Q. 72 The value of is: 

A. 4⅓  

B. 5⅚ 

C. 4 5/12 

D. 5 5/12 

Answer: C. 

Explanation: 

=> 4 5/12 

Q. 73 In ΔPQR, PQ = 24 cm and Q = 580 S and T are the points on side PQ and PR, respectively, such that STR = 1220 and If PS = 14 cm and PT = 12 cm, then the length of RT is : 

A. 14.8 cm 

B. 16 cm 

C. 15 cm 

D. 16.4 cm 

Answer: B. 

Explanation: 

∠PTS + ∠ STR = 1800 

∠PTS = 180 – 122 = 580 

∠P is a common angle. 

Δ PQR and ∠PTS are similar triangle. SO, 

PT/PQ = PS/PR 

12/24 = 14/PR 

PR = 28 cm 

RT = PR – PT = 28 – 12 = 16 cm 

Q. 74 If then the value of xy is : 

A.

B. 10 

C.

D.

Answer: A. 

Explanation: 

Q. 75 If 20x2 — 30x + 1 = 0, then what is the value of 25x2+1/16x2 

A. 58¾  

B. 53¾ 

C. 53½   

D. 58½ 

Answer: B. 

Explanation: 

English Comprehension 

Instructions 

For the following questions answer them individually 

Q. 76 Select the correctly spelt word. 

A. Beaureacracy 

B.  Recrutment 

C. Reimbursement 

D. Surveliance 

Answer: C. 

Q. 77 Select the appropriate meaning of the given idiom. 

To take the bull by the horns 

A. To speak arrogantly 

B. To murder someone 

C. To surrender to the enemy 

D. To handle difficulties 

Answer: D. 

Q. 78 Given below are four jumbled sentences. Select the option that gives their correct order. 

A. Although I had a fear of water, I thought it was an important skill I should learn. 

B. However, what I didn’t realize was that it would also make me a more confident person. 

C. I also thought it would be a good exercise that would make me physically stronger. 

D. One of the hardest things I’ve had to do was to learn how to swim. 

A. DCBA 

B. ACDB 

C. DACB 

D. BADC 

Answer: C. 

Q. 79 Select the correctly spelt word 

A. Definition 

B. Competiter 

C. Proliferete 

D. Beneficiery 

Answer: A. 

Q. 80 Select the correct synonym of the given word. 

Erudite 

A. Learned 

B. Strong 

C. Fashionable 

D. Illiterate 

Answer: A. 

Q. 81 Select the passive form of the given sentence. 

They are constructing a residential youth hostel. 

A. A residential youth hostel is constructed by them 

B. A residential youth hostel was constructed by them. 

C. A residential youth hostel has been constructed by them 

D. A residential youth hostel is being constructed by them 

Answer: D. 

Instructions 

In the following passage some words have been deleted. Fill in the blanks with the help of the alternatives given. Select the most appropriate option for each blank. 

Comprehension: 

Drug addiction is the continued use of a particular drug (1)______ harmful consequences. Drug addiction not only affects an individual’s health and relationship, but also (2)______ the society and the environment. Prevention of a particular list of drugs can be possible by (3)______ their sale without a prescription. Drug addicts should be given proper medical treatment and (4)______ facilities. Motivational and awareness camps should be organized to scale down the consumption (5)______ drugs. 

Q. 82 Select the most appropriate option for blank No. 1 

A. despite of 

B. though 

C. in spite 

D. despite 

Answer: D. 

Q. 83 Select the most appropriate option for blank No. 2 

A. .imparts 

B. impacts 

C. reflects 

D. reduces 

Answer: B. 

Q. 84 Select the most appropriate option for blank No. 3 

A. conducting 

B. restricting 

C. performing 

D. promoting 

Answer: B. 

Q. 85 Select the most appropriate option for blank No. 4 

A. regeneration 

B. resignation 

C. rehabilitation 

D. regression 

Answer: C. 

Q. 86 Select the most appropriate option for blank No. 5 

A. for 

B. of 

C. from 

D. by 

Answer: B. 

Instructions 

For the following questions answer them individually 

Q. 87 Select the correct antonym of the given word. 

Allure 

A. Revive 

B. Rewind 

C. Repulse 

D. Attract 

Answer: C. 

Q. 88 Select the word which means the same as the groups of words given. 

A. long and aggressive speech 

A. Harangue 

B. Eloquence 

C. Prologue 

D. Discussion 

Answer: A. 

Q. 89 Select the word which means the same as the groups of words given. 

The school or college in which one has been educated 

A. Alma mater 

B. Alumni 

C. Graduate 

D. Mentor 

Answer: A. 

Q. 90 Given below are four jumbled sentences. Select the option that gives their correct order. 

A. During the celebration, many people display a set of ornamental dolls known as ‘Hina’ dolls. 

B. Hinamatsuri is a celebration of the girls in Japan wishing them a bright future. 

C. These royal-looking dolls are dressed in the clothing of the Heian period. 

D. These dolls represent the Emperor, Empress and other royal representatives. 

A. CDBA 

B. ACBD 

C. DBAC 

D. BADC 

Answer: D. 

Q. 91 In the sentence identify the segment which contains the grammatical error. 

The old man did not wanted to eat any food. 

A. to eat 

B. did not wanted 

C. any food 

D. The old man 

Answer: B. 

Q. 92 Select the most appropriate word to fill in the blank. 

All students should express their ______ views in the group discussion without any fear 

A. outspoken 

B. guarded 

C. candid 

D. secluded 

Answer: C. 

Q. 93 Select the most appropriate word to fill in the blank. 

All his endeavors to win his teacher’s favour proved ______ and did not bring the desired results. 

A. perpetual 

B. futile 

C. apparent 

D. prosperous 

Answer: B. 

Q. 94 In the sentence identify the segment which contains the grammatical error. 

No least than fifty participants were present for the singing competition. 

A. No least than 

B. the singing competition 

C. fifty participants wer 

D. present for 

Answer: A. 

Q. 95 Select the correct antonym of the given word. 

Bizarre 

A. Strange 

B. Usual 

C. Happy 

D. Weird 

Answer: B. 

Q. 96 Select the most appropriate option to substitute the underlined segment in the given sentence. If no substitution is required, select No improvement. 

Many of a students were not shortlisted for the personal interview 

A. No improvement 

B. Many of students 

C. The many student 

D. Many of the students 

Answer: D. 

Q. 97 Select the appropriate meaning of the given idiom. 

To flog a dead horse 

A. To waste the efforts 

B. To accept the challenge 

C. To complete the work 

D. To make the best use of resources 

Answer: A. 

Q. 98 Select the most appropriate option to substitute the underlined segment in the given sentence. If no substitution is required, select No improvement. 

When I was working in a software company, it was mandatory to register my legally as an authorized software developer. 

A. register me legal 

B. register myself legally 

C. register mine legally 

D. No improvement 

Answer: B. 

Q. 99 Select the correct indirect form of the given sentence. 

“Hello”, he said to his friend. “What can I do for you?”” 

A. He said hello and asked his friend what can do for him 

B. He asked and greeted his friend that what he can do for him 

C. He greeted his friend and asked what he could do for him 

D. He told hello and asked his friend what he could do for him 

Answer: C. 

Q. 100 Select the correct synonym of the given word. 

Violent 

A. Kind 

B. Mild 

C. Calm 

D. Aggressive 

Answer: D. 

SSC CGL Tier-I 05 March 2020 Shift-I Previous Year Paper

SSC CGL Tier-I Exam 05 March 2020

Q. 1 Select the option that is related to the third number in the same way as the second number is related to the first number and the sixth number is related to the fifth number. 

72 : 108 :: 84 : ? :: 102 : 153 

A. 144 

B. 126 

C. 117 

D. 135 

Answer: B. 

Explanation: 

72 + 72/2 = 72 + 36 = 108 

102 + 102/2 = 102 + 51 = 153 

Similarly, 

84 + 84/2 = 84 + 42 = 126 

∴ The correct answer is option B. 

 

Q. 2 Shaan has a total of ₹5,500 with him. He buys product ‘Z’ at ₹5,000 from this sum and then sells it to another person, thus making a profit of 15% onit. With all the money he has now, he buys product ‘X’ and then sells it to another person making a profit of 25% onit. What is the total money Shaan has now? 

A. ₹7,812.50 

B. ₹7,815.50 

C. ₹6,325.50 

D. ₹7,187.50 

Answer: A. 

Explanation: 

Cost price of product ‘Z’ = 5000 

Profit = 15% 

Selling price = 5000×115/100= 5750 

Cost price of product ‘X’ = 5750 + 500 = 6250 

Profit = 25% 

Selling price = 6250×125/100 = 7812.5 

Shaan has rs. 7812.5. 

∴ The correct answer is option A. 

 

Q. 3 Select the option figure in which the given figure is embedded (rotation is NOT allowed).

Answer: B. 

Q. 4 Select the letter-cluster that can replace the question mark (?) in the following series. aYd, fTi, kOn, pJs, ? 

A. VeX 

B. uEw 

C. uFw 

D. uEx 

Answer: D. 

Explanation: 

so, next letter-cluster is ‘uJx’. 

∴ The correct answer is option D. 

Q. 5 Four Letter-clusters have been given, out of which three are alike in some manner, while one is different. Select the odd letter-cluster. 

A. TVW 

B. FHJ 

C. LNP 

D. DFH 

Answer: A. 

Explanation: 

Except option A. remaining all options have the difference of 1 in each letter. 

So, ‘TVW’ is odd. 

∴ The correct answer is option A. 

Q. 6 Select the option in which the words share the same relationship as that shared by the given pair of words. 

Blunder : Error 

A. Euphoria : Happiness 

B. War : Peace 

C. Speak : Hear 

D. Anger : Rage 

Answer: A. 

Explanation: 

Blunder is similar to error. 

Similarly, 

Euphoria is similar to happiness. 

∴ The correct answer is option A. 

Q. 7 Arrange the following in a logical sequence from small to big. 

1. Crocodile 

2. Lizard 

3. Whale 

4. Housefly 

5. Monkey 

A. 4, 3, 2, 1, 5 

B. 3, 5, 4, 1, 2 

C. 4, 2, 5, 1, 3 

D. 4, 5, 2, 1, 3 

Answer: C. 

Explanation: 

Sequence from small to big, 

Housefly, Lizard, Monkey, Crocodile, Whale 

∴ The correct answer is option C. 

Q. 8 Four words have been given, out of which three are alike in some manner, while one is different. Select the odd word. 

A. Plenty 

B. Indigence 

C. Destitution 

D. Penury 

Answer: A. 

Explanation: 

Except the ‘Plenty’ remaining all have similar meaning. 

∴ The correct answer is option A. 

Q. 9 Select the option in which the numbers are related in the same way as are the numbers in the given set. 

(13, 65, 117) 

A. (15, 75, 135) 

B. (12, 55, 109) 

C. (14, 70, 127) 

D. (17, 85, 163) 

Answer: A. 

Explanation: 

13×5 = 65 

13×9 = 117 

Similarly, 

15×5 = 75 

15×9 = 135 

∴ The correct answer is option A. 

Q. 10 Four positions of the same dice are shown. Select the number that will be on the face opposite to the one showing ‘3’.

A.

B.

C.

D.

Answer: B. 

Q. 11 Read the given statements and conclusions carefully. Assuming that the information given in the statements is true, even if it appears to be at variance with commonly known facts, decide which of the given conclusions logically follow from the statements. 

Statements: 

1. All parakeets are cuckoos. 

2. All cuckoos are rabbits. 

3. All rabbits are snakes. 

Conclusions: 

I. All parakeets are snakes. 

II. All snakes are cuckoos. 

III. All rabbits are parakeets. 

IV. All cuckoos are snakes. 

A. Only conclusions II and III follow. 

B. Only conclusions I and II follow. 

C. Only conclusions I and IV follow. 

D. All the conclusions follow. 

Answer: C. 

Explanation: 

From the Venn diagram, 

Only conclusions I and IV follow. 

∴ The correct answer is option C. 

Q. 12 Four number-pairs have been given, out of which three are alike in some manner and one is different. Select the number-pair that is different. 

A. 169-197 

B. 121-145 

C. 289-325 

D. 225-241 

Answer: D. 

Explanation: 

In 169 – 197, 

  (13)2 = 169 

(14)2 + 1 = 196 + 1 = 197

In 121-145, 

(11)2 = 121 

(12)2 + 1 = 144 + 1 = 145 

In 289-325, 

(17)2 = 289 

(18)2 + 1 = 324 + 1 = 325 

In 225-241 

(15)2  = 225 

(16)2 − 15 = 256 – 15 = 241 

∴The correct answer is option D. 

Q. 13 If each letter of the English alphabet is assigned an odd numerical value in increasing order, such as A = 1, B = 3 and so on, then what will be the code of HONEY? 

A. 132725747 

B. 152927949 

C. 132725745 

D. 152927947 

Answer: B. 

Explanation: 

H = 8 + 7 = 15 

O = 15 + 14 = 29 

N = 14 + 13 = 27 

E = 4 + 5 = 9 

Y = 25 + 24 = 49 

So, Code for ‘HONEY” = 152927949 

∴ The correct answer is option B. 

Q. 14 Select the number that can replace the question mark (?) in the following series. 

40, 37, 43, 34, 46, ? 

A. 31 

B. 41 

C. 51 

D. 61 

Answer: A. 

Explanation: 

The series follows the pattern as, 

40 – 3 = 37 

37 + 6 = 43 

43 – 9 = 34 

34 + 12 = 46 

46 – 15 = 31 

∴ The correct answer is option A. 

Q. 15 Which of the option figures is the exact mirror image of the given figure when the mirror is held at the right side?

A.

B.

C.

D.

Answer: D. 

Q. 16 Select the option that is related to the third word in the same way as the second word is related to the first word. Ministers : Council :: Sailors : ? 

A. Sea 

B. Ship 

C. Captain 

D. Crew 

Answer: D. 

Explanation: 

As ministers is related to council similarly, 

Sailors are related to crew. 

∴ The correct answer is option D. 

Q. 17 The sequence of folding a piece of paper and the manner in which the folded paper has been cut is shown in the following figures. How would this paper look when unfolded? 

Answer: B. 

Q. 18 

Study the given pattern carefully and select the number that can replace the question mark (?) in it. 

A.

B.

C.

D.

Answer: C. 

Explanation: 

(10×14) ÷ 35 = 140/35 = 4 

(15 ×5) ÷ 25= 75/25 = 3 

(14 ×6) ÷ 12 = 84/12 = 7 

(18 ×8) ÷ 16= 144/16 = 9 

∴ The correct answer is option C. 

Q. 19 How many triangles are present in the given figure? 

A. 22 

B. 20 

C. 23 

D. 21 

Answer: C. 

Q. 20 There is a family of five members: K, L, M,N and O. Among them,there is one married couple. O is unmarried and is the brother of K. is the sister of O. M is the only married female and the mother of N. L and O are the only males in the group. Who is the father of K? 

A.

B.

C.

D.

Answer: B. 

Explanation: 

In the diagram, circle shows the female, the square shows the male, vertical line son the generation, horizontal line shows the brother/sister and double horizontal lines show the couple. 

From the diagram, L is the father of K. 

∴ The correct answer is option B. 

Q. 21 Select the figure that can replace the question mark (?) in the following series. 

Answer: C. 

Q. 22 In the given Venn diagram,the triangle represents students playing table tennis, the rectangle represents students playing badminton, the circle represents female students, and the pentagon represents students playing football. The numbers given in the diagram represent the number of persons in that particular category. 

How many female students play both table tennis and badminton? 

A. 22 

B. 18 

C.

D.

Answer: C. 

Q. 23 The two given expressions on both the side of the ‘=’ sign will have the same value if two numbers from either side or both side are interchanged. Select the correct numbers to be interchanged from the given options. 

3 + 5 × 4 − 24 ÷ 3 = 7 × 4 − 3 + 36 ÷ 6 

A. 6, 3 

B. 5, 7 

C. 4, 7 

D. 24, 36 

Answer: B. 

Explanation: 

3 + 5 × 4 − 24 ÷ 3 = 7 × 4 − 3 + 36 ÷ 6 

From option B, 

On interchanging 5 and 7, 

3 + 7 × 4 − 24 ÷ 3 = 5 × 4 − 3 + 36 ÷ 6 

3 + 28 − 8 = 20 − 3 + 6 

23 = 23 

∴ The correct answer is option B. 

Q. 24 Select the set of letters that when sequentially placed in the blanks of the given letter series will complete the series.

f_hg_fh_gf_hg_fh_g 

A. f, g, h, f, g, h 

B. g, h, f, g, h, f 

C. g, f, g, f, h, f 

D. h, f, g, h, f, g 

Answer: B. 

Explanation: 

Sequence, 

fghghf/hfgfgh/ghfhfg 

∴ The correct answer is option B. 

Q. 25 In certain code languages. U is written as C, K is written as H, L is written as U, N is written as E, S is written as L, E is written as K, and C. is written as N. How will ‘KNUCKLES’ be written as in that language? 

A. KECNKUHL 

B. CHUECKN 

C. HECNHUKL 

D. HECNHULK 

Answer: C. 

Explanation: 

‘KNUCKLES’ is written as ‘HECNHUKL’. 

∴ The correct answer is option C. 

General Awareness 

Instructions 

For the following questions answer them individually 

Q. 26 Pongal festival is celebrated for four days in Tamil Nadu. What is the fourth day of Pongal called? 

A. Thai Pongal 

B. Kaanum Pongal 

C. Bhogi Pongal 

D. Mattu Pongal 

Answer: B. 

Q. 27 Name the law in Physics which states that equal volume of all gases under the same conditions of temperature and pressure contain equal number of molecules. 

A. Avogadro’s Law 

B. Boyles’s Law 

C. Charles’s Law 

D. Ohm’s Law 

Answer: A. 

Q. 28 Which of the following rivers flows through Tiruttani a famous pilgrimage place of South India? 

A. Nandi 

B. Kaveri 

C. Palar 

D. Vaigai 

Answer: A. 

Q. 29 In which year was the Nahargarh Fort in Jaipur built by Maharaja Sawai Jai Singh II ? 

A. 1800 

B. 1734 

C. 1805 

D. 1780 

Answer: B. 

Q. 30 As per the government rules, how much percentage of advance tax needs to be paid by 15th June by an individual who is liable to pay advance tax? 

A. 10% 

B. 25% 

C. 15% 

D. 30% 

Answer: C. 

Q. 31 Which dynasty built the pancha rathas of Mahabalipuram? 

A. Satavahana 

B. Pallava 

C. Chola 

D. Chera 

Answer: B. 

Q. 32 In which of the following locations was the Quit India Movement launched by Mahatma Gandhi in 1942? 

A. August Kranti Maidan 

B. Shivaji Park 

C. Pragati Maidan 

D. Jallianwala Bagh 

Answer: A. 

Q. 33 What was the theme of the 107th Indian Science Congress held in Bengaluru? 

A. Science and Technology: Rural Development 

B. Reaching the Unreached through Science and Technology 

C. Future India : Science and Technology 

D. Science and Technology for National Development 

Answer: A. 

Q. 34 The researchers of which academic institution employed the nanoscale phenomenon called ‘Electrokinetic streaming potential’ to harvest energy from flowing water on a small scale like water flowing through household water taps? 

A. IIT Guwahati 

B. IIT Delhi 

C. IIT Bombay 

D. IIT Madras 

Answer: A. 

Q. 35 In January 2020, B Sai Deepak set a Guinness World Record for most side lunges in 60 seconds. How many lunges did he do? 

A. 50 

B. 40 

C. 30 

D. 59 

Answer: D. 

Q. 36 Which National Park among the following is the largest protected area in the Eastern Himalayan sub-region? 

A. Jim Corbett National Park 

B. Namdapha National Park 

C. Keibul Lamjao National Park 

D. Bandipur National Park 

Answer: B. 

Q. 37 Chiropody is a branch of science related to which part of the body? 

A. Liver 

B. Kidney 

C. Feet 

D. Lungs 

Answer: C. 

Q. 38 G. Babita Rayudu took charge as an Executive Director for which of the following organisations in January 2020? 

A. The Securities and Exchange Board of India 

B. Insurance Regulatory and Development Authority of India 

C. Small Industries Development Bank of India 

D. Bombay Stock Exchange 

Answer: A. 

Q. 39 Which is the first Indian company to hit the ₹10 lakh crore mark in market capitalisation? 

A. HDFC. Bank 

B. ICICI Bank 

C. Tata Consultancy Services 

D. Reliance Industries 

Answer: D. 

Q. 40 The 23rd National Youth Festival (NYF) 2020 was celebrated in Lucknow to commemorate the birth anniversary of ______. 

A. Jawaharlal Nehru 

B. Sardar Vallabhbhai Patel 

C. Swami Vivekananda 

D. Mahatma Gandhi 

Answer: C. 

Q. 41 The Indian Railways has integrated its helpline numbers into a single number. What is the number? 

A. 139 

B. 145 

C. 150 

D. 160 

Answer: A. 

Q. 42 The police of which state was honoured with the President’s Colours award in December 2019? 

A. Maharashtra 

B. Kerala 

C. Tamil Nadu 

D. Gujarat 

Answer: D. 

Q. 43 In which year was the Currency Building in the BBD. Bagh or Dalhousie area of Kolkata constructed? 

A. 1833 

B. 1910 

C. 1850 

D. 1900 

Answer: A. 

Q. 44 In terms of area, which state has the largest forest cover in India? 

A. Maharashtra 

B. Odisha 

C. Madhya Pradesh 

D. Kerala 

Answer: C. 

Q. 45 VISHWAS, which is a major e-governance initiative launched by the government in January 2020, is the acronym for which of the following? 

A. Video Interface and State Wide Advanced Security 

B. Video Integration and System Wide Advanced Security 

C. Video Integration and State Wide Advanced Security 

D. Video Integration and State Wide Advanced System 

Answer: C. 

Q. 46 What is the colour of the light emitted by the Sun? 

A. Red 

B. White 

C. Orange 

D. Yellow 

Answer: B. 

Q. 47 The famous 11-day long ‘Dhanu Jatra’, considered as the largest open-air theatre of the world is celebrated in which state? 

A. Meghalaya 

B. Assam 

C. Odisha 

D. Manipur 

Answer: C. 

Q. 48 Which district has been awarded the Plastic Waste Management Award – 2020 for being the best district of India in the plastic waste management category during Swachhta Hi Seva 2019? 

A. Hojai 

B. Majuli 

C. Dibrugarh 

D. Jorhat 

Answer: C. 

Q. 49 Jasprit Bumrah has been selected to receive which of the following awards for his performance in international cricket in the 2018- 19 season? 

A. C.K. Nayudu 

B. Polly Umrigar 

C. M.A. Chidambaram 

D. Madhavrao Scindia 

Answer: B. 

Q. 50 Ishwar Sharma has been honoured with the Global Child Prodigy Award 2020. What is this award associated with? 

A. Sports 

B. Yoga 

C. Science 

D. Literature 

Answer: B. 

Quantitative Aptitude 

Instructions 

For the following questions answer them individually 

Q. 51 In an examination in which the full marks were 500, A scored 25% more marks than B, B scored 60% more marks than C and C scored 20% less marks than D. If A scored 80% marks, then the percentage of marks obtained by is: 

A. 65% 

B. 60% 

C. 50% 

D. 54% 

Answer: C. 

Q. 52 The given table represents the exports (in ₹crores) of four items A, B, C and D over a period of six years. Study the table and answer the question that follows. 

In which year, the exports of item D were 1.4 times the average exports of item B during six years? 

A. 2013 

B. 2012 

C. 2011 

D. 2014 

Answer: A. 

Explanation: 

Average exports of item B. during six years ={128 + 134 + 138 + 169 + 182 + 209}/6 

(Average = sum of the terms/no. of terms) 

= 960/6 = 160 

1.4 times the average exports of item B. during six years = 1.4×160 = 224 

In 2013, the exports of item D were 1.4 times the average exports of item B during six years. 

Q. 53 If x2 − 2√5x + 1 = 0, then what is the value of x5 + 1/x5

A. 610√5

B. 406√5

C. 408√5

D. 612√5

Answer: A. 

Explanation: 

x2 − 2√5x + 1 = 0 Divide by x, 

x − 2√5 + 1/x = 0 

x + 1/x = 2√5  —(1)

(x +1/x)2  = (2√5)2

x2 + (1/x)2 + 2 = 20

x2 + (1/x)2 = 18 —-(2)

From eq(1), 

(x +1/x)3  = (2√5)3

x3 + (1/x)3 + 3(x +1/x) = 40√5

x3 + (1/x)3 = 40√5 – 3(2√5)

x3 + (1/x)3 = 34√5

From eq(2) and (3), 

{x2 + (1/x)2}{x3 + (1/x)3}=(18) (34√5)

x5 + 1/x + x 1/x5 = 612√5

x5 + 1/x5 = 612√5 – 2√5 = 610√5

Q. 54 Sudha sold an article to Renu for ₹576 at a loss of 20%. Renu spent a sum of ₹224 on its transportation and sold it to Raghu at a price which would have given Sudha profit of 24%. The percentage of gain for Renu is: 

A. 13.2% 

B. 10.5% 

C. 12.9% 

D. 11.6% 

Answer: D. 

Explanation: 

Cost price for Sudha = 576/80 ×100 = 720 

Cost price for Renu = 576 

Final cost price for Renu = 576 + 224 = 800 

Selling price for Renu = 24% profit of sudha 

= 720 × 124/100 = 892.8 

Profit for Renu = 892.8 -800 = 92.8 

The percentage of gain for Renu = 92.8 / 800 ×100 = 11.6% 

Q. 55 If the nine-digit number 708x6y8z9 is divisible by 99, then what is the value of x + y + z? 

A.

B. 27 

C. 16 

D.

Answer: C. 

Explanation: 

To be divisible by 99, the number has to be divisible by 11 and 9 both. 

For divisibility by 11, 

7 + 8 + 6 + 8 + 9 – 0 + x + y + z 

(38 – x + y + z) has to be divisible by 11. 

For divisibility by 9, 

(38 + x + y + z) has to be divisible by 9. 

By option C), 

x + y + z = 16 

(38 – x + y + z) = 38 – 16 = 22 is divisible by 11. 

(38 + x + y + z) = 38 + 16 = 54 is divisible by 9. 

Q. 56 The ratio of the ages ofA. and B, 8 years ago, was 2 : 3. Four years ago,the ratio of their ages was 5 : 7. What will be the ratio of their ages 8 years from now? 

A. 4 : 5 

B. 5 : 6 

C. 7 : 8 

D. 3 : 4 

Answer: A. 

Explanation: 

8 years ago the ratio of the ages of A and B = 2 : 3 

Let the 8 years ago ages of A and B be 2x and 3x respectively. 

4 years age of A = 2x + 4 

4 years age of B = 3x + 4 

Four years ago,the ratio of their ages = 5 : 7 

(2x+4)/(3x+4) = 5/7

14x + 28 = 15x + 20 

x = 8 

8 years from now, Age of A = 2x + 16 = 2×8 + 16 = 32 

8 years from now, Age of B = 3x + 16 = 3×8 + 16 = 40 

Ratio of their ages 8 years from now = 32 : 40 = 4 : 5 

Instructions 

The given table represents the exports (in ₹crores) of four items A, B, C and D over a period of six years. Study the table and answer the question that follows. 

Q. 57 The total exports of item A. from 2012 to 2014 is what percentage less than the total exports of all the four items in 2015? (Correct to one decimal place) 

A. 16.7% 

B. 15.2% 

C. 14.3% 

D. 13.8% 

Answer: C. 

Explanation: 

The total exports of item A. from 2012 to 2014 = 225 + 370 + 425 = 1020 

The total exports of all the four items in 2015 = 400 + 209 + 306 + 275 = 1190 

Required percentage =(1190−1020)/ 1190×100 =170/ 1190×100 

= 14.28%≈14.3% 

Q. 58 What is the ratio of the total exports of item A in 2014 and 2015 to the total exports of item C in 2011 and 2015? 

A. 7 : 5 

B. 3 : 2 

C. 4 : 3 

D. 5 : 4 

Answer: B. 

Explanation: 

Total exports of item A. in 2014 and 2015 = 425 + 400 = 825 

Total exports of item C. in 2011 and 2015 = 244 + 306 = 550 

Ratio of the total exports of item A. in 2014 and 2015 to the total exports of item C. in 2011 and 2015 = 825 : 550 = 3 : 2 

Instructions 

For the following questions answer them individually 

Q. 59 The average of 24 numbers is 56. The average of the first 10 numbers is 71.7 and that of the next 11 numbers is 42. The next three ½ :⅓ :5/12 numbers (i.e. 22nd , 23nd, and 24th ) are in the ratio . What is the average of the numbers 22nd and  24th

A. 58 

B. 49.5 

C. 55 

D. 60.5 

Answer: D. 

Explanation: 

The average of 24 numbers = 56 

Sum of the numbers = 56×24 = 1344 

The average of the first 10 numbers = 71.7 

Sum of the first 10 numbers = 71.7×10 = 717 

The average of the next 11 numbers = 42 

Sum of the next 11 numbers = 42×11 = 462 

Ratio of the next three numbers (i.e.22nd , 23nd, and 24th ) = ½ :⅓ :5/12

Sum of next three numbers (i.e., and ) = 1344 – 717 – 462 = 165 

Q. 60 If , then what is the value of (P ÷ Q) × R

A. 2(x2 + y2)

B. x2 + y2

C. 4xy

D. 2xy 

Answer: A. 

Explanation: 

(P ÷ Q) ×

Q. 61 A shopkeeper bought 80 kg of rice at a discount of 10%. Besides 1 kg rice was offered free to him on the purchase of every 20 kg rice. If he sells the rice at the marked price, his profit percentage will be: 

A. 16⅔%

B. 15⅓ % 

C. 153/7 %

D. 142/7 %

Answer: A. 

Explanation: 

Let the Price of 1 kg rice be Rs.1. 

Total rice bought = 80 kg 

He offered 1 kg rice free on the purchase of every 20 kg rice. Free rice = 80/20 = 4 kg 

Rate of 80 kg rice = Rs.80 

Discount = 10% 

Cost price of rice for shopkeeper = 80×90/100= Rs.72 Selling price of rice for shopkeeper = 80 + 4 = Rs.84 Profit = 84 – 72 = Rs. 12 

 Profit percentage = 12/72×100 =16⅔  % 

Q. 62 D is the mid point of side BC of ABC. Point E lies on AC such that CE =⅓AC. BE and AD. intersect at G. What is AG/GD

A. 5 : 2 

B. 8 : 3 

C. 3 : 1 

D. 4 : 1 

Answer: D. 

Explanation: 

D is mid point of BC. 

To apply the mid poingt theorem in ADM, 

Q. 63 Two chords AB and CD of a circle with centre O intersect each other at P. If BOC = 70and AOD = 100, then APC is: 

A. 95

B. 70

C. 65

D. 55

Answer: A. 

Explanation: 

Q. 64 A train takes hours less for a journey of 300 km, if its speed is increased by 20 km/h from its usual speed. How much time will it take to cover a distance of 192 km at its usual speed? 

A. 3 hours 

B. 2.4 hours 

C. 4.8 hours 

D. 6 hours 

Answer: C. 

Explanation: 

Let the usual speed of the train be x km/hr. 

Distance = 300 km 

Time = 2½  hours = 5/2 hr = 2.5 hr 

Time = distance/speed 

According to question, 

300/x – 300(x + 20) = 2.5 

(x + 20)×120 – 120x = x(x + 20) 

120x + 2400 – 120x = x2 + 20x 

x2 + 20x – 2400 = 0 

x2 + 60x – 40x – 2400 = 0 

x(x + 60) – 40(x + 60) = 0 

(x + 60)(x- 40) = 0

x = 40 

Distance = 192 km 

Time taken to cover distance by usually speed = 192/40 = 4.8 hours 

Q. 65 If 12cos2θ−2sin2θ+3cosθ=3, 0<θ<90, then what is the value of (cosecθ + secθ)/(tanθ + cotθ)? 

A. (4 +√3)/4 

B. (1 +2√2)/2 

C. (1 +√3)/2 

D. (2 +√3)/4 

Answer: C. 

Explanation: 

12cos2θ−2sin2θ+3cosθ=3 

12cos2θ−2(1-cos2θ)+3cosθ=3 

14cos2θ+3cosθ=5

Put the value of θ = 60

14cos2 60+3cos 60=5 

14×1/2 + 3 ×1/2 = 5 

5 = 5 

L.H.S. = R.H.S. 

Q. 66 If 16a4 + 36a2b2 + 81b4 = 91 and 4a2 + 9b2 – 6ab = 13, then what is the value of 3ab? 

A. 3/2 

B. -3 

C. 3/2 

D.

Answer: C. 

Explanation: 

4a2 + 9b2 – 6ab = 13 

(4a2 + 9b2 – 6ab)2 = (13)2 

[(a + b + c)2 = a2 + b2 + c2 + 2(ab + bc + ac) ]

(4a2)2 + (9b2)2 + (6ab)2 +2(4a2.9b2 – 9b2.6ab – 6ab.4a2) = 169 

16a4 + 36a2b2 + 81b^4 + 2(36a2b2 – 54ab3 – 24a3b) = 169

91 + 2(36a2b2 – 54ab3 – 24a3b) = 169 

36a2b2 – 54ab3 – 24a2b = (169 – 91)/2 

36a2b2 – 54ab3 – 24a3b = 39 

6ab(6ab – 9b2 – 4a2) = 39

6ab(-13) = 39

6ab = -3 

3ab = -3/2 

Q. 67 In ΔABC, C = 90, AC= 5 cm and BC = 12 cm. The bisector of A meets BC at D. What is the length of AD? 

A. 2/3√13 cm

B. 4/3√13 cm 

C. 2√13 cm 

D. (5√13)/3 cm 

Answer: D. 

Explanation: 

By the Pythagoras theorem, 

(AB)2 = (AC)2 + (BC)2 

(AB)2 = (5)2 + (12)2 

(AB)2 = (5)2 + (12)2 

(AB)2 = 25 + 144 

AB = 13 cm 

By angle bisector theorem, 

(AB/BD) = AC/CD

Let CD be x cm. 

13/(12 – x) = 5/x 

13x = 60 – 5x 

x = 60/18 = 10/3 

In ΔACD, 

(AD)2 = (AC)2 + (CD)2 

(AD)2 = (5)2 + (10/3)2 

(AD)2 = 25 +100/9 

(AD)2 = 325/9

AD = 5√13/3 

Q. 68 The value of is: 

A. 3/4 

B. 2/3 

C. 1/2 

D.

Answer: B. 

Explanation: 

Q. 69 Sides AB and DC of cyclic quadrilateral ABCD are produced to meet at E, and sides AD and BC are produced to meet at F. If BAD = 102 and BEC = 38 then the difference between ADC and AFB is: 

A. 21 

B. 31

C. 22 

D. 23 

Answer: C. 

Explanation: 

In ΔADE, 

∠ADE=180  − (∠AED + ∠EAD) 

= 180 − (38 + 102

= 40 

⇒∠ADC = 40 

square ABCD. is a cyclic quadrilateral. 

∴∠DCB + ∠DAB=180 

⇒∠DCB = 180 − ∠DAB 

∠DCB = 180 − 102 

∠DCB = 78

In ΔDFC, 

∠DFC=180 – (∠FDC+∠FCD) 

∠DFC = 180 − (40 + 78

∠DFC = 180 − 118 

∠DFC = 62 

∠AFB = ∠DFC = 62 

Difference between ∠BAD and ∠AFB = 62 – 40 = 22 

Q. 70 If 5sinθ = 4, then the value of (secθ+4cotθ)/(4tanθ – 5cosθ) is: 

A. 3/2 

B.

C. 5/4 

D.

Answer: D. 

Explanation: 

5sinθ = 4 

sinθ = 4/5 

Perpendicular / hypotenuses = 4/5

By triplet 3-4-5, 

Base = 3 

cosθ = base/hypotenuses = 3/5

tanθ = perpendicular/base = 4/3 

(secθ + 4cotθ)/(4tanθ – 5cosθ) 

= (1/cosθ + 4/tanθ)/(4tanθ – 5cosθ) 

= [1/(3/5) + 4/(4/3)]/[4(4/3) – 5(3/5)]

= [(5/3)+5) / {4×(4/3) – 5×(3/5)}] 

= [(14/3) / {(16/3) – 3)}] 

= 14/7 = 2 

Q. 71 The diagonal of a square A is (a + b) units. What is the area (in square units) of the square drawn on the diagonal of square B whose area is twice the area of A? 

A. (a + b)2 

B. 4(a + b)2 

C. 8(a + b)2 

D. 2(a + b)2 

Answer: D. 

Explanation: 

Area of square A = (diagonal)2/2 = (a + b)2/2

Area of square B = 2 × area of square A = 2 × (a + b)2 /2 = (a + b)2

Side of B = a + b 

Diagonal of B = √(2side) = √2(a + b) 

Area (in square units) of the square drawn on the diagonal of square B = (side)2 = {√2(a+b)}2 = 2(a + b)2

Q. 72 The given table represents the exports (in ₹crores) of four items A, B, C and D over a period of six years. Study the table and answer the question that follows. 

The total exports of item D in 2010, 2012 and 2014 is what percentage of the total exports of all the four items in 2011 and 2012? 

A. 44.8% 

B. 44% 

C. 45% 

D. 46.2% 

Answer: B. 

Explanation: 

The total exports of item D. in 2010, 2012 and 2014 = 214 + 247 + 309 = 770 

The total exports of all the four items in 2011 and 2012 = 250 + 134 + 244 + 282 + 225 + 138 + 230 + 247 = 1750 

Required percentage = (770 / 1750) × 100 = 44% 

Q. 73 Pipes A and B can fill a tank in 10 hours and 40 hours, respectively. C is an outlet pipe attached to the tank. If all the three pipes are opened simultaneously, it takes 80 minutes more time than what A and B. together take to fill the tank. A and B are kept open for 7 hours and then closed and C was opened. C will now empty the tank in: 

A. 49 hours 

B. 38.5 hours 

C. 42 hours 

D. 45.5 hours 

Answer: A. 

Explanation: 

Let the total work be 40 units. 

(∵ L.C.M. of 10 and 40 is 40.) 

Efficiency of A = work/time = 40/10 = 4 units/hour 

Efficiency of B = 40/40 = 1 unit/hour 

Time time taken by pipe A and B = 40/(4 + 1) = 8 hours 

Time time taken by pipe C = 8 hours + 80/60 hours = 28/3 hours 

Efficiency of C = 40/(28/3) = 30/7 units/hour 

Work done by pipe A and B in 7 hours = (1 + 4) × 7 = 35 units 

Time taken by pipe C to empty the tank = 35/(30/7) = 8 5/7 

Q. 74 The compound interest on a certain sum at 16⅔% p.a. for 3 years is ₹6,350. What will be the simple interest on the same sum at the same rate for 5⅔ years? 

A. ₹10,200 

B. ₹11,400 

C. ₹7,620 

D. ₹9,600 

Answer: A. 

Explanation: 

Compound interest = 6350 

Rate(r) = 16⅔ %= (50/3)% 

Time(t) = 3 years 

Q. 75 The value of is: 

A. 2 6/7 

B. 2 2/9  

C. 3 4/7  

D. 10/21 

Answer: A. 

Explanation: 

English Comprehension 

Instructions 

For the following questions answer them individually 

Q. 76 In the sentence identify the segment which contains the grammatical error. 

Modern man is completely engross in the mad pursuit of material pleasures and luxuries. 

A. mad pursuit of 

B. material pleasures and luxuries 

C. Modern man is 

D. completely engross 

Answer: D. 

Q. 77 Select the correct passive form of the given sentence. 

They offered me a chair 

A. I offered a chair to them. 

B. A. chair was being offered to me. 

C. A. chair is offered to me by them. 

D. I was offered a chair by them. 

Answer: D. 

Q. 78 Select the most appropriate meaning of the given idiom 

A. bed of roses 

A. A pleasant perfume 

B. An easy and happy situation 

C. A difficult path 

D. A valley full of flowers 

Answer: B. 

Q. 79 Select the correct indirect form of the given sentence. 

“What a good idea!”, Seema remarked. 

A. Seema exclaimed that the idea is good. 

B. Seema exclaimed that it was a very good idea. 

C. Seema said what a good idea it is. 

D. Seema told what an idea! 

Answer: B. 

Q. 80 Select the most appropriate word to fill in the blank. 

I like both tea and coffee but prefer the ______. 

A. last 

B. later 

C. latter 

D. least 

Answer: C. 

Q. 81 Select the most appropriate meaning of the given idiom 

A close-fisted person 

A. A cruel person 

B. A kind person 

C. A strong person 

D. A miserly person 

Answer: D. 

Instructions 

In the following passage some words have been deleted. Fill in the blanks with the help of the alternatives given. Select the most appropriate option for each blank. 

Comprehension: 

A. (1)______ of trucks carrying soldiers was coming down the mountain road. The trucks (2) ______ slowly as there had been heavy snowfall in that area . Suddenly, with a (3) ______ a huge tree on the hill side fell bringing along with it boulders and mud. (4) ______, the driver of first truck stopped in time. The soldiers got down and started (5) ______ the road. 

Q. 82 Select the most appropriate option for blank no. 1 

A. bevy 

B. crew 

C. convoy 

D. flock 

Answer: C. 

Q. 83 Select the most appropriate option for blank no. 2 

A. are moving 

B. were moving 

C. was moving 

D. has moved 

Answer: B. 

Q. 84 Select the most appropriate option for blank no. 3 

A. crash 

B. buzz 

C. scream 

D. splash 

Answer: A. 

Q. 85 Select the most appropriate option for blank no. 4 

A. Logically 

B. Magically 

C. Fortunately 

D. Similarly 

Answer: C. 

Q. 86 Select the most appropriate option for blank no. 5 

A. altering 

B. clearing 

C. moving 

D. changing 

Answer: B. 

Instructions 

For the following questions answer them individually 

Q. 87 Select one word for the following group of words. 

One who loves his country 

A. Traitor 

B. Conspirator 

C. Patriot 

D. Collaborator 

Answer: C. 

Q. 88 Select synonym of the given word. 

RETAIN 

A. Convey 

B. Maintain 

C. Destory 

D. Gain 

Answer: B. 

Q. 89 Given below are four jumbled sentences. Out of the given options pick the one that gives their correct order. 

A. Aesop was one of them who lived in Greece about 2500 years ago. 

B. He told many interesting stories to the people. 

C. There were many talented people in ancient Greece. 

D. Although he was ugly, he had a very clever brain. 

A. CDBA 

B. BDAC 

C. BADC 

D. CADB 

Answer: D. 

Q. 90 Select the most appropriate segment to substitute the underlined segment of the given sentence. If no substitution is required, select ‘No substitution’. 

Hardly had he sit on the chair than it broke. 

A. No substitution 

B. sat onto a chair then 

C. sit in the chair when 

D. sat on the chair when 

Answer: D. 

Q. 91 Select the most appropriate word to fill in the blank. 

He tried to ______ my ring. 

A. steal 

B. still 

C. stile 

D. steel 

Answer: A. 

Q. 92 Select the most appropriate word to substitute the underlined word of the given sentence. If no substitution is required, select ‘No improvement’. 

The diver dive in the pool from a great height. 

A. dived into the pool 

B. dived at the pool 

C. dives to a pool 

D. No improvement 

Answer: A. 

Q. 93 Select antonym of the given word. 

DEXTERITY 

A. Mastery 

B. Skill 

C. Ignorance 

D. Agility 

Answer: C. 

Q. 94 Select the wrongly spelt word. 

A. Chouffer 

B. Champion 

C. Charisma 

D. Choir 

Answer: A. 

Q. 95 Select the word which means the same as the group of words given. 

Incapable of paying debts 

A. Extravagant 

B. Obsolete 

C. Corrupt 

D. Insolvent 

Answer: D. 

Q. 96 Select antonym of the given word. 

DIVIDE 

A. Unite 

B. Break 

C. Split 

D. Engulf 

Answer: A. 

Q. 97 In the sentence identify the segment which contains the grammatical error. 

My brother, who live in Delhi, has written me a letter. 

A. My brother 

B. me a letter 

C. has written 

D. who live in Delhi 

Answer: D. 

Q. 98 Select synonym of the given word. 

EXPENSIVE 

A. Gentle 

B. Dear 

C. Mild 

D. Sober 

Answer: B. 

Q. 99 Select the wrongly spelt word. 

A. Cremator 

B. Cracker 

C. Creater 

D. Cricketer 

Answer: C. 

Q. 100 Given below are four jumbled sentences. Out of the given options pick the one that gives their correct order. 

A. He is a gifted volleyball player. 

B. But now a days he does not play international matches. 

C. It is because he had an accident last year. 

D. Sanjay is my best friend. 

A. CDBA 

B. DABC 

C. ABCD 

D. DCAB 

Answer: B. 

SSC CPO 16 March 2019 Shift-III Previous Year Paper

SSC CPO 16th-March-2019-Shift-3 

Reasoning 

Instructions For the following questions answer them individually 

Q. 1 Three of the following four word-pairs are alike in a certain way and one is different. Find the odd one out. 

A. Carpenter : Saw 

B. Warrior : Sword 

C. Author : Pen 

D. Mason : Knife 

Answer: D

Q. 2 ‘Cardiology’ is related to ‘Heart’ in the same way as ‘Pathology’ is related to ‘……….’. 

A. Diseases 

B. Man 

C. Body 

D. Soil 

Answer: A. 

Q. 3 In a code language, 54831 is coded as 63922. What will be the code for 86274 in that language? 

A. 75365 

B. 77185 

C. 95365 

D. 95185 

Answer: C. 

Q. 4 Three of the following four options are similar in a certain way and one is different. Find the one that is different. 

A. 6 – 36 – 216 

B. 10 – 100 – 1000 

C. 3 – 6 – 27 

D. 5 – 25 – 125 

Answer: C. 

Q. 5 Select the option that is related to the third term in the same way as the second term is related to the first term. RY : QZ :: HK : 

A. GJ 

B. GL 

C. IL 

D. IM 

Answer: B. 

Q. 6 Select the Venn diagram that best illustrates the relationship between the following three classes.

Hospital, Doctor, Patient 

Answer: D. 

Q. 7 Three of the following four numbers are alike in a certain way and one is different. Identify the different one. 

A. 215 

B. 169 

C. 144 

D. 196 

Answer: A. 

Q. 8 A. Nurse moved 90 m in the East in a hospital to look for her duty Doctor, then she turned right and went 20 m. After this she turned right and after going 30 m she reached I.C.U. but the Doctor was not there. From there she went 100 m to her north and met her doctor. What distance did she moved to meet her duty doctor from the starting point. 

A. 100 m 

B. 120 m 

C. 80 m 

D. 140 m 

Answer: A. 

Q. 9 Three of the following four number-pairs are alike in a certain way and one is different. Identify the different one. 

A. 5 : 29 

B. 8 : 62 

C. 3 : 7 

D. 6 : 34 

Answer: A. 

Q. 10 In the following diagram, the rectangle represents doctors, the triangle represents players and the circle represents philosophers. The numbers in different segments show the number of persons. 

According to the given diagram, How many players are philosophers but not Doctors? 

A. 17 

B.

C.

D. 12 

Answer: A. 

Q. 11 Which number will come next in the following series? 1, 3, 7, 15, 31, 63, ……. 

A. 139 

B. 127 

C. 125 

D. 126 

Answer: B. 

Q. 12 Given is a set of three figures, X, Y and Z, showing the folding sequence of a piece of paper. Fig Z shows the manner in which the folded paper has been cut. Study these figures carefully and select the option that would most closely resemble this piece of paper when unfolded. 

Answer: B. 

Q. 13 Select the correct mirror image of the given figure when the mirror is placed at the right side. 

Answer: D. 

Q. 14 Two statements are given, followed by four conclusions I, II, III and IV. Assuming these statements to be true, even if they seem to be at variance with commonly known facts, decide which of the given conclusions logically follow(s) from the statements. Statements: Some boxes are dolls. All dolls are pens. Conclusions: I. Some boxes are pens. II. Some pens are boxes. III. Some pens are dolls. IV. All pens are dolls. 

A. Only conclusions I, II and IV follow. 

B. All the conclusions follow. 

C. Only conclusions I, II and III follow. 

D. Only conclusions II, III and IV follow. 

Answer: C. 

Q. 15 Select the option that gives the correct explanation for the following group of words. Bridge, Link, Chain 

A. They join two parts. 

B. They are suits of cards. 

C. They are types of medals. 

D. They are jewels. 

Answer: A. 

Q. 16 Identify the number that does NOT belong in the given series. 1, 8, 27, 64, 125, 216, 344, 512, 729 

A. 512 

B. 729 

C. 64 

D. 344 

Answer: D. 

Q. 17 Select the correct alternative to indicate the arrangement of the following words in a logical and meaningful order. 1) Alarm Set 2) Office 3) Car 4) Wake up 5) Alarm Rang 6) Get Ready 

A. 1, 5, 4, 3, 6, 2 

B. 1, 2, 3, 4, 5, 6 

C. 1, 4, 5, 6, 3, 2 

D. 1, 5, 4, 6, 3, 2 

Answer: D. 

Q. 18 Choose the option which has the relation as the words given below : boy : girl 

A. horse – roe 

B. cat – Kitten 

C. cock – hen 

D. donkey – bray 

Answer: C. 

Q. 19 Find the number of squares in the following figure. 

A.

B.

C.

D.

Answer: D. 

Q. 20 Which number will replace the question mark (?) in the following series? 2, 5, 10, 17, 26, 37, ?, 65, 82, 101 

A. 48 

B. 42 

C. 50 

D. 49 

Answer: C. 

Q. 21 If ‘A’ denotes ‘ ÷ ’, ‘B’ denotes ‘ × ’, ‘C’ denotes ‘ + ’ and ‘D’ denotes ‘ − ’, then what will be the value of the following expression? 12 B. 12 A. 4 C. 5 D. 1 

A. 39 

B. 50 

C. 40 

D. 59 

Answer: C. 

Q. 22 Three of the following four letter-clusters are alike in a certain way and one is different. Find the odd one out. 

A. UVwXXYY 

B. BCdEFFF 

C. IJkLLMM 

D. PQrSSTT 

Answer: B. 

Q. 23 Select the option that is related to the third number in the same way as the second number is related to the first number. 25 : 15625 :: 30 : …….. 

A. 2700 

B. 27000 

C. 900 

D. 2500 

Answer: B. 

Q. 24 Two statements are given, followed by two conclusions I and II. Assuming these statements to be true, even if they seem to be at variance with commonly known facts, decide which of the given conclusions logically follow(s) from the statements.

Statements:

All plants are flowers.

No flower is blue.

Conclusions:

I. Some plants are blue.

II. Those plants that are not flowers are blue. 

A. Only conclusion II follows. 

B. Only conclusion I follows. 

C. Neither conclusion I nor II follows. 

D. Both conclusions I and II follow. 

Answer: C. 

Q. 25 Three of the following four words are alike in a certain way and one is different. Find the odd one out. 

A. Cairo 

B. Paris 

C. Ottawa 

D. Kyat 

Answer: D. 

Q. 26 Which letter-cluster will replace the question mark (?) in the following series? XCA, WDZ, UFX, RIU, NMQ,? 

A. IKJ 

B. JKO 

C. JFU 

D. IRL 

Answer: D. 

Q. 27 Three different positions of the same dice are shown below. Which number is on the face opposite the face showing 4? 

A.

B.

C.

D.

Answer: B. 

Q. 28 Select the option in which the given figure is embedded. 

Answer: A. 

Q. 29 Select the option in which the words share the same relationship as that shared by the given set of words (in the same order). Hand : Finger : Ring 

A. Face : Nose : Nose pin 

B. Anklet : Foot : Ankle 

C. Earring : Ear : Jewel 

D. Cap : Head : Hair 

Answer: A. 

Q. 30 In the following number series, two numbers have been put within brackets. Select the most appropriate option for these numbers in relation to their inclusion in the series. 2, 4, 6, 8, 10, (12), 14, 16, (18), 20 

A. The first bracketed number is correct and the second is incorrect. 

B. The first bracketed number is incorrect and the second is correct. 

C. Both the bracketed numbers are incorrect. 

D. Both the bracketed numbers are correct. 

Answer: D. 

Q. 31 Select the option in which the given figure is embedded. (Rotation is not allowed) 

Answer: B. 

Q. 32 In the following number series, two numbers have been put within brackets. Select the most appropriate option for these numbers in relation to their inclusion in the series. 1, 9, 25, 49, (81), 121, 169, (215), 239 

A. Both the bracketed numbers are incorrect. 

B. The first bracketed number is incorrect and the second is correct. 

C. The first bracketed number is correct and the second is incorrect. 

D. Both the bracketed numbers are correct. 

Answer: C. 

Q. 33 Three of the following four letter-clusters are alike in a certain way and one is different. Find the odd one out. 

A. FEDC. 

B. YXVW 

C. LKIJ 

D. NMKL 

Answer: A. 

Explanation: The arrangement of the alphabets are in the alphabetical order in the following manner- 3rd alphabet-4th alphabet-2nd alphabet-1st alphabet 

Option A. violates this arrangement so is the odd one out. 

Option A. is correct. 

Q. 34 Select the option that is related to the third number in the same way as the second number is related to the first number. 13 : 84.5 :: 20 : …… 

A. 120 

B. 150 

C. 100 

D. 200 

Answer: D. 

Explanation: 

13 : 1322= 84.5 

20 : 2022 = 200 

Option D. is correct.

Q. 35 If TEMPLE is coded as ELPMET, then how will CHURCH be coded as? 

A. HCURCH 

B. HCRUHC. 

C. HCRUCH 

D. HCURHC. 

Answer: B. 

Explanation: The alphabets in TEMPLE are coded in opposite i.e ELPMET 

Therefore CHURCH is coded as HCRUHC. 

Option B. is correct 

Q. 36 Three of the following four number-pairs are alike in a certain way and one is different. Identify the different one. 

A. 64 : 6 

B. 7 : 49 

C. 10 : 100 

D. 4 : 16 

Answer: A. 

Explanation: The second number is the square of the first number, for ex 42 = 16

Option A. violates that. 

Option A. is correct 

Q. 37 Select the Venn diagram that best illustrates the relationship between the following three classes. Stationery, Pencil, Apple 

Answer: B. 

Explanation: Pencil is a stationery item and apple is not.Therefore the following diagram explains it. 

Option B. is correct. 

Q. 38 Which of the following interchanges of signs and numbers would make the given equation correct? 

18 − 3 ÷ 6 + 24 × 12 = 48 

A. × and − , 3 and 6 

B. ÷ and − , 12 and 6 

C. × and + , 3 and 6 

D. ÷ and × , 3 and 12 

Answer: A. 

Explanation: Using Option A, we put them in the equation , 18 − 3 ÷ 6 + 24 × 12 = 48

18 × 6 ÷ 3 + 24 − 12 = 48

So option A. satisfies the equation. 

Option A. is correct. 

Q. 39 If ‘ARUN’ is coded as 54, then how will ‘VARUN’ be coded as? 

A. 76 

B. 78 

C. 67 

D. 87 

Answer: A. 

Explanation: We add the corresponding numerical values of the alphabets as per the alphabetical series. 

A. = 1, R = 18, U=21, N = 14, V=22 

ARUN = 1+18+21+14 = 54 

VARUN = 22+1+18+21+14 = 76 

Option A. is correct. 

Q. 40 Select the figure that will come next in the following series. 

Answer: B. 

Explanation: Option B. is correct. 

Q. 41 In a certain code language, ‘mee muk pic’ means ‘roses are yellow”, ‘nil dic’ means ‘white flowers’ and ‘pic muk dic’ means ‘flowers are fruits’. What is the code for ‘white’ in that language? 

A. muk 

B. nil 

C. dic 

D. pic 

Answer: B. 

Q. 42 Select the option that will come next in the given series. 

Answer: A. 

Explanation: The image is getting rotated by 90 degrees in the counterclockwise direction. 

Option A. is correct. 

Q. 43 Select the option that is related to the third term in the same way as the second term is related to the first term. Apple : Fruit :: Spinach : …….. 

A. Food 

B. Flower 

C. Root 

D. Vegetable 

Answer: D. 

Explanation: Apple is a fruit and Spinach is a vegetable. 

Apple : Fruit :: Spinach : Vegetable 

Option D. is correct. 

Q. 44 Select the figure that will come next in the following series. 

Answer: A. 

Explanation: The image is getting rotated by 90 degrees and also 1 parallel line is being added. 

Option A. is correct. 

Q. 45 Select the option that is related to the third number in the same way as the second number is related to the first number. 4 : 20 :: 8 : ….. 

A. 64 

B. 40 

C. 56 

D. 32 

Answer: B. 

Explanation: 4 : 20 :: 8 : 40 

4 × 5 = 20 and 8 × 5 = 40 

Option B. is correct. 

Q. 46 Select the correct mirror image of the given figure when the mirror is placed on the right of the figure . 

Answer: C. 

Explanation: 

Option C. is correct. 

Q. 47 Which of the following two signs need to be interchanged to make the given equation correct? 

6 ÷ 5 − 5 × 5 + 6 = 35 

A. + and − 

B. + and ÷ 

C. × and − 

D. ÷ and × 

Answer: D. 

Explanation: Using option D. we get, 

6 × 5 − 5 ÷ 5 + 6 = 35 

Option D. is correct. 

Q. 48 Rama walks from his university campus 4 km towards north and then turns to the right to reach his department. Alter walking 2.5 km from the department he turns to the left and walks 4 km and reached Conference hall. Now in which direction is he from the starting place? 

A. South-West 

B. North-East 

C. South 

D. West 

Answer: B. 

Explanation: Rama walks from his university campus 4 km towards north and then turns to the right to reach his department. Alter walking 2.5 km from the department he turns to the left and walks 4 km and reached Conference hall. 

So he is in the north east direction from the starting place. (The dashed line shows the direction) 

Option B. is correct. 

Q. 49 Which letter will come next in the following series? Z, U, Q, N, L, …… 

A.

B.

C.

D.

Answer: D. 

Explanation: The alphabets are in reverse alphabetical order in the following pattern : 

Z → (−5)U → (−4)Q → (−3)N → (−2)L → (−1)K 

Option D. is correct. 

Q. 50 Select the correct alternative to indicate the arrangement of the following words in a logical and meaningful order. 1. Gateway of India 2. World 3. Mumbai 4. India 5. Maharashtra 

A. 4, 2, 5, 1, 3 

B. 2, 4, 5, 1, 3 

C. 2, 4, 5, 3, 1 

D. 4, 2, 5, 3, 1 

Answer: C. 

Explanation: The chronological order for the following is as follows : (as per geography) 

2. World > 4. India > 5. Maharashtra > 3. Mumbai > 1. Gateway of India 

Option C. is correct. 

 

General knowledge 

Instructions For the following questions answer them individually 

Q. 51 Citrullus lanatus is the botanical name for …….. 

A. Apple 

B. Watermelon 

C. Banana 

D. Orange 

Answer: B. 

Q. 52 …… founded the ‘Bal Charkha Sangh’, where children were taught to spin & weave. 

A. Vinoba Bhave 

B. Sarojini Naidu 

C. Indira Gandhi 

D. Mahatma Gandhi 

Answer: C. 

Q. 53 The 11th President of India, ……… was also known as the ‘Missile Man’. 

A. K R Narayanan 

B. Zakir Hussein 

C. APJ Abdul Kalam 

D. Fakhruddin Ali Ahmed 

Answer: C. 

Q. 54 The Hindi film ‘Kai Po Che’ is based on the book titled ……. by Chetan Bhagat. 

A. Two States 

B. One Night At The Call Centre 

C. Three Mistakes Of My Life 

D. Five Point Someone 

Answer: C. 

Q. 55 Who was the Chairperson of the NITI Aayog as of January 2019? 

A. Narendra Modi 

B. Sushma Swaraj 

C. Amit Shah 

D. Ram Nath Kovind 

Answer: A. 

Q. 56 Who is the founder of the biotech Indian company called ‘Biocon’? 

A. Ratan Tata 

B. Indu Jain 

C. Kiran Mazumdar Shaw 

D. Mukesh Ambani 

Answer: C. 

Q. 57 The fear of foreigners is called: 

A. Toxiphobia 

B. Xenophobia 

C. Sitophobia 

D. Theophobia 

Answer: B. 

Q. 58 Malus pumila is the botanical name of the ………. tree. 

A. Orange 

B. Water melon 

C. Banana 

D. Apple 

Answer: D. 

Q. 59 The Panchayat Raj System was first adopted by the state of ……… 

A. West Bengal 

B. Tamil Nadu 

C. Andhra Pradesh 

D. Rajasthan 

Answer: D. 

Q. 60 The Puma Kumbh Mela takes place every ……… years at one of the four places by rotation, Prayag, Haridwar, Ujjain and Nashik. 

A. six 

B. two 

C. twelve 

D. three 

Answer: C

Q. 61 The festival of ___________ is also known as Phagawa. 

A. Diwali 

B. Holi 

C. Sankranti 

D. Ganesh Chaturthi 

Answer: B. 

Q. 62 Which of the following flowers is botanically known as Narcissus? 

A. Carnation 

B. Rose 

C. Daffodil 

D. Lily 

Answer: C. 

Q. 63 Pung Cholam is the folk dance of which state ? 

A. Madhya Pradesh 

B. Manipur 

C. Kerala 

D. Karnataka 

Answer: B. 

Q. 64 Which of the following is the unit of Magnetic field strength? 

A. Ohm 

B. Gauss 

C. Tesla 

D. Weber 

Answer: C. 

Q. 65 Padma Vibhushan is the …………. civilian award of India. 

A. highest 

B. second highest 

C. third highest 

D. fourth highest 

Answer: B. 

Q. 66 ONGC. discovered the country’s first shale gas reserve at: 

A. Chandikhol 

B. Durgapur 

C. Jamnagar 

D. Padur 

Answer: B. 

Q. 67 ……. was born to the Chauhan King Someshvara and his queen Karpuradevi in 1166 CE. 

A. Prithviraj Chauhan 

B. Maharana Pratap 

C. Maharaja Suraj Mal 

D. Maharaja Man Singh 

Answer: A. 

Q. 68 According to the Vedic principles, which amongst the following is incorrect ? 

A. Vaishyas or the common people were born from man’s legs 

B. Kshatriyas or the warrior-kings were born from the man’s arms 

C. Shudra or the labourer and servant were born from man’s feet 

D. Brahmins or the learned ones were born from man’s ears 

Answer: D. 

Q. 69 Which of the following flowers is botanically known as Dianthus? 

A. Rose 

B. Lily 

C. Daffodil 

D. Carnation 

Answer: D. 

Q. 70 Gir National Park is known for its ………. 

A. Red silk cotton 

B. Orangutan 

C. Cherry blossom 

D. Asiatic Lions 

Answer: D. 

Q. 71 ……… was a professor of political science and economics at the University of Taxila. 

A. Kautilya 

B. Bindusara 

C. Samudragupta 

D. Ajatshatm 

Answer: A. 

Q. 72 Nyctophobia is the fear of: 

A. Birds 

B. Darkness 

C. Light 

D. Animals 

Answer: B. 

Q. 73 ………. is a textile art fiom Gujarat / Rajasthan. 

A. Bandhani 

B. Paithani 

C. Chanderi 

D. Ikat 

Answer: A. 

Q. 74 The festival of …………. is dedicated to Goddess Durga and her nine avatars. 

A. Sankranti 

B. Navratri 

C. Diwali 

D. Holi 

Answer: B. 

Q. 75 In the body, excess of _____ is the reason for arthritis. 

A. Lactic Acid 

B. Acetic Acid 

C. Nitric Acid 

D. Uric Acid 

Answer: D

Q. 76 Which of the following was NOT a part of the ‘Navratna’ at Vikramaditya’s court? 

A. Kshapanaka 

B. Kalidasa 

C. Vararuchi 

D. Surdas 

Answer: D. 

Q. 77 ……….. is situated at the confluence of Alaknanda and Mandakini rivers. 

A. Vishnuprayag 

B. Karnaprayag 

C. Nandaprayag 

D. Rudraprayag 

Answer: D. 

Q. 78 Which of the following clubs, founded in 1889, is the oldest Indian club still in operation as on January 2019? 

A. Indian Bank Recreational Club 

B. Calcutta Cricket & Football Club 

C. Mohun Bagan Athletic Club 

D. North Imphal Sporting Association 

Answer: C. 

Q. 79 The Indian Olympic Association has expressed interest to host the …………. summer Olympics. 

A. 2020 

B. 2024 

C. 2032 

D. 2028 

Answer: C. 

Q. 80 ………. city tops the list of top 10 richest cities in India, with 8 GDP of $ 209 billion. 

A. New Delhi 

B. Kolkata 

C. Bengaluru 

D. Mumbai 

Answer: D. 

Q. 81 The ancient name of the river Tungabhadra was: 

A. Vitasta 

B. Kubha 

C. Pampa 

D. Sindhu 

Answer: C. 

Q. 82 Prince Khurrarn grew up to be known as: 

A. Jahangir 

B. Akbar 

C. Humayun 

D. Shah Jahan 

Answer: D. 

Q. 83 Arundhati Roy won Man Booker Prize in 1997 for her book: 

A. History of Wolves 

B. 2 States 

C. The Minisz of Utmost Happiness 

D. The God of Small Things 

Answer: D. 

Q. 84 The BHIM app was launched by Prime Minister Narendra Modi on: 

A. 31st October, 2016 

B. 26th January, 2016 

C. 30th December, 2016 

D. 15th August, 2016 

Answer: C. 

Q. 85 Who is the only foreigner to receive the Padma Vibhushan Award in 2019? 

A. Anilkumar Manibhai Naik 

B. Teejan Bai 

C. Balwant Moreshwar Purandare 

D. Ismail Omar Guelleh 

Answer: D. 

Q. 86 Natural pearls contain more than 80% ………. 

A. sodium carbonate 

B. calcium carbonate 

C. silica 

D. quartz 

Answer: B. 

Q. 87 Which of the following is the first port in India to become a public company? 

A. Kandla 

B. Mangalore 

C. Ennore 

D. Morrnugao 

Answer: C. 

Q. 88 India was divided into India and Pakistan by the ……….. Act. 

A. India Independence 

B. Division of India 

C. Partition of India 

D. Government of India 

Answer: A. 

Q. 89 ………… is the world’s richest cricket board as of year 2018. 

A. England and Wales Cricket Board 

B. Cricket South Africa 

C. Board of Control for Cricket in India 

D. Pakistan Cricket Board 

Answer: C. 

Q. 90 Tennis player Somdev Kishore Devvarman was awarded the ………. in 2018. 

A. Padma Bhushan 

B. Bharat Ratna 

C. Padma Vibhushan 

D. Padma Shree 

Answer: D. 

Q. 91 Which of the below mentioned human organs can regrow after damage in adults? 

A. Lung 

B. Gall bladder 

C. Kidney 

D. Liver 

Answer: D. 

Q. 92 Which of the following is NOT a mint of the Sectnity Printing and Minting Corporation of India Limited (SPMCIL)? 

A. India Government Mint, Gorakhpur 

B. India Government Mint, Mumbai 

C. India Government Mint, Hyderabad 

D. India Government Mint, Kolkata 

Answer: A. 

Q. 93 …………. was affectionately called ‘Ba’. 

A. Vijayalaxmi Pandit 

B. Indira Gandhi 

C. Kasturba Gandhi 

D. Sarojini Naidu 

Answer: C. 


Q. 94 Eid ul-Fitr is celebrated on the first day of Shawwal, which falls in the ……… month of the Islamic calendar. 

A. tenth 

B. sixth 

C. eighth 

D. twelfth 

Answer: A. 

Q. 95 Name the recipient of the Padma Bhushan Award, 2019 for Trade and Industry (Food Processing). 

A. Mahashay Dharam Pal Gulati 

B. Narsingh Dev Jamwal 

C. Harvinder Singh Phoolka 

D. Vallabhbhai Vasrambhai Marvaniya 

Answer: A. 

Q. 96 Photophobia means: 

A. Addiction to light 

B. Addiction to taking photos 

C. Sensitivity to light 

D. Dislike towards taking photos 

Answer: C. 


Q. 97 Tata Iron And Steel company (TISCO) was incorporated in: 

A. 1911 

B. 1907 

C. 1913 

D. 1915 

Answer: B. 

Q. 98 Vedic civilisation flourished along the river ……….. 

A. Indus 

B. Yamuna 

C. Ganga 

D. Saraswati 

Answer: D. 

Q. 99 The last Nizam of Hyderabad: 

A. Nasir Jung 

B. Qulich Khan 

C. Mir Osman Ali Khan 

D. Muzafar Jung 

Answer: C. 

Q. 100 Bos mutus is the scientific name for: 

A. wild yak 

B. Ox 

C. Horse 

D. Buffalo 

Answer: A. 

 

Quant 

Instructions For the following questions answer them individually 

Q. 101 3 × 7 + 4 − 6 ÷ 3 − 7 + 45 ÷ 5 × 4 + 49 is equal to: 

A. 99 

B. 101 

C. 103 

D. 67 

Answer: B. 


Q. 102 A steel vessel has a base of length 60 cm and breadth 30 cm. Water is poured in the vessel. A cubical steel box having an edge of 30 cm is immersed completely in the vessel, By how much will the water rise? 

A. 12 cm 

B. 9 cm 

C. 10 cm 

D. 15 cm 

Answer: D. 

Q. 103 The given pie-chart, shows the percentage distribution of the expenditure incurred in publishing a book. Study the pie-chart and the answer the questions based on it. 

In the given pie-chart by what percentage the Promotion cost on the book is less than the Paper cost? 

A. 75 

B. 50 

C. 25 

D. 60 

Answer: D. 

Q. 104 If a + b − c = 7,ab − bc − ca = 21 , then a3 + b3 − c3 + 3abc = 

A. -98 

B. 98 

C. 124 

D. 117 

Answer: A. 

Q. 105 If two equal circles whose centres are O and O’ intersect each other at the point A. and B, OO’ = 12 cm and AB. = 16 cm, then radius of the circle is: 

A. 12 cm 

B. 15 cm 

C. 14 cm 

D. 10 cm 

Answer: D. 

Q. 106 A. boy increases his speed to 5/9 times of his original speed. By this he reaches his school 30 minutes before the usual time. How much time does he takes usually? 

A. 67.50 minutes 

B. 67.10 minutes 

C. 67.75 minutes 

D. 67.25 minutes 

Answer: A. 

Q. 107 The given bar chart, shows the sales of books (in thousand number) from six branches of a publishing company during two consecutive years 2000 and 2001. 

In the given bar-chart, the ratio of total sales of all branches (in thousand numbers) for the year 2000 to 2001 is: 

A. 55 : 48 

B. 7 : 11 

C. 45 : 58 

D. 48 : 55 

Answer: D

Q. 108 ₹15,000 was invested by A and B together to start a small business. They got a profit of ₹2,000 at the end of the year. B took his profit share of ₹600. How much did A invest? 

A. ₹10,000 

B. ₹2,000 

C. ₹10,500 

D. ₹9,000 

Answer: C. 

Q. 109 Pipe A. can fill a tank in 16 minutes and pipe B. empties it in 24 minutes. If both the pipes are opened together. after how many minutes should B. be closed, so that the tank is filled in 30 minutes? 

A. 20 minutes 

B. 18 minutes 

C. 21 minutes 

D. 15 minutes 

Answer: C. 

Q. 110 If roots of x2 −  4x + a = 0 are equal, then a =

A. -8 

B.

C. -4 

D.

Answer: B

Q. 111 If a sum becomes ₹1,460 in two years and ₹1,606 in three years due to the compound interest, then annual rate of interest is: 

A. 11% 

B. 9% 

C. 10% 

D. 8% 

Answer: C. 

Q. 112 Fresh fruit contains 68% water and dry fruit contains 20% water. How much dry fruit can be obtained from 100 kg of fresh fruits? 

A. 80 

B. 60 

C. 40 

D. 20 

Answer: C

Q. 113 Side AB of a triangle ABC is 80 cm long, whose perimeter is 170 cm. If angle ABC = 60, the shortest side of triangle ABC measures ……… cm. 

A. 25 

B. 21 

C. 17 

D. 15 

Answer: C. 

Q. 114 (8 + 4 − 2) × (17 − 12) × 10 − 89 is equal to: 

A. 4150 

B. 4120 

C. 413 

D. 411 

Answer: D. 

Explanation: (8 + 4 − 2) × (17 − 12) × 10 − 89 = 10 × 5 × 10 − 89 

= 500 − 89 = 411 

Q. 115 When the integer n is divided by 7, the remainder is 3. What is the remainder if 5n is divided by 7? 

A.

B.

C.

D.

Answer: D. 

Explanation: Dividend = n 

Divisor = 7 

Quotient = Q 

Remainder = 3 

n = 7Q+3 

Multiplying with 5, 

5n = 35Q+15 

Dividing with 7, 

5n7+35Q7+157

Here, When 15 is divided by 7, the remainder will be 1. 

Therefore, 1 will be the remainder when 5n is divided by 7. 

Q. 116 A. earns ₹40 per hour and works for 12 hours. B. earns ₹60 per hour and works for 10 hours. Find the ratio of their per day wages. 

A. 4 : 5 

B. 5 : 4 

C. 15 : 4 

D. 6 : 5 

Answer: A. 

Explanation: Salary of A. per hour = Rs.40 

Number of hours A. work per day = 12 hours 

Then, Total salary A. gets per day = 12 × 40 = Rs.480

Salary of B. per hour = Rs.60 

Number of hours B. work per day = 10 hours 

Then, Total salary B. gets per day = 60 × 10 = Rs.600

Therefore, Ratio of their salaries = 480 : 600 = 4 : 5. 

Q. 117 The perimeter of a square is equal to the perimeter of a rectangle of length 16 cm and breadth 14 cm. Find the circumference of a semicircle whose diameter is equal to the side of the square. 

A. 25.57 cm 

B. 31.57 cm 

C. 23.57 cm 

D. 21.57 cm 

Answer: C. 

Explanation: Let the side of the square be a cm. Perimeter of the square = 4a cm. 

Perimeter of the rectangle = 2(16+14) = 2*30 = 60 cm Given, 4a = 50 => a = 15 cm 

Diameter of semicircle = 15 cm 

Then, Circumference of semicircle = 22/7 × 15/2 + 15 = 23.57 + 15 = 38.57cm

Q. 118 The unequal side of an isosceles triangle is 2 cm. The medians drawn to the equal sides are perpendicular. The area of the triangle is: 

A. 1 cm 

B. 3 cm 

C. 5 cm 

D. 2 cm 

Answer: B. 

Q. 119 The line graph shows the production of product A. and B. (in thousands) during the period 2004 to 2009 and the second line Graph shows the percentage sale of these products. 

In the given line graph. what is the total sale of Product A. in the year 2005 and 2009 taken together? 

A. 17500 

B. 18500 

C. 14600 

D. 16400 

Answer: D. 

Explanation: Production of A. in 2005 = 11000 

Sale of A. in 2005 = 60% of 11000 = 6600 

Production of A. in 2009 = 14000 

Sale of A. in 2009 = 70% of 14000 = 9800 

Therefore, Total production of A. in 2005 and 2009 together = 6600+9800 = 16400. 

Q. 120 When the sun’s angle of depression changes from 30∘ to 60∘ . the length of the shadow of a tower decreases by 70 m. What is the height of the tower? 

A. 45.65 m 

B. 60.55 m 

C. 65.55 m 

D. 36.55 m 

Answer: B. 

Explanation: Let the initial distance of the shadow from the foot of the tower be x m Then, New distance will be (x-70) m 

Tan300 = hx

=13=hx

=x=h3

tan600=hx-70

=3=hx-70

=3=hh3-70

=h=3(h3-70)

=h=3h-703

=2h=703

=h=353=351.73=60.55m

Therefore, The height of the tower = 60.55 m 

Q. 121 If 20% of a = b, then b% of 20 is equal to: 

A. 4% of a 

B. 2% of a 

C. 16% of a 

D. 8% of a 

Answer: A. 

Explanation: Given, 20% of a = b 

Then, b% of 20 = 20% of b = 20% of 20% of a 

= ⅕ × ⅕ × a = 1/25× a 

= 4% of a 

Q. 122 (cosecA. − sinA)2 + (secA. − cosA)2 − (cotA. − tanA)2 is equal to. 

A.

B.

C.

D. -1 

Answer: A. 

Explanation: (cosecA. − sinA)2 + (secA. − cosA)2 − (cotA. − tanA)2 

= cosec2 A. + sin2 A. − 2cosecAsinA. + sec2 A. + cos2 A. − 2secAcosA. − (cot2 A. + tan2 A. − 2cotAtanA) 

= cosec2 A. + sin2 A. − 2 + sec2 A. + cos2 A. − 2 − cot2 A. − tan2 A. + 2 

= (cosec2 A. − cot2 A) + (sin2 A. + cos2 A) + (sec2 A. − tan2 A) − 2 

= 3 − 2 = 1 

Q. 123 0.72×0.72×0.72−0.39×0.39×0.390.72×0.72+0.72×0.39+0.39×0.39is equal to: 

A. 0.33 

B. 0.45 

C. 0.39 

D. 0.36 

Answer: A. 

Explanation: Given fraction is in the form of a3-b3a2+ab+b2 which is a − b 

Here, a = 0.72 and b = 0.39 

Therefore, a − b = 0.72 − 0.39 = 0.33 

Q. 124 A. number is first decreased by 10% and then increased by 10%. The number so obtained is 100 less than the original number. The original number is: 

A. 100000 

B. 100 

C. 10000 

D. 1000 

Answer: C. 

Explanation: Let the number be 100x. 

New number after decrease and increase of 10% = 100x 90100110100=99x

Given, 100x-99x = 100 

x = 100 

Then, The original number = 100x = 100*100 = 10000. 

Q. 125 A. man could not decide between discount of 30% or two successive discounts of 25% and 5%, both given on a shopping of ₹2,000. What is the difference between both the discounts? 

A. ₹15 

B. No difference 

C. ₹20 

D. ₹25 

Answer: D. 

Explanation: Given, Marked Price = Rs.2000 

Discount-1 = 30% of Rs.2000 = Rs.600 

Effective discount with discounts of 25% and 5% = 25-5 + 255/100 = -30 + 1.25 = -25.75%

28.75% of Rs.2000 = Rs.575 

Therefore, Difference between the discounts = Rs.600 – Rs.575 = Rs.25. 

Q. 126 At what rate percent per annum with simple interest will a sum of money double in 12.5 years? 

A.

B. 10 

C. 12.5 

D.

Answer: A. 

Explanation: Let the Principal be Rs.P 

Amount after 12.5 years = Rs.2P 

Interest = Rs.2P – Rs.P = Rs.P Let the rate of interest be R% 

=> PR12.5100=P

R=10012.5=8%

Therefore, Rate of interest = 8%. 

Q. 127 The LCM of two numbers is 168 and their HCF is 12. If the difference between the numbers is 60. What is the sum of the numbers? 

A. 122 

B. 164 

C. 112 

D. 108 

Answer: D. 

Explanation: Let the two numbers be a and b. 

Product of two numbers = Product of their LCM and HCF 

ab = 168*12 = 2016 

Given, a-b = 60 

(a + b)2 = (a − b)2 + 4ab 

(a + b)2 = 602 + 4 × 2016 = 3600 + 8064 = 11664 

Then, a + b = 108 

Therefore, Sum of two numbers = 108. 

Q. 128 If x + x −1 = 2 , then the value of x3 + x−3 is: 

A.

B. ½ 

C.

D.

Answer: D. 

Explanation: 

Given, 1x + x=2

Cubing on both sides 

Q. 129 A. unique circle can always be drawn through x number of given non-collinear points, then x must be: 

A.

B.

C.

D.

Answer: D. 

Explanation: A. unique circle can be drawn using 3 non-collinear points. Hence, x = 3. 

Q. 130 The speed of a car increases by 2 km/hr after every one hour. If the distance travelled in the first one hour was 35 km, what was the total distance travelled in 12 hours? 

A. 558 km 

B. 650 km 

C. 560 km 

D. 552 km 

Answer: D. 

Explanation: The distances travelled by the car in each hour will be 35,37,39,…. The distance travelled in the last hour will be 35/12 + (12 – 1)2 = 35 + 12 = 57 km 

Then, Total distance travelled in 12 hours = 2(35 + 57) = 6 × 92 = 552 km 

Q. 131 Seema bought a mobile at a discount of 20%. Had she received a discount of 25%, she could have saved an additional ₹1000. How much did she pay for the mobile ? 

A. ₹24,000 

B. ₹22,000 

C. ₹25,000 

D. ₹16,000 

Answer: D. 

Explanation: Let the MRP of the mobile be Rs.100x 

Discount = 20% of Rs.100x = Rs.20x 

Selling Price = Rs.100x – Rs.20x = Rs.80x 

If the discount is 25%, then Selling Price = 75% of Rs.100x = Rs.75x 

Given, Rs.80x – Rs.75x = Rs.1000 5x = 1000 

x = 200 

Then, Amount which Seema paid = Rs.80x = Rs.80*200 = Rs.16000. 

Q. 132 The average weight of 16 boys in a class is 60.25 kg and that of the remaining 10 boys is 45.75 kg. The average weight of all boys in the class is: 

A. 54.67 

B. 53.76 

C. 55.37 

D. 56.27 

Answer: A. 

Explanation: Given, Average weight of 16 students = 60.25 kg 

Then, Total weight of 16 students = 60.25*16 = 964 kg 

Average weight of 10 students = 45.75 kg 

Then, Total weight of 10 students = 45.75*10 = 457.5 kg/1421.5 26

Then, Total weight of 26 students = 964+457.5 = 1421.5 kg 

Therefore, Average weight of 26 students = 26 = 54.67kg 

Q. 133 The line graph shows the production of product A. and B. (in thousands) during the period 2004 to 2009 and the second line Graph shows the percentage sale of these products. 

In the given line graph, what is the total sale of Products A. and B. in the year 2007? 

A. 13460 

B. 10290 

C. 11500 

D. 12490 

Answer: B. 

Explanation: Production of A. in 2007 = Rs.7000 

Sale of A. in 2007 = 75% of Rs.7000 = Rs.5250 

Production of B. in 2007 = Rs.9000 

Sale of B. in 2007 = 56% of Rs.9000 = Rs.5040 

Total sale of A. and B. in 2007 = Rs.5250+Rs.5040 = Rs.10290 

Q. 134 A. cube of side 1 m length is cut into small cubes of side 10 cm each. How many such small cubes can be obtained? 

A. 10 

B. 100 

C. 10000 

D. 1000 

Answer: D. 

Explanation: Number of small cubes = Volume of large cube/Volume of each small cube

10313=10001=1000

Q. 135 A. sells a car to B. at 10% loss. If B. sells it for ₹5,40,000 and gains 20%. the cost price of the car for A. was: 

A. ₹5,10,000 

B. ₹5,40,000 

C. ₹5,20,000 

D. ₹5,00,000 

Answer: D. 

Explanation:

Q. 136 If a + b = 8,ab = −12, then a3 + b3

A. -244 

B. -833 

C. 800 

D. 833 

Answer: C. 

Explanation:  Given a + b = 8 ab = −12

Then, a3 + b3 = (a + b)3 − 3ab(a + b)

= 83 − 3(−12)(8) = 512 + 288 = 800

Q. 137 The given pie-chart, shows the percentage distribution of the expenditure incurred in publishing a book. Study the pie-chart and the answer the questions based on it. 

In the given pie-chart, by what percentage Printing and Binding cost on the book is less than the other costs? 

A. 50/3

B. 100/3

C. 47/3

D. 20/3

Answer: B. 

Explanation: Total Printing and Binding cost = 20+20 = 40% of total cost 

Other costs = 10+25+15+10 60% of total cost. 

Therefore, Required percentage = 60 

Q. 138 4/3 tan2 600 + 3cos2 300 – 2sec2 300 – ¾ cot2 600 is equal to: 

A. 8/3 

B. 5/4 

C. 7/3 

D. 10/3

Answer: D. 

Explanation: 4/3 tan2 600 + 3cos2 300 – 2sec2 300 – ¾ cot2 600 

=43(3)2+3(32)2-2(23)2-341(3)2

=433+334-243-3413

=4+94-83-14

=103

Q. 139 The average of 26 numbers is zero. Of them, how many may be greater than zero, at the most? 

A.

B. 25 

C. 20 

D. 15 

Answer: B. 

Explanation: Given that the average of 26 numbers is zero. Then, Sum of 26 numbers = 0 In that, If sum of 25 numbers is 25a, then the 26th number should be -25a. Hence, Maximum 25 numbers can be greater than zero. 

Q. 140 What will be total cost of polishing curved surface of a wooden cylinder at rate of ₹20 per m 2 , if its diameter is 40 m and height is 7 m? 

A. ₹18600 

B. ₹18400 

C. ₹17500 

D. ₹17600 

Answer: D. 

Explanation: Given, Diameter of the cylinder = 40 m Radius of the cylinder = 20 m Height of the cylinder = 7 m Curved

surface area of the cylinder = 2πrh 

= 2 × 22/7× 20 × 7 = 880m2 

For 1 sq.m, Rs.20 will be charged. 

Then, for 880 sq.m, Rs.20*880 = Rs.17600 will be charged. 

Q. 141 A. can do a work in 20 days, while B. can do the same work in 25 days. They started the work jointly. Few days later C. also joined them and thus all of them completed the whole work in 10 days. All of them were paid total of ₹700. What is the share of C? 

A. ₹75 

B. ₹55 

C. ₹70 

D. ₹65 

Answer: C. 

Explanation: Given, A. can do the work in 20 days. B. can do the work in 25 days 

Efficiency of A. = 100/20 = 5% 

Efficiency of B. = 100/25 = 4% 

After a few days, C. joined. 

If they work for 10 days, then (5+4)*10 = 90% of work will be completed by A. and B. 

Remaining 10% of work will be done by C. 

Hence, Salary of C. will be 10% of Rs.700 = Rs.70. 

Instructions: The given bar chart. shows the sales of books (in thousand number) from six branches of a publishing company during two consecutive years 2000 and 2001. 

Q. 142 In the given bar-chart, total sales of branches B1, B3 and B5 taken together for both the years (in thousand numbers) is: 

A. 560 

B. 240 

C. 310 

D. 650 

Answer: A. 

Explanation: Total sales of B1(in thousands) = 105+80 = 185 

Total sales of B3(in thousands) = 95+110 = 205 

Total sales of B5(in thousands) = 75+95 = 170 

Total sales of B1, B3 and B5 together(in thousands) = 185+205+170 = 560 

Q. 143 In the given bar-chart, the ratio of total sales of branches B1, B3 and B5 to total sales of branches B2, B4 and B6 taken together for both the years (in thousand numbers) is: 

A. 45 : 23 

B. 47 : 56 

C. 56 : 47 

D. 23 : 45 

Answer: C. 

Explanation: Total sales of B1(in thousands) = 105+80 = 185 

Total sales of B3(in thousands) = 95+110 = 205 

Total sales of B5(in thousands) = 75+95 = 170 

Total sales of B1, B3 and B5 together(in thousands) = 185+205+170 = 560 

Total sales of B2(in thousands) = 75+65 = 140 

Total sales of B4(in thousands) = 85+95 = 180 

Total sales of B6(in thousands) = 70+80 = 150 

Total sales of B2, B4 and B6 together(in thousands) = 140+180+150 = 470 

Therefore, Ratio = 560 : 470 = 56 : 47. 

Instructions For the following questions answer them individually 

Q. 144 From a point P on a level ground, the angle of elevation of the top of a tower is . If the tower is 270 m high. the distance of point P from the foot of the tower is: 

A. 476.65 m 

B. 367.65 m 

C. 467.65 m 

D. 376.65 m 

Answer: C. 

Explanation: Given, Height of the tower = 270 m 

Let the distance from the foot of the tower to P be x m 

Tan30 = AB/BP

13=270x

x=2703=2701.732=467.65m

Q. 145 The product of two numbers is 6760 and their HCF is 13. How many such pairs of numbers can be formed?   

A.

B.

C.

D.

Answer: B. 

Explanation: Let the two numbers be 13x and 13y since HCF is divisible by two numbers. 

13x × 13y = 6760 

169xy = 6760  

xy = 40 

For HCF to be 13, there should be no common factors except 1. 

Then, (x,y) can be (5,8) and (1,40). 

Therefore, The numbers can be 13*5,13*8 = 65,104 and 13*1,13*40 = 13,520 

Therefore, There can be 2 pairs of such numbers. 

Q. 146 12 buckets of water fill a tank when the capacity of each bucket is 13.5 litres. How many buckets will be needed to fill the same tank. if the capacity of each bucket is 9 litres? 

A. 18 

B. 16 

C. 15 

D. 17 

Answer: A. 

Explanation: Number of buckets1 x Capacity of each bucket1 = Number of buckets2 x Capacity of each bucket2 

12 × 13.5 = 9 × x

=> x = 18

Therefore, Number of buckets required = 18 litres. 

Q. 147 The line graph shows the production of product A. and B. (in thousands) during the period 2004 to 2009 and the second line Graph shows the percentage sale of these products. 

In the given line graph, what is the total sale of Product B. in the year 2004 and 2008 together? 

A. 12500 

B. 14600 

C. 11950 

D. 11825 

Answer: D. 

Explanation: Production of B. in 2004 = 8500 

Sale of B. in 2004 = 55% of 8500 = 4675 

Production of B. in 2008 = 11000 

Sale of B. in 2008 = 65% of 11000 = 7150 

Total sale of B. in 2004 and 2008 together = 4675+7150 = 11825. 

Q. 148 Original breadth of a rectangular box is 20 cm. The box was then remade in such a way that its length increased by 30% but the breadth decreased by 20% and the area increased by 100 cm2. What is the new area of the box? 

A. 2500 cm2

B. 2200 cm2

C. 2400 cm2 

D. 2600 cm2

Answer: D. 

Explanation: Let the length of the rectangle be l cm 

Breadth of the rectangle = 20 cm 

Area of the rectangle = 20l sq.cm 

New length of the rectangle = 130% of l = 1.3l cm 

New breadth of the rectangle = 80% of 20 = 16 cm 

New area of the rectangle = 16*1.3l = 20.8l sq.cm 

Given, 20.8l – 20l = 100 

=> 0.8l = 100 => l = 125 cm 

Therefore, New length of the rectangle = 1.3*125 = 162.5 cm 

New area of the rectangle = 162.5*16 = 2600 sq.cm 

Q. 149 The given pie-chart shows the percentage distribution of the expenditure incurred in publishing a book. Study the pic-chart and answer the questions based on it. 

In the given pie-chart, by what percentage the Royalty on the book is less than the Printing cost? 

A. 20 

B. 25 

C. 10 

D. 15 

Answer: B. 

Explanation: Expenditure of Royalty of book = 20% 

Expenditure of printing = 15% 

Required percentage = 20 – 1520100=520100=25%

Q. 150 3 men, 4 women and 6 children can complete a work in 7 days. A. woman does double the work a man does and a child does half the work a man does. How many women alone can complete this work in 7 days? 

A.

B.

C.

D.

Answer: D. 

Explanation: Efficiency of 2 children = Efficiency of 1 men = Efficiency of ½ women

Efficiency of 1 child = Efficiency of ¼ women

Efficiency of 6 children = Efficiency of 3/2 women

Therefore, Total efficiency of 3 men, 4 women and 6 children = 3/2 + 4 + 3/2 = 3 + 4 = 7 women

Therefore, 7 women are required to complete the work in 7 days.

 

English 

Instructions For the following questions answer them individually 

Q. 151 Select the most appropriate words to fill in the blanks. 

It is known that …………, extreme weather, and diseases are the factors responsible for crop failure. 

A. draught 

B. draft 

C. daft 

D. drought 

Answer: D. 

Instructions: Read the following passage and answer the questions given after it. Passage: 

By the 19205 the improvements in street lighting, domestic lighting and a surge in coffee houses – which were sometimes open all night — was complete. As the night became a place for legitimate activity, the length of time people could dedicate to rest dwindled.” Evening’s Empire puts forward an account of how this happened. “Associations with night before the 17th Century were not good,” it says. “The night was a place populated by people of disrepute – criminals, prostitutes and drunks. Even the wealthy, who could afford candlelight, had better things to spend their money on. There was no prestige or social value associated with staying up all night.” 

That changed in the wake of the Reformation and the counter- Reformation. Protestants and Catholics became accustomed to holding secret services at night. If earlier the night had belonged to reprobates, now respectable people became accustomed to exploiting the hours of darkness. This trend migrated to the social sphere too, but only for those who could afford to live by candlelight. With the advent of street lighting, however, socializing at night began to filter down through the classes. 

In 1667, Paris became the first city in the world to light its streets, using wax candles in glass lamps. It was followed by Lille in the same year and Amsterdam two years later, where a much more efficient oil powered lamp was developed. A. small city like Leipzig in central Germany employed 100 men to tend to 700 lamps. London didn‘t join their ranks until 1684 but by the end of the century, more than 50 of Europe’s major tovms and cities were lit at night. Night became fashionable and spending hours lying in bed was considered a waste of time. 

“People were becoming increasingly time-conscious and sensitive to efficiency, certainly before the 19th Century,” says Roger Ekirch. “But the industrial revolution intensified that attitude by leaps and bounds.” Strong evidence of this shifting attitude is contained in a medical journal from 1829 which urged parents to force their children out of a pattern of first and second sleep. “If no disease or accident there intervene, they will need no further repose than that obtained in their first sleep.” 

Today, most people seem to have adapted quite well to the eight-hour sleep, but Ekirch believes many sleeping problems may have roots in the human body’s natural preference for segmented sleep as well as the ubiquity of artificial light. This could be the root of a condition called sleep maintenance insomnia, where people wake during the night and have trouble getting back to sleep, he suggests. The condition first appears in literature at the end of the 19th Century, at the same time as accounts of segmented sleep disappear. 

Q. 152 Which of the following is NOT true regarding the nights in 19205 in Europe? 

A. Mostly people of disrepute haunted the streets at night. 

B. Nights became a time for legitimate activity 

C. Protestants and Catholics started holding secret services at night. 

D. Socializing at night became common among all classes. 

Answer: A. 

Q. 153 Which of the following phenomena helped classdivision vanish vis-a-vis nightlife? 

A. Counter-reformation 

B. Reformation 

C. Streetlights 

D. Cheaper candles 

Answer: C. 

Q. 154 The author observes, “by the end of the century, more than 50 of Europe’s major towns and cities were lit at night.” Which century was he referring to? 

A. 17th century 

B. 19th century 

C. 16th century 

D. 20th century 

Answer: A. 

Q. 155 In which city were 100 men employed to tend to 700 lamps? 

A. London 

B. Lille 

C. Amsterdam 

D. Leipzig 

Answer: D. 

Q. 156 Which is the most significant cause of sleep maintenance insomnia? 

A. ubiquity of artificial light 

B. waking up during the night 

C. a gowing belief in eight-hour-sleep blocs 

D. ignoring the human body’s natural preference for segmented sleep 

Answer: D. 

Instructions For the following questions answer them individually 

Q. 157 Select the wrongly spelt word. 

A. miniature 

B. municipality 

C. monopoly 

D. manpulates 

Answer: D. 

Q. 158 Identify the segment in the sentence, which contains the grammatical error. 

The problem of world hunger arises because of the economic inequality that distort food distribution. 

A. that distort food distribution. 

B. The problem of 

C. because of the economic inequality 

D. world hunger arises 

Answer: A. 

Q. 159 Identify the segment in the sentence, which contains the grammatical error. Economic growth is sustainable only if all countries has food security. 

A. sustainable only if 

B. Economic growth is 

C. has food security. 

D. all countries 

Answer: C. 

Q. 160 Select the synonym of the given word PLEDGED. 

A. promised 

B. respected 

C. fulfilled 

D. honored 

Answer: A. 

Q. 161 Select the most appropriate meaning of the idiom. Go against the grain. 

A. Something that is ugy and unpopular 

B. Something in conflict with one’s value system 

C. Something that we can accept with dilficulty 

D. Something done deliberately 

Answer: B. 

Q. 162 Select the most appropriate meaning of the given idiom. To beat a dead horse 

A. A. significant effort 

B. A. filtile effort 

C. A. deliberate effort 

D. A. joint effort 

Answer: B. 

Q. 163 Select the most appropriate word for the given group of words. constitutional right to cast vote 

A. licensed 

B. franking 

C. fianchise 

D. duty 

Answer: C. 

Q. 164 Select the antonym of the given word. PROVISIONAL 

A. planned 

B. permanent 

C. insured 

D. supplied 

Answer: B. 

Q. 165 Select the most appropriate word for the given group of words. power of reading thoughts of others 

A. telepathy 

B. medium 

C. astrology 

D. instinct 

Answer: A. 

Q. 166 Select the most appropriate meaning of the given idiom. Bite the bullet 

A. to restrain yourself from doing something unpleasant 

B. to force yourself to do something unpleasant 

C. to amuse yourself by doing something unpleasant 

D. to dislike yourself for having done something unpleasant 

Answer: B. 

Q. 167 Select the most appropriate option to substitute the underlined segment in the given sentence. If there is no need to substitute it, Select No improvement. 

With cloth in demand both at home, where the population was increasing, and abroad, where British colonies were a captive market, improving spinning methods were essential to meet the demand. 

A. improve spinning methods were essential 

B. improved spinning methods were essential 

C. improved spirming methods was essential 

D. No improvement 

Answer: B. 

Q. 168 Select the most appropriate word for the given group of words. a place for the collection of dried plants 

A. museum 

B. aquarium 

C. herbarium 

D. flora 

Answer: C. 

Q. 169 Select the synonym of the given word. BENEFICIARIES 

A. addressers 

B. recipients 

C. profiteers 

D. donors 

Answer: B. 

Q. 170 Select the most appropriate option to substitute the underlined segment in the given sentence. If there is no need to substitute it, Select No improvement. 

With Mumbai’s mad rush and heavy traffic, it becomes extremely difficult for people to reach their destination on time. 

A. reached their destination on time. 

B. to reaching their destination on time. 

C. to reach their destination at time. 

D. No improvement 

Answer: D. 

Q. 171 Select the synonym of the given word PREVENTIVE 

A. prophetic 

B. prepared 

C. predicted 

D. protective 

Answer: D. 

Q. 172 Select the most appropriate word to fill in the blank. During the Great Depression, America’s dance marathon craze was more of a ……… strategy than a form of ……….. 

A. basic; entertainment 

B. fundamental; celebration 

C. survival; entertainment 

D. survival; education 

Answer: C. 

Q. 173 Select the synonym of the given word. NEGLECTED. 

A. superfluous 

B. unknown 

C. ignored 

D. negated 

Answer: C. 

Q. 174 Select the antonym of the given word. PROVINCIAL 

A. pastoral 

B. public 

C. rustic 

D. metropolitan 

Answer: D. 

Q. 175 Select the wrongly spelt word. 

A. fasinated 

B. fashion 

C. facsimile 

D. fastidious 

Answer: A. 

Q. 176 Select the most appropriate option to substitute the underlined segment in the given sentence. If there is no need to substitute it, Select No improvement. 

Eighteenth-century machines typically used water power, hence the early factories was located on the fast-flowing rivers. 

A. were located near the fast-flowing rivers. 

B. was located near the fast-flowing rivers. 

C. were located on the fast-flowing rivers. 

D. No improvement 

Answer: A. 

Q. 177 Select the most appropriate word to fill in the blank. 

Trachoma is a debilitating and painful ……… which can cause the eyelashes to turn inwards and stab the eye. 

A. taint 

B. infection 

C. contagion 

D. factor 

Answer: B. 

Q. 178 Select the most appropriate meanmg of the given idiom. Caught red-handed 

A. To discover a murderer using the clues 

B. To catch a thief who steals red paints 

C. To catch someone doing something illegal 

D. To try to arrest someone without any evidence 

Answer: C. 

Q. 179 Select the most appropriate meaning of the given idiom. Always a bridesmaid never a bride 

A. Someone who always comes second 

B. Someone who is born unlucky 

C. Someone who fulfills his or her potential 

D. Someone who does not deserve to win 

Answer: A. 

Q. 180 Select the wrongly spelt word. 

A. parakeets 

B. pupeteer 

C. puppies 

D. parapet 

Answer: B. 

Q. 181 Select the antonym of the given word. SUBDUED. 

A. subtle 

B. excited 

C. hushed 

D. motivated 

Answer: B

Q. 182 Identify the segnent in the sentence; which contains the grammatical error. 

That his body did not decompose confirmed what the Greeks thought about him and what Alexander use to believe about himself-that he was not an ordinary man, but a god. 

A. thought about him 

B. he was not an ordinary man 

C. use to believe about himself 

D. his body did not decompose 

Answer: C. 

Q. 183 Identify the segment in the sentence; which contains the grammatical error. 

When Alexander the Great died in Babylon in 323 B.C., his body didn’t begin to show signs of decomposition for a full six day, according to historical accounts. 

A. his body didn’t begin to show 

B. When Alexander the Great died 

C. signs of decomposition for a full six day 

D. according to historical accounts 

Answer: C. 

Q. 184 Select the wrongly spelt word. 

A. indicate 

B. innovate 

C. invite 

D. innundate 

Answer: D

Q. 185 Select the most approprlate Viord for the given group of words. excessive bureaucratic fuss 

A. footnotes 

B. red-tape 

C. fastidious 

D. officious 

Answer: B. 

Q. 186 Select the most appropriate word to fill in the blank. 

Moisture created ………. multitudes of visitors over the years has damaged King Tut’s tomb more than any vandalism and so conservators are working to …………… the historic monument …………. the impact of heavy-breathing, heavysweating tourists. 

A. by; guard; against 

B. by; dry; off 

C. from; dry; against 

D. from; guard; from 

Answer: A. 

Q. 187 Select the most appropriate word for the given group of words. a book that contains information on various subjects 

A. magazine 

B. omnibus 

C. guide 

D. encyclopedia 

Answer: D. 

Q. 188 Select the antonym of the given word. INDICTED. 

A. imprisoned 

B. examined 

C. hanged 

D. released 

Answer: D. 

Q. 189 Select the most appropriate word to fill in the blank. 

Scientists and politicians are ……….. becoming aware of another factor that could seriously threaten the ………… balance between production and consumption of food and climate change. 

A. duo; questionable 

B. alike; shaky 

C. both; tenuous 

D. alike; feeble 

Answer: C. 

Q. 190 Select the most appropriate option to substitute the underlined segment in the given sentence. If there is no need to substitute it, Select No improvement. 

The growing demand for cotton spurred the Industrial Revolution in Britain and inducing Southern planters in America to grows more cotton. 

A. No improvement 

B. inducing Southern planters in America grew more cotton 

C. induce Southern planters in America to grow more cotton 

D. induced Southern planters in America to grow more cotton 

Answer: D. 

Instructions: In the following passage some words have been deleted. Fill in the blanks with the help of the alternatives given. Select the most appropriate option for each number. Passage: 

Eating white bread and ready meals could be killing us, according to the first major study linking ‘fiilh‘a-processed” food with ………… (1) death. The study of 45,000 middle-aged people found that every 10 per cent increase in ……………. (2) of “ultra-processed food” was linked to a 14 per cent ……………. (3) risk of death within the next eight years. Previous research has linked consumption …………… (4) foods like white bread, ready meals, sausages, sugary cereals, fizzy drinks to a higher risk of high blood ………… (5) and cancer. 

Q. 191 Select the most approprlate option to fill in blank No.1. 

A. quiet 

B. early 

C. easy 

D. late 

Answer: B. 

Q. 192 Select the most approprlate option to fill in blank No.2. 

A. input 

B. digestion 

C. making 

D. intake 

Answer: D. 

Q. 193 Select the most approprlate option to fill in blank No.3. 

A. declined 

B. decreased 

C. improved 

D. increased 

Answer: D. 

Q. 194 Select the most approprlate option to fill in blank No.4. 

A. on 

B. in 

C. from 

D. of 

Answer: D. 

Q. 195 Select the most approprlate option to fill in blank No.5. 

A. coagulation 

B. clotting 

C. pressure 

D. infection 

Answer: C. 

Instructions For the following questions answer them individually 

Q. 196 Select the wrongly spelt word. 

A. elementary 

B. alleviasion 

C. olfactory 

D. election 

Answer: B. 

Q. 197 Identify the segment in the sentence, which contains the grammatical error. 

Hemophilia, a blood clotting disorder, can be passed about the maternal line vdthin families; men are more likely to develop it, while women are usually carriers. 

A. likely to develop it 

B. a blood clotting disorder 

C. can be passed about 

D. the maternal line 

Answer: C. 

Q. 198 Select the most appropriate option to substitute the underlined segment in the given sentence. If there is no need to substitute it, Select No improvement. 

As a result of various inventions during Victoria’s reign, travel and communication became better and improved sanitation techniques turn filthy streets in clean roads. 

A. turning filthy streets in clean roads. 

B. turn filthy streets into clean roads. 

C. No improvement 

D. turned filthy streets into clean roads. 

Answer: D. 

Q. 199 Select the synonym of the given word. DISPROPORTIONATELY 

A. unscientifically 

B. unreasonably 

C. unvaryingly 

D. unknowingly 

Answer: B. 

Q. 200 Select the antonym of the given word. ASSAULT 

A. attack 

B. retreat 

C. pacify 

D. affront 

Answer: B. 

×

Hello!

Click one of our representatives below to chat on WhatsApp or send us an email to info@vidhyarthidarpan.com

×